0% found this document useful (0 votes)
331 views269 pages

Mathematics - Phase 1

The document is a table of contents for a textbook on basic calculus. It includes sections on functions, differential calculus, integral calculus, and differential equations. Some key topics covered are functions and their domains/ranges, constant and variable quantities, elementary functions like polynomials and trigonometric functions, and an introduction to differential and integral calculus concepts like finding rates of change and antiderivatives. The document also lists exercises and solved problems at the end.

Uploaded by

Sarthak Anand
Copyright
© © All Rights Reserved
We take content rights seriously. If you suspect this is your content, claim it here.
Available Formats
Download as PDF, TXT or read online on Scribd
0% found this document useful (0 votes)
331 views269 pages

Mathematics - Phase 1

The document is a table of contents for a textbook on basic calculus. It includes sections on functions, differential calculus, integral calculus, and differential equations. Some key topics covered are functions and their domains/ranges, constant and variable quantities, elementary functions like polynomials and trigonometric functions, and an introduction to differential and integral calculus concepts like finding rates of change and antiderivatives. The document also lists exercises and solved problems at the end.

Uploaded by

Sarthak Anand
Copyright
© © All Rights Reserved
We take content rights seriously. If you suspect this is your content, claim it here.
Available Formats
Download as PDF, TXT or read online on Scribd
You are on page 1/ 269

PINNACLE-CBSE

BASIC CALCULUS
CONTENTS

 Introduction 1
 Functions 1

 Differential Calculus 3

 Exercise 1 6

 Exercise 2 8

 Behaviour of Functions 9

 Exercise 3 11

 Integral Calculus 11

 Exercise 4 14

 Definite Integral 15

 Differential Equations 16

 Exercise 5 17

 Answers to Exercises 18

 Solved Problems 19

 Chapter Practice Problems 30

 Assignment Problems 32

 Answers to Chapter 44
Practice Problems
 Answers to Assignment 45
Problems
Introduction:
Calculus is the study of the concepts of functions and their behavior. These concepts are based
on the theory of real numbers. Broadly, it is classified into two parts: Differential calculus and
Integral Calculus. Differential Calculus deals with finding the rate of change of one physical
quantity with respect to (written as w.r.t. from now onwards) another when the two are connected
by a functional relation.
Integral Calculus, on the other hand, deals with finding one physical quantity in terms of the other
(i.e., the functional dependence) given the rate of change of the first with respect to the second.

Variable and Constant

A variable is a quantity that takes on various numerical values. A constant is a quantity whose
numerical value remains fixed e.g. when a body falls towards the surface of the earth, its velocity
increases but the acceleration (due to gravity) remains constant.

It should be noted that when considering specific physical phenomena, it may happen that one
and the same quantity in one phenomena is a constant while in another it is a variable. For
example, the velocity in a uniform motion is constant, whereas the velocity in a uniformly
accelerated motion is variable.

Quantities that have the same value under all circumstances are called absolute constants. For
example, the ratio of the circumference of a circle to its diameter (an absolute constant ), period
of rotation of earth about its axis or around the sun, universal gravitational constant, permeability
of free space, etc.

Functions

In the study of natural phenomena and the solution of technical and mathematical problems, it is
necessary to consider the variation of one quantity as dependent on the variation of another. For
example, in studies of motion, the path traversed is regarded as a variable, which varies with
time. Here we say that the distance traversed is a function of time. The area of a circle, in terms
2
of its radius R, is R . If R takes on various numerical values, the area assumes different
numerical values. So the variation of one variable brings about a variation in the other. Hence
area of the circle is a function of its radius R.
If to each value of variable x (within a certain range) there corresponds a unique value of another
variable y, then we say that y is a function of x, or, in functional notation y = f(x). The variable x is
called the independent variables or argument and the variable y is called the dependent variable.
The relation between the variable x and y is called a functional relation. The letter 'f' in the
functional notation y = f(x) indicates that some kind of operation must be performed on the values
of x in order to obtain the corresponding values of y.

P-2022-CBSE-P1-MATHEMATICS-BASIC CALCULUS
2
f(x) L
y = f(x)
C f(x, y) =0
y3
y2 B
y2

y1 y1
A

x2
x1 x3 x x0 x
Fig. (a) Fig . (b)

These figures show the graph of two arbitrary curves. In fig.(a) any line drawn parallel to the y-axis
would meet the curve at only one point. That means each element of x would have one and only one
image. Thus fig (a) would represent the graph of a function.
In fig.(b) certain line (e.g. line L) would meet the curve in more than one points (A, B and C). Thus
element xo of x would have three distinct images. Thus this curve will not represent a function.
The set of all possible values which the independent variable (here 'x') is permitted to take for a
given functional dependence to be defined is called the domain of definition or simply the
domain of the function.
e.g. The function y = sin x is defined for all values of x. Therefore its domain of definition is the
infinite interval  < x < .
1
The function y = is defined for all x  1  its domain is (1,  ).
x 1

Range of a Variable
The set of all numerical values of a dependent variable quantity is called the range of the
variable. e.g. The range of values of the variable y = cos x for all possible values of x is the
interval [1, 1] i.e. –1  y  1.

Basic Elementary Functions


i) Constant function: y = c where 'c' is a constant, defined for all real x.
ii) Power function: y = x
(a)  is positive integer. The function is defined in the infinite interval   < x < .
(b)  is negative integer. The function is defined for all values of x except for x = 0.
iii) General exponential function: y= ax, where a is positive not equal to unity. This function
is defined for all values of x.
iv) Logarithmic function: y = logax, a > 0 but a  1. This function is defined for all x > 0.
v) Trigonometric functions: y = sinx, y =cosx are defined for all real x,

y = tanx, y = secx, are defined for R – (2n + 1) ,
2
y = cotx, y =coescex, are defined for R - n, where n  I.
It must be noted that in all of these functions, the variable x is expressed in radians. All
these trigonometric functions have a very important property that is Periodicity.

P-2022-CBSE-P1-MATHEMATICS-BASIC CALCULUS
3
vi) Algebraic functions:
(a) Polynomial function: y = a0xn + a1xn-1 + … + an, where a0, a1….an are real
constants (a0  0) and n is a non-negative integer, is called a polynomial of degree n.
e.g: y = ax + b, a  0 (a linear function),
2
y = ax + bx +c, a0 (a quadratic function),
A polynomial function is defined for all real values of x.
a0 xn  a1xn 1  .....  an
(b) Rational function: y = ; n, m  I+
m m1
b0 x  b1x  ....  bm
e.g: y = a/x (inverse variation).
The rational function is defined for all values of x except for those where the denominator
becomes zero.
(c) Irrational function:
2x 2  x
e.g. y= .
1  5x 2

Differential Calculus

Let y = f(x) be a function of x. Putting the values of 'x' in this relation, we obtain the corresponding
values of 'y'. Suppose we start putting some values of 'x' in increasing order. The respective
values of 'y' that we obtain may turn out to be in increasing order, or in decreasing order, or they
may remain constant, or they may even have a mixed trend, depending upon the type of function.
Let us take two values of x: x1 and x2 (x1 < x2), so that, y1 = f(x1) and y2 = f(x2).
 y  y1 
Then, the quantity  2  will tell us the average rate of change of y w.r.t. x in the
 x 2  x1 
interval [x1, x2].
y2  y1
Let y2 > y1  is positive  Function is increasing on an average.
x2  x1
y2  y1
If y2 < y1  is negative  Function is decreasing on an average.
x2  x1
y2  y1
If y2 = y1  is zero  Function is constant on an average.
x2  x1
 y  y1 
As you can see, if x1 and x2 are sufficiently far apart, the quantity  2  can not give the
 x 2  x1 
exact idea of the variation of y w.r.t. x in the interval [x1,x2]. It just provides an overall information.
For example, if y2 = y1, it does not necessarily mean that y is the same for all x in the interval
[x1,x2]. Thus, in order to obtain a "sufficiently accurate information", we have to choose x1 and x2
" sufficiently close" to each other. This 'sufficiently close' is the key word here. To know the rate of
change of y w.r.t. x at x = x1, we take x2 very near to x1 ( as much as possible), i.e, 'x2 tends to x1'
y  y1
and then calculate 2 . In the limiting case,
x2  x1

P-2022-CBSE-P1-MATHEMATICS- BASIC CALCULUS


4
we say that x2 nearly coincides with x1 and represent it as y
dy y  y1 y2 –y1
x2  x1. We use the notation for 2 as x2 x1.
dx x  x x2  x1 x2-x 1
1

'dx' means small change in x ( near x = x1) and


'dy' means the corresponding change in y.
dy x1 x2 x
We call the derivative or the differential coefficient of y w.r.t. x.
dx
(You can understand it physically by taking x as time and y as displacement of a body.
dy dy df(x)
Then denotes the magnitude of velocity). is also represented as f '(x) or .
dx dx dx
dy dy
Graphically, (i.e., computed at x=x1) denotes the slope of the tangent to the curve
dx x  x1 dx
y = f(x) at x = x1.
We will not derive here the formulae for f '(x) for various functions, but we give the results of the
derivations here.
Basic Differentiation Formulae
dy dy 1
y = constant  0 y = tan-1x  =
dx dx 1  x2
dy dy 1
y = xn  = nxn-1 y = cot-1x  =
dx dx 1  x2
dy dy 1
y= sinx  = cosx y = cosec–1 x  =
dx dx x x2  1
dy
y = cosx  = –sin x dy 1
dx y = sec –1 x  =
dy dx x x2  1
y = tanx  = sec2x
dx dy
y = ax  = ax ln a
dy dx
y = cotx  = – cosec2x
dx dy
y = ex   ex
dy 1 dx
y = sin-1x  =
dx 1  x2 dy 1
y = logax  
dy 1 dx x ln a
y= cos-1x  =
dx dy 1
1  x2 y = ln x  
dx x
Example of Velocity and Acceleration
Let the distance covered by a particle moving with constant acceleration a in time t be given by
1
s(t) = ut + at 2 along some line.
2
Then velocity at any time t will be the rate of change of s w.r.t. t.
ds
i.e. = u + at i.e. v(t) = u + at .
dt

Some Important Theorems


The following are very important theorems which can be applied directly:

P-2022-CBSE-P1-MATHEMATICS-BASIC CALCULUS
5
Theorem 1
dy df(x)
If a function is of the form y = k f(x), where k is a constant, then =k .
dx dx
Theorem 2
The derivative of the sum or difference of a finite number of differentiable functions is equal to the
sum or difference of the derivatives of these functions,
i.e. if y = u (x) + v(x) + w(x), then y ' = u'(x) + v'(x) + w'(x).

1
Illustration 1. Find the derivative of y = 3x  (x)1/3  .
x

Solution: Here we have y = u + v + w,


1/3 1
where u = 3x , v =x and w = . Hence we can use theorem 2, so that
x
1
dy 1 1 1 1 3 1 1 1
 3 x1/2-1  x 3  1.x 11 =   .
dx 2 3 2 x 3  x  2/3 x2

Theorem 3
The derivative of the product of two differentiable functions is equal to the product of the
derivative of the first function with the second function plus the product of the first function with
the derivative of the second function: i.e. if y = uv , then y' = u'v + uv'.
This formula can be extended for the derivatives of the product of any (finite) number of functions.

Illustration 2. Find the derivative of y = (a + x) ex w.r.t. x.

Solution: Using theorem 3,


dy d x d
 a  x  e  ex a  x 
dx dx dx
  a  x  e x  e x .1  e x  a  x  1 .

Theorem 4
u(x) dy u ' v  uv '
If y = , then y' =  .
v(x) dx v2

ax
Illustration 3. Find the derivative of y = w.r.t. x.
ax

Solution: Here, by theorem 4,


dy  1  a  x   1 a  x  a  x  a  x 2a
 2
 2
 2
.
dx a  x  a  x  a  x 
Theorem 5
If y = uv, where u and v are functions of x, then y' = vuv-1 u' + uv ln u.v'.

P-2022-CBSE-P1-MATHEMATICS- BASIC CALCULUS


6
Illustration 4. Differentiate the following functions w.r.t. x :
x x2
(i) y = x , (ii) y =  sin x  .

dy
Solution: (i) y = xx  x
 x.x x 1.1  x x ln x.1 = x (1+lnx).
dx
x2 dy 2 d 2 dx2
(ii) y =  sin x    x 2 (sin x )x 1 (sin x)  (sin x) x l n sinx
dx dx dx
2 2
= x 2 (sin x )x -1
cos x  (sin x)x (ln sinx).2x .

Derivative of a Composite Function


Given a composite function y = f(x), i.e. a function represented by
y = F(u), u =  (x) or y = F[(x)],
dy dF du
then y'=  .
dx du dx
This is called the chain rule. The rule can be extended to any number of composite variables;
dy df du dv
e.g. If y = f(u(v)), then y =  . . .
dx du dv dx

Illustration 5. Find the derivative of the following functions w.r.t. x :


(i) y = sinx2, (ii) y= (lnx)3,
(iii) y = sin(lnx)3, (iv) y = cos-1 (lnx).

Solution: (i) y = sin(x)2. Let u = x2  y = sin u


dy d d 2
  (sin u) (x )  cosu.2 x  2x cosx 2 .
dx du dx
dy d(u3 ) du 1 3
(ii) y = (ln x)3. Let u = ln x  y = u3    3u2  (ln x)2 .
dx du dx x x
(iii) y = sin(lnx)3 Let u = ln x, v = u3  y = sin v
dy dy dv du d (sin v) d(u)3 d
   (ln x)
dx dv du dx dv du dx
1 3(ln x)2
= cosv. 3u2 .  cos (ln x)3  .
 
x x
-1
(iv) Let u = ln x  y = cos u
dy 1 du 1 1 1
    .
dx 1  u2 dx 1  u2 x x 1  (ln x)2

Exercise 1.
i) Find the derivative of the following functions w.r.t. x :
7/2 5/2
(a) y = 6x + 4x – x, (b) y = tan (ln x),
2 x
(c) y = ln (x +1), (d) y = (sin x) .
2x dy dy
ii) (a) For y = 2 , find at x = 0. (b) If y = x 2  1 , find at x =  1.
x 1 dx dx
iii) Find the derivative of
x
(a) y = ax 2  2bx  c , (b) y = ( e ) x
(c) y = cos(cotx) (d) (sinx)cosx

P-2022-CBSE-P1-MATHEMATICS-BASIC CALCULUS
7
Parametric Representation of a Function and it’s Derivative
Y
We find the trajectory of a load dropped from an aeroplane moving
horizontally with uniform velocity v0 at an altitude y0. vO
We take the co-ordinate system as shown and assume that the
load is dropped at the instant the aeroplane cuts the y-axis. Since (x,y)
yO
the horizontal translation is uniform, the position of the load at any
time t, is given y
gt 2 X
x = v0t, y = y0 - .
2
These two equations are called the parametric equations of the trajectory because the two
variables x and y have been expressed in terms of the third variable ‘t’ (parameter)
i.e. two equations x =  (t), y =  (t)
where t assumes values that lie in a given interval (t1, t2).
dy dy / dt (d / dt)  '(t)
Then    .
dx dx / dt (d / dt)  '(t)

Illustration 6. The function y of x is given by, x = a cos t, y = a sin t. Find the derivative of y
w.r. t. x.

dy (a sint)' cos t
Solution:     cot t .
dx (a cost)'  sin t
dy 
If we want to compute at a particular t, say t = , then
dx 4
 dy  
 dx  =  cot  1 .
 x  / 4 4

Illustration 7. Find the slope of the tangent to the cycloid x = a(t-sint), y = a(1-cost), 0 t  2.

dy a(1  cos t) ' a sint 2 sin t/2cos t/2


Solution: We have   
dx  a(t  sin t)  ' a(1  cos t) 2 sin2 t / 2
 t 
= cot (t/2) = tan    .
 2 2

Derivative of an Implicit Function


If x and y are related by the rule F(x, y) = 0 such that y cannot be obtained entirely or exactly in
terms of x, then y is said to be an implicit function of x.
2 2 2
e.g. x + y = a  y =  a2  x2 .
Here we do not get a unique value of y for each x.
2 3 2
Another example can be x + y – 3x y = 0.
dy
Here also y cannot be obtained entirely in terms of x. To find in such cases start
dx
differentiating the given equation as it is (using rule of composite functions)
dy dy x
e.g., for x2 + y2 = a2, we have 2x + 2y 0   .
dx dx y

P-2022-CBSE-P1-MATHEMATICS- BASIC CALCULUS


8
For x2 + y3 – 3x2 y = 0,
2 dy  2 dy  dy 6xy  2x
we have 2x + 3y 3  x dx  y 2x   0  dx  .
dx   
3 y 2  x2 
If y can be expressed entirely in terms of x, then y is said to be an explicit function of x.

Second Derivative of a Function


dy
The second derivative of y w.r.t. x is the function obtained by differentiating w.r.t. x. It is
dx
d2 y
represented as or y  or f (x).
dx2
dy
e.g., if y = x5 then = 5x4
dx
d2 y d  dy  d
so that 2
dx
   
dx  dx  dx
 
5x 4  5.4x 3  20 x 3 .

The acceleration 'a' of a particle is the second derivative of the distance 's' (given as a function of
time).
ds d2 s dv
i.e., if s = f(t) then v =  f '(t) and a = 2   f ''(t) .
dt dt dt

d3y
Illustration 8. Find at x = /4, where y = sin2x.
dx 3

dy d2 y
Solution:  2 sin x cos x = sin2x   2 cos 2x .
dx dx2
d3 y
  4 sin2x
dx3
 d3 y 
  3  4
 dx  x  / 4

Exercise 2.
dy
i) Find if:
dx
(a) x = a cos3t, y = b sin3t (b) x = at2 , y= 2at.
d2y
II) Find if (a) y = enx, (b) y = ln x2
dx 2
dy
iii) Find if (a) x = y ln (xy), (b) y = cos(x + y).
dx
dy x .... 
iv) Find if y = xx .
dx

P-2022-CBSE-P1-MATHEMATICS-BASIC CALCULUS
9
Behaviour of Functions
The study of the flight of a shell in empty space yields a formula that gives the dependence of the
v 20 sin 2
range R upon the angle of elevation  and the initial velocity v0 i.e. R= , where g is the
g
acceleration due to gravity. We can now determine at what angle , the range R will be greatest
or least, and what must be the condition for the range to increase as the angle of elevation  is
increased.
In the oscillations of a load on a spring (of a railway car or automobile),the deviation y of the load
from the position of equilibrium is given by y = e-kt(A cos wt + B sin wt) where k, A, B, w (having
definite meaning) are considered constant. This formula gives the information as to what values
of the time t yield the maximum or minimum deviation and for what values of t, y will increase with
increasing t.

i) Increase and Decrease of Functions


Increasing Function:
If y = f(x) and x2 > x1 implies y2 > y1 for any pair x1, x2 belonging to the interval [a, b], then y is
said to be an increasing function of x.
f(x) increases in [a, b]  f (x) > 0 for all values of x in (a, b).
Decreasing Function:
If x2 > x1  y2 < y1 for any pair x1, x2 belonging to [a, b], then y is said to be a decreasing
function of x.
f(x) decreases in [a, b]  f(x) < 0 for all values of x in (a, b).
For a constant function, f(x) = 0.
For a non-decreasing function, f(x)  0.
For a non-increasing function, f(x)  0.

Illustration 9. Find the intervals of increase and decrease of the function y =x4.

Solution: y = x4  y' = 4x3.


For x > 0, y' > 0  the function increases in (0, ).
For x < 0, y' < 0  the function decreases in (, 0).
3 2
Illustration 10. Separate the intervals in which f(x) = 2x  15x + 36x + 1 is increasing or
decreasing.

Solution: We have f (x) = 6x2  30x + 36 = 6 (x  2) (x  3).


Thus for x < 2, f(x) > 0,
over 2 < x < 3, f(x) < 0 and for x > 3, f(x) > 0.
Hence the given function is increasing in (, 2) and (3, ), and decreasing
in (2, 3).

ii) Maxima and Minima of Functions:


A function 'f' is said to have a maximum at x = xo if f(x) < f(x0) for all x in the immediate
neighborhood of x0.

P-2022-CBSE-P1-MATHEMATICS- BASIC CALCULUS


10
Similarly, a function 'f' is said to have a minimum at x = x0 if f(x) > f(x0) for all x in the
immediate neighborhood of x0
We have used the word 'immediate' here because a given function may have any number of
'high' and 'low' points. It is just like moving on an uneven surface (which has many 'bumps'
and 'depressions'). Mathematically, these bumps are called the points of local maxima and
the depressions are called the points of local minima. The highest of all the bumps is the
global maximum and the lowest of all the depressions is the global minimum.
We state here the preliminary methods only to find the maxima and minima of functions:
dy
(a). Find or f'(x) .
dx
dy
(b). Find the points at which becomes zero. These points are called critical points.
dx

To Find the Points of Maxima and Minima, We Resort to Either of the Following Tests:
A) First Derivative Test:
Suppose x = x0 is a critical point i.e. f (x0) = 0.
If f(x) changes sign from positive to negative in the neighborhood of x = x0
 Local maximum at 'x0'.
If f(x) changes sign from negative to positive in the neighborhood of x = x0
 Local minimum at 'x0'.

B) Second Derivative Test:


d2 y
(i) Find or f ''(x) .
dx2
(ii) Compute the value of f''(x) at the critical points.
If it is positive  Minima at those values of x.
If it is negative  Maxima at those values of x.
If the function is defined in an interval [a, b], then to find the maximum and minimum values
(i.e., global maximum and global minimum) of the function in that interval we compare the
values of the function (i.e., y) at all the critical points and also at the end points (i.e., f(a) and
f(b)). Then the largest among them gives the global maximum value and smallest gives the
global minimum value.
4
Illustration 11. Test y = 1  x for maximum and minimum.

Solution: Here y =  4x3 = 0 for x = 0


 x = 0 is the critical point.
Now y=  12 x2 = 0 at x = 0.
It is thus impossible to determine the character of the critical point by means of
the sign of the second derivative. Thus we investigate the character of the given
function in an interval containing the point x = 0.
For x < 0, y > 0  the function is increasing for x < 0
For x > 0, y < 0  the function is decreasing for x > 0.
Consequently, at x = 0, the function has a maximum i.e. y x=0 = 1 and minimum
does not exist.

P-2022-CBSE-P1-MATHEMATICS-BASIC CALCULUS
11
3
Illustration 12. Determine the maximum and minimum values of the function y = x  3x + 3 on the
interval [ 3, 3/2].

2
Solution: For the given function, y = 3x  3.
2
For the critical points, 3x  3 = 0  x =  1.
Then y = 6x > 0 at x = 1,
<0 at x=  1.
Hence there is a local maximum at x =  1 at which y =  1 + 3 + 3 = 5.
Also there is a local minimum at x = 1, y = 1  3 + 3 = 1.
Now at x = 3, y =  27 + 9 + 3 = 15
and at x = 3/2, y = 15/8.
Hence the minimum value of the given function is  15 at x = 3 and the
maximum value is 5 at x = 1.

Exercise 3.
i) Find the intervals of increase and decrease of the following functions
(a) y = 4x3 - 21x2 + 18x +20, (b) y = sinx , 0  x  .
ii) Find the points of local maxima and minima of the following functions:
x3
(a) y =  2x2  3x  1 , (b) y = (x 2) (x 3).
3
iii) Find the maximum and minimum values of x + sin 2x in [0,  ].
iv) Find the maximum and minimum values of sinx + cosx in [0, 2].

INTEGRAL CALCULUS
The Anti derivative of a Function:
A function F(x) is called the antiderivative of the function f(x) if f(x) = F'(x).

x3  x3 
2
For example, the antiderivative of the function f(x) = x is , as   = x2. The functions
3  3 
x3 x3 2 x3
 2 and  1 are also antiderivatives of f(x) = x . Infact,  C , where C is an arbitrary
3 3 3
constant, is the antiderivative of x2 . So if a function f(x) possesses an anti-derivative F(x), then it
possesses infinitely many antiderivatives, all of them being contained in the expression F(x) + C,
where C is a constant.
If the function F(x) is an antiderivative of f(x), then the expression F(x) + C is called the indefinite
integral of the function f(x) and is denoted by the symbol f(x)dx. Thus, by definition,
 f(x) dx = F(x) + C, if F'(x) = f(x). The process of finding the antiderivative of a function f(x) is
called integration. Two different integrals of a given function differ by a constant.

Standard Elementary Integrals:


In the following integrals, c stands for an arbitrary constant.
xn 1
 xndx   c, n  1  tan x dx =  log|cosx| + c = log|secx| + c
n 1
( f ( x )n 1  cot x dx = log|sin x| + c =  log|cosecx| + c
 ( f ( x )n f ' ( x )dx   c n  1
n1  secxdx = log|secx+tanx|+c

P-2022-CBSE-P1-MATHEMATICS- BASIC CALCULUS


12
1 dx 1 x
a tan1  c
 x dx  log | x | c 2
x 2

a a
x x dx
 e dx  e  c.  1 x 2
 tan1 x  c

 sin x dx =  cosx + c 
dx
 sin1
x x
 c or  cos1  c
a2  x 2 a a
 cos x dx = sinx + c dx
2
  sin1 x  c or  cos1 x  c
 sec x dx = tanx + c 1 x 2

2 dx
 cos ec x dx =  cotx + c  2
 sec 1 x  c or  cos ec 1x  c
x x 1
1
x1/ 21 2
x2 + c = . x3/2 + c,
e.g (i)
 dx =
1
1
3
2
1 x 2 1 1
(ii)
2  dx = +c= x2 dx =
+ c.
x  2  1 x
The following points are to be noted:
1

x 
dx = log x + c if x is positive

= log (x) + c if x is negative


d 1 1 1
because
dx
(log (x)) =
x
(1) =
x

x
dx = log x + c. 
1
dx = sin–1x or  cos–1 x. It does not mean that sin–1x =  cos–1 x.

 1 x 2

The only legitimate conclusion is that they differ by some constant. In fact
sin-1x – (-cos –1 x ) = sin –1x + cos –1 x =  /2 .

 If a is a constant, then
 a f(x) dx = a
 f(x) dx.


 [ f(x)  g(x)] dx, =  f(x) dx   g(x) dx.

Illustration 13. Evaluate


2  2 x 
(i)  (a0 + a1x + a2x ) dx, (ii)   cos x  x  e  dx,

x2 x4
(iii) 1  x 2
dx, (iv) x 2
1
dx.

2 2
Solution: (i)  (a0 + a1x +a2x )dx = a0  dx + a1  x dx + a2  x dx

x2 3
= a0 x + a1 + a2 x + c.
2 3
 2 x  1 x
(ii)   cos x   e  dx = cos x dx + 2
 x   x dx 
e dx

= sin x + 2 log x  ex + c.

P-2022-CBSE-P1-MATHEMATICS-BASIC CALCULUS
13
2 2
x x  1 1  1 
(iii)  1 x 2
dx =  1  x2
dx =  1  1  x 2  dx

dx
=  dx   1 x 2

= x  tan-1 x + c.

x4 x4  1  1  2 1
(iv) x 2
1
dx =  1 x 2
dx =   x  1  dx
1 x  2

dx x3
=  x2 dx 
 dx +  1 x 2
=  x + tan –1 x + c.
3
Methods of Integration:
1. Integration by substitution

This method consists of expressing the integral  f(x) dx, where x is the independent variable,
in terms of another integral where some other variable, say 't', is the independent variable; x and t
being connected by the relation x =  (t). i.e ,  f(x)dx =  f[φ(t)] φ '(t) dt.
This method is useful only when a relation x = (t) can be so selected that the new integrand
dx
f(x) is of a form whose integral is known.
dt

Illustration 14. Integrate the following w.r.t. x:


x3
(i) sin2 x cosx, (ii) .
1  x8

Solution: (i) Let sinx = t  cos x dx = dt


t3 sin3 x
sin2 x cos x dx = t 2 dt =
  3
+c=
3
+ c.
1
(ii) Let x4 = t  4 x3 dx = dt  x3 dx = dt
4
x3 1 1 dt 1 1
  dx = dt =   = tan –1 t + c = tan –1 x4 + c. 
1 x 8
4 1 t 2 4 1 t 2 4 4 
1
Illustration 15. Evaluate  a  x2
2
dx .

Solution: Let x = a sin  dx = a cos d and a2 – x2 = a2 cos2


dx acos θdθ x
    1.dθ    c  sin1  c.
 
2
a x 2 acos θ a

dx 1 1 bx  k
Note:  a2   bx  k 
2

b
sin
a
+c

P-2022-CBSE-P1-MATHEMATICS- BASIC CALCULUS


14
dx 1 dx
e.g.  2
2x  3x  4

2  41 
2 2
 3
    x  
 4   4
1 4x  3
 sin1 + c.
2 41

2. Integration by parts
d du dv
If u, v be two function of x, we have uv   v  u .
dx dx dx
Integrating both sides, we get
dv du dv du
uv = u dx
dx  v
dx 
dx 
 u dx .dx = uv   v. dx .dx .
dv
Let u = f(x) and  (x)
dx
 df 
  f(x)φ(x)dx  f(x) φ(x)dx     dx  (x)dx  dx .
Integral of the product of two functions = first function  integral of second  integral of
(derivative of first  integral of second).

Illustration 16. Evaluate


x ex dx, sin-1x dx.
(i)
 (ii)

Solution: (i) Let f(x) = x, (x) = ex
  xex dx = x.ex   1.ex dx = xex – ex + c.

(ii) Let f(x) = sin-1x, (x) = 1 sin-1x dx = sin-1x. 1 dx


 
1
= sin-1x.x 
 1  x2
x dx

= x sin-1 x   x(1 x2)-1/2 dx

= x sin1 x + 1  x2  c .

Exercise 4.
Integrate the following functions w.r.t x :
1/3 1/2 3/5 2
i) (a) x +x +x (b) 5 cos x  3 sin x + ,
cos 2 x
3x2
ii) (a) (b) sin4x cos x
1  x3
2 1
iii) (a) x sinx (b)
x log x
iv) (a) x2 . ex (b) lnx

P-2022-CBSE-P1-MATHEMATICS-BASIC CALCULUS
15
Definite Integral
The difference in the values of an integral of a function f(x) for two assigned values of the
independent variable x, say a, b, is called the definite integral of f(x) over the interval (a,b) and is
b b
denoted by 
a
f(x) dx. Thus  f(x) dx = F(b) – F(a), where F(x) is the anti derivative of f(x).
a
Or,

b
we write  f(x) dx= F(x) ba = F(b)  F(a). a is called the lower limit and b, the upper limit of
a

integration.
Note:
b a


a
f(x) dx =   f(x) dx
b
b c b


a
f(x) dx = a
f(x) dx +  f(x) dx, where c is any point inside or outside the interval (a, b).
c
b
 Geometrically, the definite integral  f(x) dx denotes the area bounded by the curve
a

y = f(x), the axis of x and the two ordinates x = a and x = b.


Also, work done on a body as it is displaced from the position x = a to x = b is equal to
area under the force – displacement graph bounded by x = a & x = b
b


i.e., W = F .dx ; where F is the force applied to the body.
a

Illustration 17. Find the area of the triangle bounded by the sides y = x, y = 0 and x = 2.

Solution: Area in question is enclosed by the curve y


y = x and the axis of x and the two ordinates x y=x
2
= 0, and x = 2. The required area =  y dx
0
2 2 2
x 
=  x dx =  2 
0 0
= 2 sq. units.
o 2
x

/2
cos x
Illustration 18. Evaluate  1  sin
0
2
x
dx .

Solution: Let sin x = t  cosx dx = dt.


When x = 0 , t = 0 and for x = /2 , t = 1
/2 1
cos xdx dt 1
  1  sin 2
x
  1 t 2 
 tan1 t  0
0 0
 
= tan–1 1  tan10 = 0  .
4 4

P-2022-CBSE-P1-MATHEMATICS- BASIC CALCULUS


16
Differential Equations
An equation consisting of the independent variable, the dependent variable and the derivatives of
the dependent variable with respect to the independent variable is called a Differential Equation.
For example:
dy d2 y
+ y cosx = sin x; 2
 a2 x  0;
dx dx
3/2
dy   dy 2  d2 y
dx

 2x  3y   7x 2  8y  0 ,  1    
  dx  

dx 2
.

The order of a differential equation is the order of the highest derivative occurring in it. A solution
or integral of a differential equation is a relationship between the dependent and the independent
variables, not involving the differential coefficients such that this relation and the derivatives
obtained from it satisfy the given differential equation. This also implies that a differential equation
can be derived from its solution by the process of differentiation and elimination.
Illustration 19. Show that y = A cosx + B sinx is a solution of the differential equation
d2y
y 0.
dx 2
dy d2 y
Solution: We have =  A sinx + B cosx and   A cos x  B sin x  y
dx dx2
d2 y
  y  0.
dx2

How to solve a differential equation:


We will consider the first order linear differential equation. In order to solve a differential equation
we need to separate the variables so that we can integrate. The general form of the differential
equation shall be or shall be brought to the form f(x) dx + (y) dy = 0
or f(x) + (y) dy/dx = 0. ... (1)
Integrating with respect to x, we get
dy
 
f(x)dx  (y)
dx
dx  c

or  f(x)dx +  (y)dy = c which is the solution of (1).

Illustration 20. Solve (xy2 + x) dx + (yx2 + y) dy = 0.


2 2
Solution: We have x(1 + y )dx + y(1+ x )dy = 0
x y
or 2
dx  dy  0
1 x 1  y2
x y
so that 
1 x 2
dx  
1  y2
dy  c

1 1
 log(x 2  1)  log(1  y 2 )  c
2 2
or (x2 + 1) (y2 + 1) = e2c = a, is the required solution containing arbitrary
constant 'a'.

P-2022-CBSE-P1-MATHEMATICS-BASIC CALCULUS
17

Exercise 5.
i) Evaluate the following:
 /2 1 4
2 1 x
(a) 
0
sin x cos x dx , (b)  sin
0
x dx , (c) e
1
dx .

x
ii) Find the area enclosed by the curve y = e , the x-axis and the two ordinates x=  1
and x = 1.
dy  dy 
iii) Solve (a) = cot x .coty, (b) ln   = x+ y.
dx  dx 
iv) Solve differential equation, sec2x tany dx + sec2y tanx dy = 0.

P-2022-CBSE-P1-MATHEMATICS- BASIC CALCULUS


18
ANSWERS TO EXERCISES

Exercise 1:
(i). (a) 21x5/2 + 10 x3/2  1 (b) (1/x). sec2 (ln(x)
(c) 2x/[(x2 +1) ] (d) (sinx)x [ln (sinx) + x cotx]
1
(ii) (a) 2 (b)
2
ax  b xx x
(iii) (a) (b)  e  . x . (1 + log x)
2
ax  2bx  c
2 cosx
(c) sin(cotx) cosec x (d) (sinx) {cotx cosx  sinx ln(sinx)}

Exercise 2:
b 1
(i). (a)  tan t (b)
a t
2
(ii). (a) n2 enx (b) 
x2
xy  sin  x  y 
(iii). (a) (b)
x 1  ln xy  1  sin  x  y 
y2
(iv)
x 1  ln y 

Exercise 3:
1   1
(i). (a) Decreasing in  ,3  , Increasing in  ,   (3,)
2   2 
π 
(b) Increasing in [0, /2], decreasing in  , 
2 
(ii). Maxima Minima
(a) x = 1 x=3
(b)  x = 5/2
(iii) 0,  (iv) 2,  1

Exercise 4:
3 4/3 2 3/2 5 8/5
(i) (a) x  x  x c (b) 5 sin x + 3 cos x + 2 tan x + c
4 3 8
3 sin5 x
(ii). (a) log |(1+x )| + c (b) +c
5
(iii). (a)  x2 cosx + 2x sinx + 2 cosx + c (b) ln |ln x| + c
(iv). (a) (x2  2x + 2)ex + c (b) x(lnx  1) + c

Exercise 5:
1  2
(i). (a) (b) 1 (c) 2e
3 2
1
(ii). e
e
(iii). (a) sin x = c sec y (b) ex + e–y = c
(iv) tanx tany = c

P-2022-CBSE-P1-MATHEMATICS-BASIC CALCULUS
19
SOLVED PROBLEMS

Subjective:

Level – 0

dy
Problem 1. If y  x sin x cos x. , find .
dx

Solution: For y  x sin x cos x.


dy ' ' '
dx
= x   sinxcosx  x  sin x  cosx  x sinx  cosx 

1
= x 1/ 2 sinxcosx  x cos x. cos x  x sinx  -sinx 
2
1 1 1
= x 1/ 2
2 2

sin 2x  x cos2 x  sin2 x =
4 x
 sin2x  x.cos2x .

x3
Problem 2. Differentiate w.r.t. x .
cos x

x3 dy (x3 ) ' (cosx) - x3 (cosx) ' 3x2 cosx  x 3 sinx


Solution: Let y = so that y' =   .
cos x dx cos2 x cos2 x

dy
Problem 3. If x = 2 ln cot t and y = tan t + cot t, find .
dx

dy dy/dt
Solution: We have  . . . (1)
dx dx/dt
dx cosec 2 t 2 4
where 2  
dt cot t cost sin t sin 2t
dy sin2 t  cos2 t cos2t
and  sec 2 t  cosec 2 t  2 2
 4.
dt sin t cos t sin2 2t
dy  4cos2t   sin2t  cos2t
Hence     cot. 2t .
dx  sin2 2t    4   sin2t

1/2 3/2
Problem 4. Evaluate 3x (1 + x ) dx.

Solution: Here 3x1/2(1 + x3/2) is the derivative of (1 + x3/2)2


3/2 2
 I = (1 + x ) + c.

P-2022-CBSE-P1-MATHEMATICS- BASIC CALCULUS


20
Level – I
x 1
Problem 5. Find the maximum and the minimum values of y = on the interval 0  x  4.
x 1

dy
Solution: For critical points =0
dx
1.(x  1)  1(x  1)
 0
(x  1)2
2
 0
(x  1)2
2
 No such value of x exists. Infact > 0 in the given interval.
 x  12
That means y is entirely increasing in [0, 4].
Hence there is no critical point in the interval 0  x  4. At x = 0, y = 1, and at
x = 4, y = 3/5.
Hence the function is minimum at x= 0, with y = 1 and is maximum at x = 4 with
y = 3/5.

Problem 6. Find the maximum and the minimum values of f (x) = 2x3 –24x + 107 in the
interval [1, 3].

Solution: We have f (x) = 2x3  24x + 107


so that , f (x) = 6x2  24.
Now, f(x) = 0  6x2 –24 = 0  x =  2.
But x =  2  [1, 3]. Hence x = 2 is the only stationary point.
Now, f (1) = 2  24 + 107 = 85, f (2) = 2(2)3  24 (2) + 107 = 75.
And, f (3) = 2(3)3  24  3 + 107 = 89.
Hence, the maximum value of f (x) is 89 which is attained at x = 3 and the
minimum value is 75 which is attained at x = 2.

dy
Problem 7. Solve  e x  y  x 2 e y .
dx

dy e x x 2 y 2 x
Solution: We write   as e dy = (x + e )dx.
dx e y e y
x3
Integrating both sides, we get ey =  e x  c which is the solution of the above
3
differential equation, where c is an arbitrary constant.

2
Problem 8. Evaluate x dx .
1
2 0 2
Solution: Let I = x dx = x dx  x dx .
1 1 0
b c b
By property,  f  x  dx   f  x  dx  f  x  dx , so that

a a c

P-2022-CBSE-P1-MATHEMATICS-BASIC CALCULUS
21
0 2
I=   x dx   x dx (since |x| = −x for x < 0 and |x| = x for x  0)
1 0
0 2
 x2   x2   1  4  5
=− 
 2     = −  0      0   .
  1  2  0  2  2  2

dy
Problem 9. Find , when x = a ( + sin ), y = a (1 + sin )
dx

dx dy
Solution: We have = a (1 + cos ) and = a cos 
d d
dy
dy d a cos  cos 
    .
dx dx a(1  cos ) 1  cos 
d

dy
Problem 10. Find , when x = et(sin t + cot t), y = et log t.
dx

dx
Solution: Here = et (cos t  sin t) + et (sin t + cos t) = 2et. cos t,
dt
dy 1  1
and  et log t  et  et  log t   .
dt t  t

Level – II

sin 3 x  cos 3 x
Problem 11. Evaluate  sin 2 x cos 2 x
dx.

sin3 x  cos3 x sin3 x cos3 x


Solution: We have  2
sin x cos x 2
dx =  sin 2
x cos x2
dx +  sin
2
x cos2 x
dx

=
 tan x sec x 
dx + cot x cos ecx dx = sec x – cosec x + c.

x
cos e x dx.
Problem 12. Evaluate e
dx
Solution: Let ex = t  ex = 1  dx = e– x. dt
dt
x
cos e x dx = x x x
 e  e .cos e .e dt =  cos t dt
x
= sin t + c = sin e +c.
1
2 2x
Problem 13. Evaluate x e
0
dx .

Solution: Integrating by parts we have


1 1 1 1
 e2x  e2x e2
 x e dx   x2
2 2x
  2x  dx   xe2x dx 
 2 2 2
0  0 0 0

P-2022-CBSE-P1-MATHEMATICS- BASIC CALCULUS


22
1 1 1
e2  e2x  e2x e2 e2 1 2x
=  x
2  2
  1
0 0 2
dx  2
 
2 2
0
e dx 
1
1  e2x  1  e2 1  e2  1
=       .
2  2 0 2  2 2  4
x -x
Problem 14. Find the differential equation if y = ae + be .
Solution: Given y = aex + be-x
dy
so that on differentiation  aex  be  x ……(1)
dx
Again, differentiating equation (1)
d2 y x x d2 y
 ae  be  y.
dx2 dx 2

dy sin 2 ( a  y )
Problem 15. If sin y = x sin (a + y), prove that  .
dx sin a

sin y
Solution: The given equation can be written as x = so that
sin(a  y)
dx sin(a  y)cos y  sin y cos(a  y)

dy sin2 (a  y)
sin(a  y  y) sin a
= 2
 2
sin (a  y) sin (a  y)
2
dy sin (a  y)
  .
dx sin a
Alternate: Differentiate the given equation with respect to x. We get
dy dy
cos y = sin (a + y) + x cos (a + y)
dx dx
dy sin(a  y)
 
dx cos y  x cos(a  y)
sin2 (a  y) sin2 (a  y)
=  .
cos y sin(a  y)  sin y cos(a  y) sin a

dx
Problem 16. Evaluate I   sin 2
x.cos2 x
.

Solution: Transform the integrand in the following way.


1 sin2 x  cos2 x d
2 2
 2 2
 sec 2 x  cos ec 2 x =  tan x  cot x 
sin x cos x sin x cos x dx

  sec 
2
Hence I = x  cos ec 2 x dx = tanx – cotx + c.

dy et  log t  1 
dy  t  t log t  1

Hence  dt   .
dx dx 2e t cos t 2t  cos t
dt

P-2022-CBSE-P1-MATHEMATICS-BASIC CALCULUS
23

Objective:

Level – 0
True / False

1 t2 2at dy a(t 2  1)
Problem 1. If x  and y  , then 
1 t2 1  t2 dx 2t

Solution: True
1 t2 2at
x 2
and y 
1 t 1  t2
Differentiating with respect to t, we get
dx (1  t 2 )(0  2t)  (1  t 2 )(0  2t) 4t
 2 2

dt (1  t ) (1  t 2 )2
dy (1  t 2 )2a  2at(2t) 2a(1  t 2 )
and  
dt (1  t 2 )2 (1  t 2 )2
dy dy / dt a(1  t 2 ) dy a(t 2  1)
   ;   .
dx dx / dt 2t dx 2t

 t dy
Problem 2. If x  a  cos t  logtan  ,y  a sin t, then  tan t
 2  dx

Solution: True
 t
Given that x  a  cos t  logtan  and y  a sin t . Differentiating with respect to t,
 2
dy
we get  a cos t
dt
.....(i)
dx   t  1   t  
and  a   sin t  cot    sec 2   
dt   2  2   2 
 1  cos2 t
 a   sin t  a  a cos t cot t .....(ii)
 sin t  sin t
dy
From (ii) and (i), we get  tan t .
dx

1 4 2 dy 2 2
Problem 3. If y  u ,u  x 3  5 , then  x (2x3  15)3
4 3 dx 27

Solution: True
dy dy du
 .  u3 .2x 2
dx du dx
3
2  2 2
  x3  5  .2x 2  x (2x3  15)3 .
 3  27

P-2022-CBSE-P1-MATHEMATICS- BASIC CALCULUS


24
1
Problem 4. The differential coefficient of f[log(x)] when f(x)  log x is
xlog x

Solution: True
 f(x)  log x ;  f[log x]  loglog x
1 d 1
f '[log x]  . log x  .
log x dx xlog x

 5x  1  dy  5x  1  dy  5x  1 
Problem 5. If y  f  2  and f '(x)  cos x , then dx  cos    
 10x  3   10x2  3  dx  10x2  3 

Solution: True
5x  1
Suppose that t  , so y  f(t)
10x 2  3
dy dt
  f (t). [Since f '(x)  cos x ]
dx dx
dy  5x  1  d  5x  1 
 cos    .
dx  10x 2  3  dx  10x 2  3 

Fill in the Blanks

 dy
Problem 6. If x  2 cos t  cos 2t , y  2sin t  sin 2t , then at t  ,  _______
4 dx

Solution: 2 1
dx dy
 2sin t  2 sin2t and  2cos t  2cos 2t
dt dt
dy cos t  cos 2t
 
dx sin 2t  sin t
  dy 
Put t  , we have  
4  dx  t  /4
cos  / 4  cos  / 2
  2  1.
sin  / 2  sin  / 4

dy
Problem 7. If x 3  8xy  y3  64 ,then  _______
dx

3x 2  8y
Solution: 
8x  3y 2
 dy  dy
x 3  8xy  y3  64  3x2  8  y  x   3y2 0
 dx  dx
dy 3x 2  8y
  .
dx 8x  3y 2

dy
Problem 8. If ax 2  2hxy  by 2  2gx  2fy  c  0 , then  _______
dx

P-2022-CBSE-P1-MATHEMATICS-BASIC CALCULUS
25
ax  hy  g
Solution: 
hx  by  f
ax 2  2hxy  by 2  2gx  2fy  c  0
Differentiating w.r.t. x of y, we get
 dy  dy dy
2ax  2h  y  x   2by  2g  2f 0
 dx  dx dx
dy
 (2hx  2by  2f )  (2ax  2hy  2g)
dx
dy (ax  hy  g)
or  .
dx (hx  by  f )

Level – I

MCQ Single Correct

dy
Problem 9. If cos(x  y)  y sin x, then 
dx
sin(x  y)  y cos x sin(x  y)  y cos x
(A)  (B)
sin x  sin x  y) sin x  sin(x  y)
sin(x  y)  y cos x
(C)  (D) None of these
sin x  sin x  y)

Solution: A
cos(x  y)  (y sin x)
 dy  dy
  sin(x  y)  1    y cos x  sin x dx
 dx 
dy y cos x  sin(x  y)
  .
dx sin(x  y)  sin x

dy
Problem 10. x 1  y  y 1  x  0 , then 
dx
(A) 1  x (B) (1  x)2
(C) (1  x)1 (D) (1  x)2

Solution: D
x 1  y  y 1  x  0  x 2 (1  y)  y 2 (1  x)
 d2 s 
 2   8  x  y  xy  0,  x  y
 dt t 1/2
dy 1
  .
dx (1  x)2

Problem 11. The derivative of F[f {(x)}] is


(A) F [f { (x)}] (B) F[f { (x)} ] f {(x)}
(C) F[f { (x)}] f {(x)} (D) F [f { (x)}] f {(x)}  (x)

P-2022-CBSE-P1-MATHEMATICS- BASIC CALCULUS


26
Solution: D
y   F[f {(x)}] f  {(x)} (x) .

Problem 12. Let f(x)  e x , g(x)  sin1 x and h(x)  f(g(x)), then h'(x) / h(x) 
1
(A) esin x (B) 1/ 1  x 2
(C) sin1 x (D) 1/(1  x 2 )

Solution: B
f(x)  e x and g(x)  sin1 x and h(x)  f(g(x))
1
 h(x) = f(sin1 x)  esin x

1
 h(x)  esin x
1 1 h(x) 1
 h(x)  esin x
.   .
1 x 2 h(x) 1 x2

1
Problem 13. If f(x)  , then the derivative of the composite function f[f {f(x)}] is equal to
1 x
1
(A) 0 (B)
2
(C) 1 (D) 2

Solution: C
1 1 x
f(x)   f {f(x)} 
1 x x
x
 f[f {f(x)}]  x
x  1  x
 Derivative of f[f {f(x)}]  1.

Level – II

MCQ Single Correct

1 1 dy
Problem 14. If x 2  y 2  t  , x 4  y 4  t 2  2 , then x 3 y 
t t dx
(A) 1 (B) 2
(C) 3 (D) 4
Solution: A
2
 1
x 4  y 4   t    2  (x2  y 2 )2  2
 t
1
 x 2 y 2  1  y 2   2
x
dy 2 dy
Differentiating, we get 2y  3 or x 3 y  1.
dx x dx

2t 2t dy
Problem 15. If tan y  2
and sin x  2
, then 
1 t 1 t dx
2 1
(A) (B)
1  t2 1  t2
(C) 1 (D) 2

P-2022-CBSE-P1-MATHEMATICS-BASIC CALCULUS
27
Solution: C
2t
tan y  .....(i)
1 t2
2t
and sin x  .....(ii)
1 t2
From (i), differentiating w.r.t. t of y, we get,
dy 1 cos( sin x  cos x ) dy 2(1  t 2 ) 1
 (cos x  sin x) and  .
dx 2 sin x  cos x dt (1  t ) (1  tan2 y)
2 2

dy 2(1  t 2 ) 1 2
or  .  .....(iii)
dt (1  t 2 )2   2t 2  1  t 2
1   2 

  1  t  
and from (ii), differentiating w.r.t. t of x, we get
dx 2(1  t 2 )
cos x 
dt (1  t 2 )2
dx 2(1  t 2 ) 1 2
or   .....(iv)
dt (1  t 2 )2 (2t) 2 1  t2
1
(1  t 2 )2
dy
Hence  1.
dx

1 t2 2t dy
Problem 16. If x  2
and y  2
, then 
1 t 1 t dx
y y
(A) (B)
x x
x x
(C) (D)
y y

Solution: C
1 t2 2t
x and y 
1 t2 1  t2
Put t  tan  in both the equations, we get
1  tan2  2 tan 
x 2
 cos 2 and y   sin2 .
1  tan  1  tan2 
Differentiating both the equations, we get
dx dy
 2sin2 and  2cos 2.
d d
dy cos2 x
Therefore   .
dx sin 2 y

d2 y
Problem 17. If x  at 2 ,y  2at , then 
dx2
1 1
(A)  (B)
t2 2at 3
1 1
(C)  3 (D) 
t 2at 3
Solution: D

P-2022-CBSE-P1-MATHEMATICS- BASIC CALCULUS


28
dy dy / dt 2a dy 1 2a
    
dx dx / dt 2at dx t y
2
dy d2 y  dy 
 y 2a  y 2     0
dx dx  dx 
2 2
d y (dy / dx) 1
 2   .
dx y 2at 3

dy
Problem 18. If sin y  x sin(a  y), then 
dx
sin2 (a  y) sin2 (a  y)
(A) (B)
sin(a  2y) sin(a  2y)
sin2 (a  y) sin2 (a  y)
(C) (D)
sina cosa
Solution: C
sin y
sin y  x sin(a  y)  x 
sin(a  y)
dy dy
cos y. .sin(a  y)  sin y cos(a  y)
 1 dx dx
sin2 (a  y)
dy
.sin(a  y  y)
dy sin2 (a  y)
 dx 2   .
sin (a  y) dx sina

dy
Problem 19. If tan(x  y)  tan(x  y)  1, then 
dx
sec 2 (x  y)  sec 2 (x  y) sec 2 (x  y)  sec 2 (x  y)
(A) (B)
sec 2 (x  y)  sec 2 (x  y) sec 2 (x  y)  sec 2 (x  y)
sec 2 (x  y)  sec 2 (x  y)
(C) (D) None of these
sec 2 (x  y)  sec 2 (x  y)

Solution: B
tan(x  y)  tan(x  y)  1
Differentiating w.r.t. x of y, we get
 dy   dy 
 sec 2 (x  y)  1    sec 2 (x  y) 1  0
 dx   dx 
dy sec 2 (x  y)  sec 2 (x  y)
  .
dx sec 2 (x  y)  sec 2 (x  y)

dy
Problem 20. If y sec x  tan x  x2 y  0 , then =
dx
2xy  sec 2 x  y sec x tan x 2xy  sec 2 x  sec x tan x
(A) (B) 
x 2  sec x x 2  sec x
2xy  sec 2 x  y sec x tan x
(C)  (D) None of these
x 2  sec x

Solution: C

P-2022-CBSE-P1-MATHEMATICS-BASIC CALCULUS
29
2
y sec x  tan x  x y  0
dy dy
 sec x  y sec x tan x  sec 2 x  2xy  x 2 0
dx dx
dy 2xy  sec 2 x  y sec x tan x
  .
dx x2  sec x

x dy
Problem 21. If sin(xy)   x2  y, then 
y dx
y[2xy  y 2 cos(xy)  1] [2xy  y 2 cos(xy)  1]
(A) (B)
xy 2 cos(xy)  y 2  x xy 2 cos(xy)  y 2  x
y[2xy  y 2 cos(xy)  1]
(C)  (D) None of these
xy 2 cos(xy)  y 2  x

Solution: A
x
sin(xy)   x2  y
y
Differentiating both sides,
d  1  dy 1 dy
cos(xy) (xy)  x   2    2x 
dx  y  dx y dx
x dy 1
 [x cos(xy)  2  1]  2x   y cos(xy)
y dx y
dy  2xy 2  y  y3 cos(xy) 
  .
dx  xy 2 cos(xy)  x  y 2 

dy
Problem 22. If sin2 x  2cos y  xy  0 , then 
dx
y  2sin x y  sin 2x
(A) (B)
2 sin y  x 2 sin y  x
y  2sin x
(C) (D) None of these
sin y  x

Solution: B
sin2 x  2cos y  xy  0
dy dy
 2sin x cos x  2sin y yx 0
dx dx
dy y  sin2x
  .
dx 2sin y  x

P-2022-CBSE-P1-MATHEMATICS- BASIC CALCULUS


30
CHAPTER PRACTICE PROBLEM

Subjective:

Level – I

 1  x2  dy
1. If y = tan1  2  , then find at x = 1.
 1 x  dx

2. Find the derivative of the following functions w.r.t. x.


y = sin[(lnx)3]
3. Find the derivative of the following functions w.r.t. x.
y = cos-1 (x2)

Level – II

4. Differentiate with respect to x


sin x
(i) y = (ii) y = x log sin(x 2  1)
x 1

2t 2t dy
5. If tany = 2
and sinx = 2
, then find .
1 t 1 t dx

Objective:

MCQ Single Correct

1. The differential equation whose solution is y = c1sinx + c2cosx, where c1, c2 one arbitrary
is
d2 y dy
(A) 2
y0 (B)  y2  0
dx dx
d2 y
(C) y0 (D) none of these
dx2

2. lim

sin  cos2 x  equals to
2
x 0 x
(A)  (B) 

(C) (D) 1
2

2  ab x
3. If y  tan1  tan  , where a  b  0 then
 ab 2 
a2  b2 
dy 1 d2 y b sin x
(A)  (B) 2

dx a  bcos x dx (a  b cos x)2
dy 1 d2 y b sin x
(C)  (D)  .
dx a  bcos x dx2 (a  bcos x)2

P-2022-CBSE-P1-MATHEMATICS-BASIC CALCULUS
31

4. The derivative of f(tanx) w.r.t. g(secx) at x  , where f(1) = 2, g( 2)  4 , is
4
1
(A) (B) 2
2
(C) 1 (D) none of these.

1  cos2  x  1
5. The value of lim
x 1  x  1
(A) exists and it equals 2
(B) exists and it equals  2
(C) does not exist because (x – 1)  0
(D) does not exist because left hand limit is not equal to right hand limit.

nk sin2  n! 
6. The value of lim ,0  k  1 , is equal to
n  n2
(A)  (B) 1
(C) 0 (D) None of these

x tan2x  2x tan x
7. lim 2
is
x 0
1  cos2x 
(A) 2 (B) 2
1 1
(C) (D) 
2 2

MCQ Multi Correct

1. Let h(x)  min{x,x 2 }, for every real number of x. Then


(A) h is continuous for all x (B) h is differentiable for all x
(C) h'(x)  1 , for all b  1 (D) h is not differentiable at two values
of x

Numerical Based

 1
 x sin , x0
1. If f(x)   x , then lim f(x) 
x 0
 0, x0

x 3 cot x
2. If lim  k then k =
x 0 1  cos x

x(e x  1)
3. If lim   then  =
x 0 1  cos x

x
4. If f(x)  for x  R, then f '(0) 
1 | x |

P-2022-CBSE-P1-MATHEMATICS- BASIC CALCULUS


32
ASSIGNMENT PROBLEMS

Subjective:

Level – 0

1  3x  1  3x
1. Evaluate : lim
x 0 x

 x2 
2. Evaluate : lim  
x 2  3x  2  x  2 

 1  x  x2  1 
3. Evaluate : lim  
x 0  x 
 

 tan3x  2x 
4. Evaluate : lim  
x 0  3x  sin2 x 

 sin3x  7x 
5. Evaluate : lim  
x 0  4x  sin 2x 

 2 sin x  sin2x 
6. Evaluate : lim  
x 0  x3 

x  sin x
7. Evaluate : lim
x  x  cos2 x

 tan x  x 
8. Evaluate : lim  2 
x 0  x tan x 

9. 
Differentiate w.r.t. x, y = e x x 3  x 
10. Differentiate w.r.t. x, y = (tanx + secx) (cotx + cosecx)

x2 sin x
11. Differentiate w.r.t. x, y =
1  sin x

x tan x
12. Differentiate w.r.t. x, y =
sec x  tan x

sin x  x cos x
13. Differentiate w.r.t. x, y =
x sin x  cos x

14. Solve the differential equation tan x. tan y dy + sec2 x sin2 y dx = 0

P-2022-CBSE-P1-MATHEMATICS-BASIC CALCULUS
33
15. Find the interval of increase and decrease of the function
f (x) = 6x4  8x3  12x2 + 24x  5 = 0.

1 1 dy
16. If x = t + , y = t  , find in term of x and y.
t t dx

sin x
17. Differentiate with respect to x, y =
x 1

18. Differentiate with respect to x


y = x log sin ( x 2  1)

dy
19. Find if x = a cos2 t, y = a sin2 t
dx

dy
20. If x = a( - sin), y = a(1  cos), then find .
dx

dy
21. If y = xsinx, find .
dx

d2 y
22. If x = a(t + sint) and y = a(1 – cost), find .
dx 2

x2  5x  6
23. Evaluate : lim
x 1 x2  x  1

24. Evaluate : lim


x 2

 2x  1
x 1  x  1

x3  8
25. Evaluate : lim
x 2 x  2

P-2022-CBSE-P1-MATHEMATICS- BASIC CALCULUS


34
Level – I

1. Evaluate the following limits:


x15  1 1  x n  1
(i) lim (ii) lim
x 1 x10 1 x 0 x

dy
2. Find , if
dx
2  3cos x 2 x
(i) y  (ii) y = x e sinx
sin x

x
e sin x
3. Evaluate :  dx
2 x

dy
4. Find
dx
if y = tan  cos e
  1 x 2
 
x4
5. If  eln f x  dx  + c, then find f (x).
4

1 1 dy
6. If x2 + y2 = t + and x4 + y4 = t2 + 2 , then find .
t t dx

dy 1  y2
7. If 1  x2  1  y 2  a  x  y  , then prove that  .
dx 1  x2

2x  9
8. Evaluate  x 2  9x  30 dx .

log  x  1  log x
9. Evaluate  dx .
x  x  1

10. f : R  R defined by f(x) = x2  3x + 2  x  R. Find the interval in which f(x) increases or


decreases.

/4
2
11. Evaluate  sin
0
x dx .

P-2022-CBSE-P1-MATHEMATICS-BASIC CALCULUS
35
Level – II

dy
1. The slope at any point of a curve y = f(x) is given by = 3x2 + x and it passes through
dx
(1, 0).Then find the equation of curve.
dy
2. Find , when x and y are given by (i) x = acos3t, y = b sin3t,
dx
(ii) x = (v0 cos) t, y = (v0 sin) t – gt2/2, where v 0,  and g are constants.

3. Find the integrals of the following functions:


cos x ax n1 1
(i) (ii) (iii)
1  sin x 2 bx n  c   xx
π π
4 4
cos x  sin x 2  3 sin x
4. Evaluate: (i) 
0
cos x  sin x
dx (ii) 
0
cos2 x
`dx

x 2  5x  4
5. (i) (3x2  x  4) (x  1) (2x  3) > 0 (ii) 0
 3  x 3  x  2  4
2

(iii)
x

x 1
(iv)
x 2
x  0
x 1 x 4 5
 x  2   x  1
sec 2 (loge x) x 1  cos x
6. Evaluate: (i)  dx (ii) e tan e x sec e x dx (iii)  3
dx
x x  sin x
7. Integrate the following functions:
x  x  1 x  log x 2
(i) sin x cos x (ii) (iii)
(x 2  1)1/ 3 2x
1 1
(iv) log x (v)
x x  x 1
8. Find the value of the following
/2 /2
1 2
(i)

0
1  cos x
dx (ii)
 1  cos x 
0
sin x dx

9. Find the value of the following


/4 /2 /2
1  sin 2x 3 2
(i)  dx (ii)  cos  d (iii)  sin x cos 2 x dx
cos x  sin x
0 0 0
10. Evaluate the following
2x 3  2x 2  1  x  110   x  2 10   x  3 10
(i) lim (ii) lim 10
x  3x 3  x  2 x 
x10   x  3 

dy
11. Find (i) y = xx (ii) xy = y x
dx
a
ax
12. Evaluate  dx , a > 0.
ax
0

P-2022-CBSE-P1-MATHEMATICS- BASIC CALCULUS


36

Objective:

Level – I

/2
cos x
1.

0
1  sin x
dx is equal to

(A) 2  2  1 (B) 
2  1
(C) 2  2  1 (D) none of these

x  sin x  1
2. lim is equal to
x  x  cos x
(A) 1 (B) 2
(C) 3 (D) 4

2
2  ax 
3
3. The value of the integral  bx  c dx depends on the
(A) value of b (B) value of c
(C) value of a (D) value of a and b

3
2
4. x sin x3 e x dx equals to

x3
  3
1  x3 sin x  cos x
3
 
(A) e  sin x 3
 cos x 3
 (B)
3 
e
2 
3
(C) e x sin x3 (D) None of these

x x 
a  a d2 y
5. If y =  e  e a  and = y, then a equals
2 


 dx 2
(A) 2 (B) 1
1 1
(C) (D)
2 2

x dy
6. If y = 25 , than equals to
dx
x x
(A) 25 ln8 (B) 25  5 x ln8
x
(C) 25  5 x ln5ln2 (D) None

2 dy
7. If y = sin (cox3x), then equals
dx
(A) 3sin(3x) sin(2cos3x) (B) -3sin3x sin(2cos3x)
(C) -3sincos(6x) (D) 3sin(cos6x)

dy
8. If y = f(e-x) and f(x) = lnx then at x = ln2 is
dx
 ln2
(A) 2ln2 (B)
2
(C) ln2 (D) none of these

P-2022-CBSE-P1-MATHEMATICS-BASIC CALCULUS
37
dy
9. If y = (x  a) (x  b), then the value of x for which  0 is
dx
a b
(A) (B)
2 2
ab ab
(C) (D)
2 2

3x 3
10. a dx is equal to

a3x 3 a3x 3
(A) c (B) c
log a 3 log a
(C) a3x 3 log a  c (D) 3a3x 3 log a  c

/2
4 cos x
11. The value of  dx is equal to
 0
cos sin x
(A) 1 (B) 2
(C)  1 (D) 0

/ 3
1
12. If  dx = k, then k is equal to
/4
sin x cos2 x
2

2
(A) (B) 3
3
(C) 2 (D) 5

 dy 
13. The Solution of the differential equation loge    3x  5y is
 dx 
5y 3x
(A) 5e = 3e + c (B) 3e5y – 5e3x = c
5y -3x
(C) 5e = - 3e + c (D) 3e5y – 5e-3x = c

14. The equation of a curve whose slope at any point is thrice its abscissa and which passes
through
(-1, -3) is
(A) y = x2 – 4 (B) 3y = 2(x2 – 4)
2
(C) 2y = 3(x – 3) (D) None of these

15. Number of positive integral values satisfying x3  48  4x (3x  11) are


(A) 2 (B) 3
(C) 4 (D) 5

ex 1  x 
16.  cos (xe ) dx is equal to
2 x

(A)  cot  xe  + c x
(B) tan xe x  +c
(C) tan  e  + c x
(D) none of these

P-2022-CBSE-P1-MATHEMATICS- BASIC CALCULUS


38
log(sin x)
17. e dx is equal to
(A) sin x + c (B) -cos x + c
log cos x
(C) e +c (D) None of these


18. 0 4 x tan x2 dx is equal to
1
(A) log2 (B) log2
2
1
(C) log2 (D) None of these
4

19. A differentiable function f is defined for all x > 0 and satisfies f(x) = x4 for all x > 0, then f
(16) is equal to
(A) 64 (B) 16
(C) 8 (D) None

1  ln x
20.  4  x ln x 2 dx equals

1 1
(A) ln 4  xln x2  c (B) ln 4  xln x 2  c
2 4
1 1
(C) ln 2  x ln x2  c (D)  ln 2  x ln x2  c
2 2


21. Let f(x) = ln|x  1|, x  1, then the value of f  
 2
(A) is 2 (B) is 2
(C) is 0 (D) does not exist

dx
22. If   x  1  x  2  A ln x  1  Bln x  2  c , then
(A) A + B = 0 (B) AB =  1
A
(C)  2 (D) A  B = 0
B

Level – II

Multi-choice (One or More correct answers)

x 1
1.  2x3 / 2 dx equals to

x2  1
(A) x1/ 2  x 1/ 2  c (B) c
x3 / 2  x1/ 2
2

(C)
x2  2x  1
c (D)
x 1/ 2
1  3
c
x3 / 2  x1/ 2 x1/ 2

P-2022-CBSE-P1-MATHEMATICS-BASIC CALCULUS
39
 
1 x 1
2.   2  x  1  2 x2  1  2 x2  1  dx = Alog (x 1) + Blog (x2 + 1) + C tan-1x + K then,
 e e

     
1 1
(A) A  (B) B  
2 4
1 1
(C) C   (D) C 
2 2
log  log, x 
dy
If y  x 
log x 
3. , then is
dx
y y
(A)  
ln x x 1  2ln x ln  ln x   (B) log x loglog x   2log log x   1
x x
y  2 y log y
(C) ln x   2ln ln x  (D) 2log  log x   1 .
x ln x  x log x 

cos ec 2 x sin x
4.  dx  2 dx equals
2
cot x  1 1  2 sin2 x
cos2x
(A)  cos ec 2 x  2 dx (B)  dx
2 sin x
cos2x
(C)  dx (D)  cos ec 2 x  1dx
sin x

5. Solution of the equation (1 + x2)dy = (1 + y2)dx is


(A) y – x = c(1 + xy) (B) y – x = c (1 – xy)
(C) tan-1y = tan-1x + c (D) tan-1x + cot-1y = c
4
xdx  a tan3 x  b tan x    x  , then
6. If  tan
1
(A) a = (B) b = 1
2
(C) (x) = x + c (D) b =  1

2t 2t dy
7. If tany = 2
and sinx = 2
, then is equal to
1 t 1 t dx
(A) 1 (B) sin2 + cos2
2 2
(C) sec   tan  (D) none of these

dy
8. If y = (sin x)x , then is
dx
(A) (sin x)x (x cot x + ln sin x) (B) y(x cot x + ln sin x)
x1
(C) (sin x) (x cos x + sinx ln sin x) (D) none of these

x x dy
9. If y  e  e , then is equal to
dx
x x
e  e e x
 e x
(A) (B)
2 x 2x
1 1
(C) y2  4 (D) y2  4
2 x 2 x

P-2022-CBSE-P1-MATHEMATICS- BASIC CALCULUS


40
dy
10. Let y  x  x  x  ... , then is equal to
dx
1 x
(A) (B)
2y  1 x  2y
1 y
(C) (D)
1  4x 2x  y

11. If 1 is a twice repeated root of the equation ax3 + bx2 + bx + d = 0, then


(A) a = b = d (B) a + b = 0
(C) b + d = 0 (D) a = d

12. If x3 – 2x2y2 + 5x + y – 5 = 0 and y(1) = 1, then


(A) y  1  4 / 3 (B) y   4 / 3
22
(C) y 1  8 (D) y  1  2 / 3
27

13. f(x) = |x2 – 3|x| + 2|. Then which of the following is/are true?
(A) f(x) = 2x – 3 for x  (0, 1)  (2, )
(B) f(x) = 2x + 3 for x  (–, –2)  (–1, 0)
(C) f(x) = –2x – 3 for x  (–2, –1)
(D) None of these.

x 4  x2  1 dy
14. If y  2
and  ax  b , then the value of a – b is
x  3x  1 dx
 
(A) cot (B) cot
8 12
5 5
(C) tan (D) tan .
12 8

x  2 x 1
15. Let f  x   x . Then
x 1 1
(A) f(10) = 1 (B) f(3/2) = – 1
(C) domain of f(x) is x  1 (D) range of f(x) is (–2, –1]  (2, )

Assertion Reason
Questions (9 – 10) contain STATEMENT-1 (Assertion) and STATEMENT-2 (Reason).
Option (A) if both the statements are TRUE and STATEMENT-2 is the correct explanation of
STATEMENT-1
Option (B) if both the statements are TRUE but STATEMENT-2 is NOT the correct
explanation of STATEMENT- 1
Option (C) if STATEMENT-1 is TRUE and STATEMENT-2 is FALSE.
Option (D) if STATEMENT-1 is FALSE and STATEMENT-2 is TRUE.

16. STATEMENT -1 : The function f(x) = x + sinx is non decreasing  x  R.


because
STATEMENT -2 : f(x)  0  x  R.

P-2022-CBSE-P1-MATHEMATICS-BASIC CALCULUS
41
17. Let f(x) = sinx  x  [0, 2]
STATEMENT -1 : f(x) is invertible in the given domain.
because
STATEMENT -2 : f(x) is many one-onto.

COMPREHENSIONS
(I) Read the following write up carefully and answer the following questions:

x
Let f  x  
1 x

18. Domain of f(x) is


(A) R  {1} (B) R  {0}
(C) R (D) none of these
19. Interval of increase of f(x) is
(A) ( , 0) (B) (0, )
(C) (1, 1) (D) R
20. Range of f(x) is
(A) (1, 1) (B) [1, 1]
(C) [1, 0)  (0, 1] (D) R
(II) Read the following write up carefully and answer the following questions:

d3 y
Consider the differential equation 0
dx3

21. The degree of the above differential equation is


(A) 0 (B) 3
(C) 1 (D) not defined
22. The solution of the above equation is
(A) y = ax2 + bx + c (B) y2 = ax2 + bx + c
(C) x = ay2 + by + c (D) none of these

Matrix Match
23. Match the following:
Column  I Column  II
1 (p) tanx – cotx + c
(A)  a2 cos2 x  b2 sin2 x dx
3 1
(B)  sin xdx b 
(q) tan1  tan x   c
ab a 

(C)
dx sec 2 x
 sin x cos3 x (r) log tanx +
2
c
2
(D)  sec x cos ec 2 x dx 1 a 
(s) cot 1  cot x   c
ab b 
cos3 x
(t) – cosx + c
3

P-2022-CBSE-P1-MATHEMATICS- BASIC CALCULUS


42
24. Match the following
List–I List–II
(P) Let y = f(x) be given by x = t5 – 5t3 – 20t + 7 and y = 4t3 – (1) 0
2 dy
3t – 18t + 3. Then – 5  at t = 1 is
dx
(Q) Let P(x) be a polynomial of degree 4, with P(2) – 1, P(2) = (2) –2
0, P(2) = 2, P(2) = – 12 and Piv(2) = 24. Then P(3) is
dy (3) 2
4
1 1 y
(R) y . Then is
x dx
1  x4
 2x  3y  2f  x   3f  y  (4) –1
(S) f  and f(0) = p and f(0) = q. Then
 5  5
f(0) is
Codes :
P Q R S
(A) 2 3 1 4
(B) 4 3 2 1
(C) 2 3 4 1
(D) 4 3 1 2
25. Matching List Type(3 Column & 4 Rows)
 Column – 1 represents functions
 Column – 2 represents interval in which function is increasing or decreasing
 Column – 3 represents min or max value of function
Column – 1 Column – 2 Column – 3
(I) f(x) = x2 + 1 (i)  3  (P) max value of function is 2
increases in  , 
 2
(II) g(x) = |sin x| (ii) increases in [0,1] (Q) min value of function is 1
(III) h(x) = cos2x (iii)    (R) min value of function is 0
decreases in   , 
 2 2
(IV) l(x) = tanx (iv)   (S) l(x) = 100 has a solution
increases in  , 
2 

(I). Which of the following is correct combination


(A) (I) (ii) (Q) (B) (II) (i) (R)
(C) (III) (iv) (P) (D) (IV) (iii) (S)

(II). Which of the following is correct combination


(A) (III) (iv) (R) (B) (I) (i) (Q)
(C) (IV) (iv) (S) (D) (IV) (iii) (Q)

(III). Which of the following is correct combination


(A) (I) (ii) (Q) (B) (III) (iii) (P)
(C) (II) (ii) (R) (D) (IV) (ii) (P)

P-2022-CBSE-P1-MATHEMATICS-BASIC CALCULUS
43
Numerical Based
 /2
26. 
 /2
sin x dx is equal to ______

2
27.  2 x dx
1
is equal to ______

2 3
28. If the minimum value of x + x + 1 is , then k is equal to _______
k
3 2
29. The maximum value of the function f(x) = 2x  15x + 36x  48 on the set
A = {x|x2 + 20  9x} is ________

P-2022-CBSE-P1-MATHEMATICS- BASIC CALCULUS


44
ANSWERS TO CHAPTER PRACTICE PROBLEM

Subjective:

Level – I

1. –1

3(ln x)2
2. cos (ln x)3 
x
2x
3.
1 x4

Level – II

x 2 cos(x 2  1)
4.
sin(x 2  1)

 log sin x 2  1 

5. 1

Objective:

MCQ Single Correct

1. A 2. B 3. A 4. A

5. C 6. C 7. C

MCQ Multi Correct

1. A, C

Numerical Based
1. 0 2. 2 3. 2 4. 1

P-2022-CBSE-P1-MATHEMATICS-BASIC CALCULUS
45
ANSWERS TO ASSIGNMENT PROBLEMS

Subjective:

Level – 0
1
1. 3 2. 8 3.
2
1 5
4. 5. 6. 1
3 3
1  1 
7. 1 8. 9. e x  x 3  3x 2   x
3  2 x 
10. (secx  cosex) (secx + tanx) (cosecx + cotx)
x2 1  x  cos x  x sin x  2  x  x sec x  sec x  tan x   tan x
11. 12.
1  x 
2
 sec x  tan x 
x2
13. 2
14. tan x tan y = ec = k constant.
 x sin x  cos x 
15. For increasing  1 < x < 1, x > 1, for decreasing x <  1.
2
xy
1
dy  2  2 (x  1)cos x  sin x
16.  17.
dx  x  y  2 2(x  1)3/2
  1
 2 
x 2 cos( x 2  1)
18. 2
sin( x  1)
 log sin x 2  1   19. 1

 sin x 
20. cot /2 21. x sin x   cos x log x 
 x 
1 t 2
22. sec 4   23.
4a 2 3

24. 0 25. 12

Level – I

3
1. (i) (ii) n
2
2. (i) cosecx(3cosecx  2cotx) (ii) xex(xcosx + xsinx + 2sinx)

et
3. I= (sin t  cos t) + c.
2
dy x
4.
dx 
  sec 2 cos e 1 x2
 sin e  e
1 x 2 1 x 2

1  x2
.

3 y
5. f (x) = x . 6. 
x

P-2022-CBSE-P1-MATHEMATICS- BASIC CALCULUS


46
2
2 1   x  1 
8. log x  9x  30  c 9.  log    c
2   x 
3 3  
10. increasing  x > decreasing  x < 11. 
2 2 8 4

Level – II

x2 1
1. y = x3  
2 2
b gt
2. (i)  tan t (ii) tan   sec 
a v0
1 a
3. (i)  c (ii) loge bx n  c  K
1  sin x bn
(iii) 2loge(1+x) + c
1
4. (i) ln 2 (ii) 3 2 1
2

5. (i) x  ( ,  1)  (1, 4/3)  (3/2, ) (ii) x  (1, 3)  (4, )


 1
(iii) x   0,   1,   (iv) x  (1, 2)  (2, )  { 1, 0}
 2

6. (i) tan (loge x) + c (ii) sec (ex) + c


3
(iii)  x  sin x 2 / 3  c
2

2 3/2 3 2 2/3
7. (i)
3
 sin x   c (ii)
4

x 1 c 
1 3 log x 2
(iii)  x  log x   c (iv) c
6 2
2 3/2 2
(v)  x  1  x3 / 2  c
3 3

8. (i) 1 (ii) 7/3


9. (i) 1 (ii) 2/3 (iii)
16

10. (i) 2/3 (ii) 3/2

dy dy y  x loge y  y 
11. (i)  x x 1  loge x  (ii)   
dx dx x  y loge x  x 

 
12. a   1
2 

P-2022-CBSE-P1-MATHEMATICS-BASIC CALCULUS
47

Objective:

Level – I

1. A 2. A 3. B 4. B
5. B 6. C 7. B 8. C
9. D 10. B 11. A 12. A
13. B 14. C 15. D 16. B
17. B 18. C 19. D 20. A
21. A 22. A

Level – II

1. A, B, C 2. A, B, C 3. B, D 4. A, C
5. A, C, D 6. C, D 7. A, B, C 8. A, B, C
9. A, C 10. A, C, D 11. B, C, D 12. A, C
13. A, B, C 14. B, C 15. A, B, D 16. D
17. D 18. C 19. D 20. A
21. C 22. A
23. (A)  (q), (s) (B)  (t) (C)  (r) (D)  (p)
24. C
25. (I). B (II). C (III). A
26. 2 27. 5 28. 4 29. 7

P-2022-CBSE-P1-MATHEMATICS- BASIC CALCULUS


PINNACLE-CBSE

LINEAR INEQUALITIES
CONTENTS SYLLABUS

 Introduction 1
CBSE: Linear inequalities. Algebraic solutions of

 Solution of linear 1 linear inequalities in one variable and their


inequations of one variable
representation on the number line. Graphical
 Exercise 1 3
solution of linear inequalities in two variables.
 Linear inequations in two 3
variables

 Exercise 2 6

 Answers to Exercises 7

 Solved Problems 8

 Assignment Problems 11

 Answers to Assignments 18
Introduction
A relation of the type f(x, y) > 0, f(x, y) < 0, f(x, y)  0, or f(x, y)  0 is called an inequality or an
inequation. It is also called a constraint or a condition. If f(x, y) is linear in x and y, then the
inequality is called a linear inequality or a linear inequation. For example
3x + 2y – 4 > 0, ax + by  c
are linear inequations in x and y. The solution set of a linear inequation is the set of all values of
(x, y) which satisfy the given inequation.
The above examples involve two variables, namely x and y and the inequations are called linear
inequations in two variables x and y.
The relations ax + b  0 or cx + d  0 are linear inequations in one variable x.

Solutions of linear inequations of one variable.


Let us suppose that a person has with him Rs. 200/- and he wishes to buy sugar which is
available in 1 kg packets at a price of Rs. 19/- per packet. If x denotes the number of packets of
sugar he buys, then the total amount spent is Rs, 19x. Since sugar can be bought in packets
only, the entire amount of Rs. 200/- cannot be spent. Hence
19x < 200 …(i)
and this statement is not an equation.
Clearly, x cannot be negative or a fraction. Now consider the inequation (i):
For x = 0, L.H.S. = 19x = 0 < 200 = R.H.S. and this is true.
For x = 1, L.H.S. = 19x = 19 < 200 = R.H.S which is true.
For x = 2, L.H.S. = 19x = 38 < 200 = R.H.S. which is again true.
…….. ……. ……. ……. ……. ……. ……. ……
For x = 9, L.H.S. = 19x = 19 x 9 = 171 < 200 = R.H.S. This is true.
For x = 10, L.H.S. = 19x = 19 x 10 = 190 < 200 = R.H.S.,which is ture.
For x = 11, L.H.S. = 19x = 209 < 200, which is not true.
For all x > 11, the statement 19x < 200 is not true.
We, thus, find that the only values of x which make the inequation a correct statement are
x = 0, 1, 2, …, 10. These values of x are called the solutions of the inequation (i).

Note:
(i) Equal numbers may be added to (or subtracted from) both sides of an inequality without
affecting the sign of the inequality.

(ii) Both sides of an inequality can be multiplied (or divided) by the same positive number. But
when both sides are multiplied or divided by a negative number, then the sign of the inequality is
reversed.
Let us reconsider the inequation (i).
We have 19x < 200
19x 200
or 
19 19
200
or x  .
19
(i) When x is a natural number.
In this case the solutions are

P-2022-CBSE-P1-MATHEMATICS-LINEAR INEQUALITIES
2
1, 2, 3, ….10.
These solutions can be represented on the number line by ten points as shown

-3 -2 -1 0 1 2 3 4 5 6 7 8 9 10

(ii) When x is an integer.


In this case, the solutions are
…., –3, –2, –1, 0, 1, 2, 3, 4, …., 10.

Illustration 1. Solve 3x + 5 < x – 7 when


(i) x is an integer,
(ii) x is a real number.

Solution: We have 3x + 5 < x – 7 or, 3x + 5 – 5 < x – 7 – 5


2x 12
or, 3x – x < x – x – 12 or, 
2 2
or x < –6.

(i) For integral solutions


x = …., –9, –8, –7.

–9 –8 –7 –6 –5 –4 –3 –2 –1 0 1
(ii) For real values, the solutions are all real numbers x which are less than –6.
A circle over –6 implies that –6 is not included in the solution set.

–8 –7 –6 –5 –4

2x  3 4x
Illustration 2. Solve  10  4  .
4 3

2x  3 4x  2x  3   4x 
Solution: Here  10  4  or, 12   10   12  4 
4 3  4   3 
or, 6x  9  120  48  16x or, 6x  111  48  16x
10x 63 63
or, 10x  63 or,  or, x 
10 10 10

4 5 6 7
i.e. all real numbers which are less than or equal to 6.3 are solutions of the given
inequality.

Solutions of system of linear inequations in one variable


In order to solve a system of linear inequations, we first find the solution set of each of the
inequations separately. Then, we find the values of the variable which are common to them or the
intersection of all these sets.

P-2022-CBSE-P1-MATHEMATICS-LINEAR INEQUALITIES
3
Illustration 3. Solve for x, the following system of inequalities:
2x – 7 > 5 – x, 11 – 5x  1.

Solution: Here, 2x – 7 > 5 – x  3x > 12 or x > 4,


and 11 – 5x  1  10  5x or x  2.
From the above, we find that the value of x which satisfies both the inequalities
is x > 4.
x>4

–3 –2 –1 –0 1 2 3 4 5 6
x2

Illustration 4. Solve for x, the inequations 4 x  3  2x  17, 3 x  5  2 .

Solution: Here 4x + 3  2x + 17  2x  14 or x  7.
Also, 3x – 5  –2  3x < 3 or x < 1.
x<1

–2 –1 0 1 2 3 4 5 6 7

Here x is less than one and at the same time greater than or equal to seven,
which is not possible. Hence the given system has no solution.

Exercise 1:
Solve the inequations:
i) 4x + 3 > 3x; –(x + 3) + 4  –2x + 5
4x 9 3 7x 1 7x  2
ii)   x ;   x
3 4 4 3 6
3x
iii) 5 x  7  3( x  3 ) ; 1   x4
2
2x  3 4x
iv) 2( 2 x  3 )  10  6( x  2 ); 6  2  .
4 3

Linear inequations in two variables


A person has Rs. 200/- and wants to buy some pens and pencils. The cost of a pen is Rs. 16/-
and that of a pencil is Rs. 6/-. If x denotes the number of pens and y, the number of pencils, then
the total amount spent is Rs. (16 x + 6 y), and we must have
16 x + 6 y  200. …(i)
In this example x and y are whole numbers and can not be fractions or negative numbers. In this
case we find the pair of values of x and y which make the statement (i) true. The set of such pairs
is the solution set of (i).
To start with, let x = 0 so that
100 1
6y  200 or y   33 .
3 3

P-2022-CBSE-P1-MATHEMATICS-LINEAR INEQUALITIES
4
As such the values of y corresponding to x = 0, can be 0, 1, 2, 3, …., 33. Hence the
solutions of (i) are (0, 0), (0, 1), (0, 2) ,….., (0, 33).
Similarly, the solutions corresponding to x = 1, 2, 3, …, 12 are
(1, 0), (1, 1), …. , (1, 30),
(2, 0), (2, 1), …., (2, 28),
…………………………..
(11, 0), (11, 1), …, (11, 4),
(12, 0), (12, 1).
We note that the values of x and y cannot be more than 12 and 33 respectively. We also note that
some of the pairs namely (5, 20), (8, 12), and (11, 4) satisfy the equation
16x + 6y = 200 which is a part of the given inequation.

Let us now extend the domain of x and y from whole Y


number to the real numbers. Consider the equation 16
x + 6 y = 200
and then draw the straight line represented by it. This 16x + 6y = 200
line divides the coordinate plane in two half planes.
For (0, 0), (i) yields, 0  200 which is true and hence (0, 0)
we conclude that (0, 0) belongs to the half plane X
represented by (i). Hence the solution of (i) consists of
all the points belonging to the shaded half plane which
consists of infinite number of points.

Note:
In order to identify the half plane represented by an inequation, it is sufficient to take any known
point (not lying on the line) and check whether it satisfies the given inequation or not. If it satisfies,
then the inequation represent that half plane, containing the known point, otherwise the
inequation represents the other half plane.

Illustration 5: Draw the graph of the solution region satisfied by the in-equation 3x  5y + 6 > 0.

Solution: The boundary line is given by the


equality 3x  5y + 6 = 0 . It is a
straight line which meets the x-axis
at (2, 0) and the y-axis at (0, 1.2).
Substituting (0, 0) in the given (0, 1.2)
expression, we have
00+6>0
which is true. Hence, the origin is a (2, 0)
point in the solution region. The
solution region is, thus, on the
same side of the line as the origin.
Since equality is not included in the
given inequation, the points on the
boundary of the line are not part of
the solution. The solution set is
shown in the figure.

P-2022-CBSE-P1-MATHEMATICS-LINEAR INEQUALITIES
5
For a set of linear inequations, in two variables, the solution region may be
(i) a closed region inside a polygon (bounded by straight lines),
(ii) an unbounded region (bounded partly by straight lines),
or (iii) empty.

Illustration 6. Draw the graph of the solution region satisfied by the inequalities
2x + y  4, y  2, x  0.

Solution: The solution region is bounded by the straight lines


2x  y  4 , …(i)
y = 2, …(ii)
x = 0. …(iii)
The straight line (i) meets the coordinate axes at (2, 0) and (0, 4). Moreover, for
(0, 0), the inequation 2x + y  4 gives 0  4, which is true. Hence the half plane
represented by 2x + y  4, contains the origin (0, 0).
Also for (0, 0), y  2 gives 0  2 which is (0, 4)
true, so that y  2 also contains the origin. y=2
With x  0, the shaded region is the
required solution region. All the boundary
(0, 0) (2, 0)
lines are part of the solution region.
Moreover, the solution region is an x=0
bounded region, bounded by the three
boundary lines.

Illustration 7. Draw the graph of the solution region satisfied by the inequations
x + y  1, 2x + y  4, x  0, y  0.

Solution: The solution region is bounded by the straight (0, 4)

lines
x+y = 1, (1)
2x + y = 4, (2)
(0, 1)
x = 0, (3) (2, 0)
(0, 0) (1, 0)
y = 0. (4)
The straight line (1) meets the coordinates
axes at (1, 0) and (0, 1). For (0, 0),
x + y  1 gives 0  1 which is not true.
Hence (0, 0) does not lie in the half plane represented by x + y  1. The line (2)
meets the coordinate axes in (2, 0) and (0, 4) and 0  4.Hence (0, 0) lies in the
half plane 2x + y  4. Hence the region bounded by x + y  1 and 2x + y  4

P-2022-CBSE-P1-MATHEMATICS-LINEAR INEQUALITIES
6
belongs to either the fourth quadrant, or the first quadrant or the second
quadrant. But the point belonging to x  0, y  0, all lie in the third quadrant.

Hence no point satisfies all the four given inequations.


Hence the solution set is empty.

Illustration 8. Find the solution region satisfied by the inequalities 2x + y  4, 3x + 3y 1,

x – y  1, x  0, y  3.

Y
Solution: The solution region is bounded
(0, 4)
by the straight lines y=3
(0, 3)
2x + y = 4, … (1)
3x + 3y = 1, … (2)
x – y = 1, … (3)
x = 0, … (4) (0, 1/3)
(0, 0) (1, 0) (2, 0) X
y = 3. … (5) (1/3,0)
(0, –1)
The points where the first three lines meet
the x-axis are (2, 0), (1/3, 0), (1, 0)
respectively.
The points where the first three lines meet the y-axis are (0, 4), (0, 1/3), (0, 1)
respectively. Moreover, (0, 0) belongs to the half planes 2x + y  4 and
x – y  1, y  3 but not to the half plane 3x + 3y  1.
The solution region is the shaded region. All the boundary lines are part of the
solution region and the closed region inside a polygon.

Exercise 2.
Draw the graphs of the region satisfied by the solutions of
i) 2x + 3y  3,
ii) 2x + y > 0, x + 2y  2,
iii) x – y  1, x + y  1, y > 1,
iv) 2x + 3y  6, x + y  1, 2x – y – 4  0, y  0.

P-2022-CBSE-P1-MATHEMATICS-LINEAR INEQUALITIES
7
ANSWERS TO EXERCISES

Exercise 1:
i) 3<x4
ii) 4<x<9
iii) 2x<8
iv) x

Exercise 2:
i)

(0, 1)

(3/2,0)

ii)

x+2y =2

2x+y =0

(iii) Empty set or null set.

iv)

2x+3y =6

x+y =1
2xy =4

P-2022-CBSE-P1-MATHEMATICS-LINEAR INEQUALITIES
8
SOLVED PROBLEMS

Problem 1: Solve for x, the following system of inequations:


x + 2y  3, 3x + 4y  12, x 0, y  1.

Solution: The solution region is bounded by the


straight lines (0, 3)
x + 2y = 3, … (1)
3x + 4y = 12, … (2)
x = 0, … (3) (0, 3/2)
y = 1. … (4) y=1

The straight lines (1) and (2) meet the x=0 (4, 0)
(3, 0)
x-axis in (3, 0) and (4, 0) and for (0, 0),
x + 2y  3  0  3, which is true.

Hence (0, 0) lies in the half plane x + 2y  3. Also the lines (1) and (2) meet the
y-axis in
(0, 3/2) and (0, 3) and for (0, 0)
3x + 4y  12
 0  12 which is not true. Hence (0, 0) doesn’t belong to the half plane
3x + 4y  12. Also x  0, y  1  that the solution set belongs to the first
quadrant. Moreover all the boundary lines are part of the solution. From the
shaded region, we find that there is no solution of the given system. Hence the
solution set is an empty set.

Problem 2. Find all the pairs of consecutive odd natural numbers, both of which are larger
than 10, such that their sum is less than 40.

Solution: Let x be one of the odd natural numbers so that the other one is x + 2 (being
consecutive odd numbers). Here x + 2 > x and x > 10.
Also x + x + 2 < 40  2x < 38 or x < 19.
Hence 10 < x < 19, so that the required pairs are (11, 13), (13, 15), (15, 17),
(17, 19).

1
Problem 3. Solve 3x  2  .
2

1
Solution: Since 3x  2  , we have
2
1 1 1 1
  3x  2  or,   2  3x   2
2 2 2 2
1 3 1 5 1 5
or,   x   or,  x  .
3 2 3 2 2 6
Hence solution set consists of all real numbers lying between
1 5 1 5
and including and .
2 6 2 6

Problem 4. A man wants to cut three lengths from a single piece of wire of total length 91
cm. The second length is to be 3 cm longer than the shortest and the third length
is to be twice as long as the shortest. What are the possible lengths of the
shortest piece if the third length is to be at least 5 cm longer than the second.

P-2022-CBSE-P1-MATHEMATICS-LINEAR INEQUALITIES
9
Solution: Let the length of the shortest piece of wire be x so that the second and the third
lengths are x + 3 and 2x respectively. Now, it is given that third length is to be at
least 5 cm longer than the second  2x  (x  3)  5
or x  8 . (1)
Also the total length is 91 cm so that
x + (x + 3) + (2x)  91  4x  88
or, x  22. (2)
From (1) and (2), we find that 8  x  22.

x 8 x 5
Problem 5. Solve, for x, the inequations (i)  1, ( ii ) 0.
x 2 x2

x8 6
Solution: (i) The given inequation is rearranged as  1  0 or 0
x2 x2
 x + 2 > 0 or, x > –2.
x5
(ii) For  0 , we first consider that x + 2 > 0 or x >  2 so that x – 5 < 0 or
x2
x < 5. Combining the two results, we get that –2 < x < 5.
If, on the other hand, x + 2 < 0, then x – 5 > 0
 x < –2 and x > 5 which is not possible.
Hence the only solution is –2 < x < 5.

Problem 6. Draw the graph for the solution region satisfied by the inequalities
5
x  y  5, x  2y  1, x  y  4, x  0, y  0.
2

Solution: The solution region lies in the first quadrant x + 5y/2 = 5


x– y=4
and is bounded by the straight lines (0, 2)
5 x + 2y = 1
x  y  5, (1) (0, 1/2)
2 (1, 0) (4, 0) (5, 0)
x  2y  1, (2)
xy  4. (3)
The line (1) meets the coordinate axes at (5, 0)
5
and (0, 2). For (0, 0), x  y  5  0  5 which
2
is true.
The line (2) meets the coordinate axes at (1, 0) and (0, 1/2). For (0, 0), 0  1
which is not true. Also the line (3) meets the coordinate axes at (4, 0) and (0, 4).
For (0, 0), 0  4 which is true.
The solution set belongs to the shaded region, which is an enclosed region
bounded by the straight lines (1), (2), (3) and x = 0, y = 0.

Problem 7. Find the solution region satisfied by the inequalities


x + y  5, x  4, y  4, x  0, y  0, 5x + y  5, x + 6y  6.

P-2022-CBSE-P1-MATHEMATICS-LINEAR INEQUALITIES
10
Solution: We find that the solution set satisfies x  0, y  0, 5x + y = 5

x 4, y 4 so that the solution region lies within


the square enclosed by the lines x= 0, y= 0, x = 4, x+y=5
y = 4. Moreover, the solution region is bounded
by the lines
x + y = 5, … (1) x + 6y =6
5x + y = 5, … (2) (0,0) x=4
x + 6y = 6. … (3)

Line (1) meets the coordinate axes in (5, 0) and (0, 5) and the lines x = 4 and
y = 4 in (4, 1) and (1, 4), and 0 < 5 is true. Hence (0, 0) belongs to the half plane
x + y  5. But (0, 0) does not belong to the half planes 5x + y  5 and x + 6y  6.
The line 5x + y = 5, meets the coordinate axes in (1, 0) and (0, 5), and meets the
line x = 4 in (4,  15), where as it meet the line y = 4 in (1/5, 4).
Similarly x + 6y = 6 meets x = 4 in (4, 1/3) and y = 4 in (–18, 4).
The solution is marked as the shaded region.
Problem 8. Draw the graph of the solution region of the inequations x  0, y  0, x – y  1,
x – y  –1.
Solution: Here the region belongs to the first quadrant as Y

x  0, y  0.
Moreover, the region is bounded by the straight
lines x – y = –1 x–y=1
x – y = 1, (1)
x – y = –1. (2)
X
These are parallel lines.
Moreover, both 0  1 and 0  –1 are not true.
Hence the origin does not belong to the half planes
x – y  1 and x – y  –1
 Both the half planes are away from the origin and are disjointed.
Hence the solution set is empty.

Problem 9. Solve the system: |2x  3|  11, |x  2|  3.

Solution: |2x  3|  11  11  2x  3  11


or  8  2x  14   4  x  7 … (1)
Further, |x  2|  3  x  2  3 for x > 2
and 2  x  3 for x < 2
 x  5 or x   1. … (2)
From (1) and (2), we find that x  [ 4,  1]  [5, 7].

2  3x 1  x 3  4 x
Problem 10. Solve for x:   .
5 3 2

Solution: The given system may be written as


2  3x 1  x 1  x 3  4x
 and 
5 3 3 2
or 6  9x < 5  5x and 2  2x < 9 + 12x or 1 < 4x and  7 < 14x
1 1
or x > and x >  .
4 2
1 
Hence x   ,   .
4 

P-2022-CBSE-P1-MATHEMATICS-LINEAR INEQUALITIES
11
ASSIGNMENT PROBLEMS

1. Solve the system of inequations:


(i) 2x – 7 < 11, 3x + 4 < –5, (ii) 4x – 5 < 11, –3x – 4  8.

2. Represent the following inequations graphically in the two dimensional plane:


(i) x – 2y + 4  0, (ii) y + 8  2x, (iii) 2x  6 – 3y.
3. Represent the following systems of inequations graphically:
(i) x + y  9, y > x, x  1 (ii) 3x + y  60, x + 3y  30, x  0, y  0.
(iii) 2x + y  4, x + y  3, 2x – 3y  6. (iv) 3x + 2y  24, 3x + y  15, x  4.
(v) 3x + 2y  24, x + 2y  16, x + y  10, x  0, y  0.

4. Find all pairs of consecutive even positive integers, both of which are larger than 5 such
that their sum is less than 23.
5. In a drilling process, the temperature T (in degree Celsius) x km below the surface of the
earth, was found to be given by T = 30 + 25(x – 3), 3 < x < 15.
At what depth will the temperature be between 200C and 300C .

6. An aeroplane can carry a maximum of 360 passengers. The airlines reserves at least 20
seats for business class. But at least 8 times as many passengers prefer to travel by
economy class than by business class. Find the possible graphic distribution of
passengers in the two classes.

7. A company makes chairs and tables. The company has two different types of machines
on which the chairs and tables are manufactured. The time required in hours for
manufacturing each item and total available time in a week is given below:
Chairs Tables Total available time (in hours)
Machine 1 5 2 24
Machine 2 4 8 64
Show graphically the sets of chairs and tables that can possibly be manufactured in a
week.
8. Solve the systems:
2x  5  2x  5 
(i) 2x  3  5, 3  Hint : Write  3  0
x7  x7 
x2
(ii)  3, 2x  7  5 .
x2

x  3 2x  5 3x  5
9. Solve for x :   .
4 5 7
10. A plumber can be paid either (i) Rs. 600 and Rs. 50 per hour or (ii) Rs. 170 per hour. If
the job takes n hour, for what value of n the first mode i.e. (i) earns better wages for the
plumber.
11. A toy manufacturer produces dolls and machine guns under the constraints:
(a) he cannot produce more than 6 dolls per day and more than 5 machine guns per day.
(b) he must use minimum of 3 hours of labour per day.
(c) the raw material available is atmost 63 units per day.
If 2 hours of labour is required to produce a doll or a machine gun whereas 7 units of raw
material is required for a doll and 9 units for a machine gun, construct the problem.

12. A dealer deals in two items; radio and two-in-one. A radio costs Rs 500 and a two-in-one
costs Rs 2,500. Construct the problem under the constraints that he has Rs 50,000/ to
spend and space for 60 pieces to store.

P-2022-CBSE-P1-MATHEMATICS-LINEAR INEQUALITIES
12

Objective:

Level – I

1. The value of x for which 12x  6 < 0, 12  3x < 0


(A)  (B) R
(C) R  {0} (D) none of these

x3 x 1 x  2
2. The value of x for which x  , 2  x > 2x  8
4 2 3
 10   10 
(A)  1,  (B)  1,
 3  3 
(C) R (D) none of these

 27  x   27  9x 
3. The least integer satisfying 49.4     47.4    is
 10   10 
(A) 2 (B) 3
(C) 4 (D) none of these

1
4. If 0 < |3x + 1| < , then x belongs to
3
 4 2  4 2
(A)   ,   (B)   ,  
 9 9   9 9 
 4 2   1  4 2  1
(C)   ,      (D)   ,     
 9 9   3  9 9  3

2
5. If  3 , then x belongs to
x
2   2
(A)  ,   (B)  , 
3   3
2 
(C)  ,     , 0  (D) none of these
3 

6. (x + 1)2 < 25, then x belongs to


(A) (6, 4] (B) (6, 4)
(C) (4, 6) (D) (4, 6)

7. If log4x > 1, then x belongs to


(A) (0, ) (B) (4, )
(C) (4, ) (D) (, 4)

8. If |3  x| = x  3, then x is
(A) x < 3 (B) x > 3
(C) x  3 (D) x  3

x 1
9. The value of x for which 2
x
(A) (0, 1) (B) (–, –1)
(C) (–, 0) (D) [–1, 0)

P-2022-CBSE-P1-MATHEMATICS-LINEAR INEQUALITIES
13

x3 x 1 x  2
10. The value of x for which x  , 2 – x > 2x – 8
4 2 3
 10   10 
(A)  1, (B)  1, 
 3   3 
(C) R (D) none of these

Level – II

Multi-choice (Single Correct Answers)

1. The value of x, |x + 3| > |2x  1| is


 2   2 
(A)   , 4  (B)   ,  
 3   3 
(C) (0, 1) (D) none of these

2. If |x + 4| > 5, then x belongs to


(A) (, 1) (B) ( , 9)
(C) R  [9, 1] (D) none of these

3. If |x  1| + |x| + |x + 1|  6, then x lies in


(A) (, 2] (B) (, 2]  [2, )
(C) R (D) 

4. If |3x + 2| < 1, then x belongs to


 1  1 
(A)  1,   (B)   ,  1
 3  3 
 1
(C)  1,   (D) none of these
 3

5. If |3x  2|  1, then x belongs to


1  1 
(A)  , 1 (B)  , 1
3  3 
1   1
(C)  , 1 (D)   ,   1,  
3   3 

6. If |2x  3| < |x + 2|, then x belongs to


 1 1 
(A)   ,  (B)  , 5 
 3 3 
 1
(C) (5, ) (D)   ,   (5, )
 3

7. If |x  1|  |x  3|, then
(A) x  2 (B) x  2
(C) x  [1, 3] (D) none of these

8. If ||x|  1| < |1  x|, x  R, then x belongs to


(A) (1, 1) (B) (0, )
(C) (1, ) (D) none of these

P-2022-CBSE-P1-MATHEMATICS-LINEAR INEQUALITIES
14
9. If |x  1|  3 and |x  1|  1, then x belongs to
(A) [2, 4] (B) (, 2)  [4, )
(C) [2, 0]  [2, 4] (D) none of these

1
10. If x   2 , then x is
x
(A) R  {0} (B) R  {1, 0, 1}
(C) R  {1} (D) R  {1, 1}

1
11. If  2  4 , then x belongs to
x
 1 1 
(A)   ,   (B)  ,  
 2  6 
 1 1  1  1 
(C)   ,  (D)   ,     ,  
 2 6  2  6 

12. If 2  |x  3| < 4, then x belongs to


(A) (1, 1)  [5, 7) (B) [4, 2]
(C) (1, 7)  [5, ) (D) none of these

13. If log1/2x  log1/3x, then x belongs to


(A) (0, 1] (B) (0, 1)
(C) [0, 1) (D) none of these

x2  6
14.  1 , then x belongs to
5x
(A) (, 3) (B) (, 3)  (3, )
(C) R (D) (, 3]  [2, 0)  (0, 2]  [3, )

15. If x2 + x + |x| + 1 < 0, then x belongs to


(A) (0, ) (B) (, 0)
(C) R (D) 

16. The value of x satisfying |x + 3| > |2x – 1| is


 2   2 
(A)   , 4  (B)   ,  
 3   3 
(C) (0, 1) (D) none of these

Multi-choice (One or More Correct Answers)

17. If 0  x < 10  x, then solution set of x is a subset of


(A) N (B) R+
(C) R (D) W

18. The solution set of (x  1)2 < 0 is same as that of


(A) |x  1| < 0 (B) (x  1)4 < 0
x
(C) 2 < 0 (D) 2x  0

19. If 2  x  5, then |x| belongs to


(A) [2, 5] (B) [0, 5]
(C) (0, 5] (D) (0, 5)

P-2022-CBSE-P1-MATHEMATICS-LINEAR INEQUALITIES
15

20. If x  1, x, x + 1 are length of sides of a triangle then x belongs to


+
(A) R (B) [2, )
(C) (2, ) (D) (3, )

21. If a triangles has integral sides and length of two sides are 2014 and 2015, then number
of such triangles is
(A) 2400 (B) 4003
(C) 4027 (D) none of these

Assertion  Reason

22. Statement  I : x, 2x + 1, 3x + 2 can be sides of an equilateral triangle.


because
Statement  II : Sides of an equilateral triangle are equal and positive.
(A) Statement -1 is True, Statement -2 is True; Statement-2 is a correct explanation for
Statement-1
(B) Statement -1 is True, Statement -2 is True; Statement-2 is NOT a correct
explanation for Statement-1
(C) Statement -1 is True, Statement -2 is False
(D) Statement -1 is False, Statement -2 is True

23. Statement  I : If x2  2, x  R then x  .


because
Statement  II : Square of a real number is zero or positive always.
(A) Statement -1 is True, Statement -2 is True; Statement-2 is a correct explanation for
Statement-1
(B) Statement -1 is True, Statement -2 is True; Statement-2 is NOT a correct
explanation for Statement-1
(C) Statement -1 is True, Statement -2 is False
(D) Statement -1 is False, Statement -2 is True

COMPREHENSIONS

Read the following write up carefully and answer the following questions:

The marks obtained by a student of class X in first and second terminal examination are 70 and
40, respectively, then

24. The minimum marks he should get in the annual examination to have an average of at
least marks 65 marks
(A) 71 (B) 81
(C) 85 (D) 95

25. Marks he should get in the annual examination to have average between 40 to 60
(A) 40 to 70 (B) 30 to 60
(C) 10 to 50 (D) 10 to 70

Read the following write up carefully and answer the following questions:

If x, x + 1, x + 2 are three sides of a triangle then the value of x of

26. The triangle is acute angled triangle


(A) x > 1 (B) x < 3
(C) x > 3 (D) x  3

P-2022-CBSE-P1-MATHEMATICS-LINEAR INEQUALITIES
16

27. The triangle is obtuse angled triangle


(A) 1 < x < 3 (B) x < 3
(C) 1 < x > 2 (D) 0 < x < 3

28. The triangle is right angled triangle


(A) 3 (B) 1, 3
(C) 1 (D) 1, 3

Matrix Match

29. Match the following:


Column  I Column  II
(A) |x|  |x + 1|  1
(p) x    ,  
 2
(B) |x  1| < |x| < |x + 1| 1 
(q) x   , 
2 
(C) |x| > |x + 1|  1 1
(r) x   , 
 2 2
(D) |x  1| > |x| > |x + 1|  1 1
(s) x   , 
 2 2

30. Matching List Type(3 Column & 4 Rows)


Column – 1 Column – 2 Column – 3
Inequalities Inequalities number of integers in
solution set
(I) –12x < 30 (i) 3x – 7 > 5x – 1 (P) 12
(II) 4x + 3 < 5x + 7 (ii) 3(2 – x)  2(1 – x) (Q) 4
(III) 2(x + 1)  3(x + 3) (iii) x x (R) 11
 1
3 2
(IV) x x (iv) 5x – 3 < 7 (S) 0
  x  11
3 2

(I). Which of the following is correct combination


(A) (II) (ii) (Q) (B) (III) (i) (P)
(C) (II) (i) (S) (D) (II) (i) (S)

(II). Which of the following is not correct combination


(A) (IV) (iv) (R) (B) (II) (i) (R)
(C) (III) (i) (Q) (D) (I) (iv) (Q)

(III). Which of the following is correct combination


(A) (IV) (iii) (R) (B) (III) (i) (R)
(C) (III) (i) (P) (D) (I) (iv) (Q)

Numerical Based

31. A solution of nitric acid is to be kept between 30° and 35° Celsius, then find the difference
between the highest and lowest temperature in degree Fahrenheit if conversion formula
5
is given by C = (F  32), where C and F represents temperature in degree Celsius and
9
degree Fahrenheit respectively.

P-2022-CBSE-P1-MATHEMATICS-LINEAR INEQUALITIES
17

32. A solution of 8% hydrochloric acid is to be diluted by adding a 2% hydrochloric acid


solution to it. The resulting mixture is to be more than 4% and less than 6% of
hydrochloric acid. If we have 640 litres of the 8% solution and Q denotes the quantity of
Q  Q1
2% solution that will be added, then find the value of 2 where Q1 and Q2 marks the
320
minimum and maximum value of Q.

33. If V denotes the volume of water that will have to we added to 1125 litres of the 45%
solution of acid so that the resulting mixture will contain more than 25% but less than
V
30% acid content, then find the value of max .
100

MA
34. If IQ of a student can be given by IQ =  100 where MA and CA represents the
CA
mental age and chronical age of the student respectively. If IQ of a class of 12 years old
students varies from 80 to 140 and M1 and M2 represents the minimum and maximum
mental age of the group, then find the value of M2  M1  0.2.

P-2022-CBSE-P1-MATHEMATICS-LINEAR INEQUALITIES
18
ANSWERS TO ASSIGNMENT PROBLEMS

1. (i) x<3 (ii) x4

2. (i) (ii)
x2y+4=0
(0, 2)

y +8=2x
(4, 0)

(iii) y

2x=63y

3. (i) y (ii)

x =y
x+3y=30

x
x +y=9
x =1 3x+y=60

(iii) y

2x+y=4

x
x +y =3

2x3y=6

(iv) null or empty set.

P-2022-CBSE-P1-MATHEMATICS-LINEAR INEQUALITIES
19
(v)

(4, 6)
x+2y =16

x x+y =10
3x+2y =24

4. (6, 8), (8, 10), (10, 12).

5. Between 9.8m and 13.8m.

6. x = number in business class


y = number in economy class
 x  0, y  0, x + y  360, x  20, y  8x. y =8x

x+y =360

x =20

7. x = number of chairs, y = number of tables


 x  0, y  0, 5x + 2y  24, 4x + 8y  64

4x+8y=64

5x+2y =24

8. (i) x  ( 16,  7), (ii) x  [ 4,  2)

5 
9. x   , 
3 

10. n<5

11. If number of dolls is x and that of machine guns is y, then x  6, y  5, x  0, y  0,


2x + 2y  3, 7x + 9y  63.

12. For x radios and y two-in-ones, x + y  60, x + 5y  100, x  0, y  0.

P-2022-CBSE-P1-MATHEMATICS-LINEAR INEQUALITIES
20

Objective:

Level – I

1. A 2. B 3. B 4. C
5. C 6. B 7. B 8. C
9. D 10. B
Level – II

1. A 2. C 3. B 4. C
5. D 6. B 7. B 8. D
9. C 10. B 11. D 12. A
13. A 14. D 15. D 16. A
17. C, D 18. A, B, C, D 19. A, B 20. C, D
21. C 22. D 23. A 24. C
25. D 26. C 27. D 28. A
29. (A)  (q, s) (B)  (q) (C)  (p) (D)  (p)
30. (I). D (II). C (III). D
31. 9 32. 3 33. 9 34. 7

P-2022-CBSE-P1-MATHEMATICS-LINEAR INEQUALITIES
PINNACLE-CBSE

LOGARITHMS

CONTENTS SYLLABUS
 Introduction 1
 Logarithmic function 1 CBSE: Definition, Graph of Logarithm Function and
 Exercise 1 3
their Properties.
 Taking Logarithms 4
 Change of variable 4

 Exercise 2 5

 Exercise 3 6
 Common logarithms 6
 Miscellaneous Exercise 7
 Answers to Exercises 8
 Solved Problems 9
 Chapter Practice Problem 23
 Assignment Problems 25
 Answers to Chapter 38
Practice Problems
 Answers to Assignment
Problems 39
Introduction:
Logarithms and exponents are closely related. Repeated addition of the same number (or
quantity) evolves multiplication and the repeated multiplication of the same factor evolves
exponentiation. The inverse of these operations leads to extracting roots and taking logarithms.
9 3
e.g. 2 = 512, 9 = 729
i.e. ninth power of 2 is 512 and third power of 9 is 729. In general, for a positive real number a
and rational number m, we have am = b, where b is a real number. We can state that the mth
power of base a is b. We can also say that the logarithm of b to the base a is m.

Logarithmic Function:
If a > 0 and  1, then logarithm of a positive number m is defined as the index x of that power of
‘a’ which equals m i.e. loga m = x iff ax = m
 aloga m  m . ….(1)
The function defined by f (x) = loga x, a > 0, a  1 is called logarithmic function. Its domain is
(0, ) and range is R.
When base is ‘e’ then the logarithmic function is called natural logarithmic function and when
base is 10, then it is called common logarithmic function.

Graph of logarithmic functions


y = logax, a > 1
i) When 0 < x < 1, x = ay. y

That is, we have to choose those values of y for which


0 < ay < 1.
Since a > 1, y  0  y  (, 0).
(1, 0)
y O x
ii) When x = 1, x = a .
We have to choose that value of y for which x becomes 1
 y = 0.
Graph of logax, a > 1
iii) When x > 1, x = ay.
We have to choose those values of y for which x > 1.
Since a > 1, 0 < y < .

y = logax, 0 < a < 1, x = ay y

i) When 0 < x < 1,


we have to choose those values of y for which 0 < ay < 1.
Since 0 < a < 1, y > 0.
(1, 0)
O x
ii) When x > 1,
y
we have to choose those values of y for which a > 1.
Since a < 1, y < 0.
Graph of logax, 0<a<1

P-2022-CBSE-P1-MATHEMATICS-LOGARITHMS
2
Properties of Logarithmic Functions

1. aloga m  m (Fundamental logarithms identity).


Proof: From the definition of logarithm of loga m  x  a x  m
Substituting value of x, aloga m  m .

2. logaxn = nlogax
Proof: Let logaxn = t
Using definition of logarithm  xn = at
Now taking 1/n power, x = at/n
Using definition of logarithm
logax = t/n  t = nlogax  logaxn = n logax.
1
3. logam x  loga x
m
Proof: logam x  t  x = (am)t  x = amt
Now using definition of logarithm
1
logax = mt  t = 1/mlogax  logam x  loga x
m
loga b
4. logc a = = logb a  logc b
logb c

Proof: Let logba = x  a = bx,


logca = y  a = cy and logbc = z  c = bz. Hence a = cy = (bz)y = byz.
Also a = bx = byz x =yz logba = logca logbc.
In general, logba = logca  logdc  loged  ... logbk.
log a
Also = logca = b ,
logb c
log a 1
logaa = b  1 and logba = .
logb a loga b

5. loga x1 + loga x2 + … + loga xn = loga (x1  x2 … xn)


Proof: Let us consider y1 = logax1  x1 = a y1 ....(1)
y2
and y2 = logax2  x2 = a ....(2)
.... ... .... ...
.... ... .... ...
yn = logaxn  xn = a yn . ....(n)
Multiplying these n equations, we get
x1x2x3 .... xn = a y1 a y2 a y3 ....a yn
or x1x2x3 .... xn = a y1  y 2  y 3 .... yn 
or we can write it as y1 + y2 + y3 + ... + yn = loga(x1x2 .... xn)
 logax1 + logax2 + .... + logaxn = loga(x1x2 ....xn)
x 
6. logax1  logax2 = loga  1 
 x2 

Proof: Now from equations (1) and (2)

P-2022-CBSE-P1-MATHEMATICS-LOGARITHMS
3
x1 x 
 a y1  y2  y1  y2 = loga  1 
x2  x2 
x 
i.e. logax1  logax2 = loga  1 .
 x2 

Remark:
 1
 If x1 = 1 and x2 = x then loga1 logax = loga  
x  
 1
 loga   =  logax. ( loga 1 = 0)
x
 alogb c  clogb a
Proof: Let k = alogb c so that
logc
 
log k = log alogb c  logb c (loga) =
logb

 loga  logb a(logc)  log clogb a 
 k = c logb a .

loga x
Illustration 1: Prove that = 1 + loga b, for permissible values of letters involved in the
logab x
result.

Solution: LHS = loga x . logx ab


= loga x (logx a + logx b)
= loga x . logx a + loga x . logx b = 1 + loga b = RHS.

43
1
Illustration 2: Show that  log n
 log n 43 ! .
r 2 r

43
1 1 1 1
Solution:  log n
 
log2 n log3 n
 ... 
log43 n
r 2 r
43
1 1
Using the property loga b =
logb a
  log n
 logn 2  logn 3  ...  logn 43
r 2 r

Here the base is same and logarithm is in summation


Using the property, loga x1 + loga x2 = logz x1  x2
43
1
  logn  2  3  ...  43   logn  43! .
r 2 logrn

Exercise 1.
1 1 1
i) Prove that   1.
loga abc logb abc logc abc
ii) Solve loge (x  a) + loge x = 1.
xy 1
iii) If for x, y  R+, ln  (ln x + ln y), show that x = y.
2 2
iv) If log216 = 4, find log162.
(v) Find the value of x in (loge2) (logx625) = (log1016) (loge10).

P-2022-CBSE-P1-MATHEMATICS-LOGARITHMS
4
Algorithm of Solving Logarithmic Equations:

Solution of equations based on definition of a logarithm


We know by definition of logarithm that
b
logax  b  a = x, where x > 0

Illustration 3: Find the value of x satisfying the equation log3(5 + 4log3(x – 1)) = 2.

Solution: log3(5 + 4log3(x – 1)) = 2


Using definition of logarithm
 5 + 4log3(x – 1) = 32  4log3(x – 1) = 4  log3(x – 1) = 1
Again by definition of log
x – 1 = 3  x = 4.

Solving equations by using add/subtract property of logarithm and using fundamental


logarithmic identity
(i) When we see two log terms with same base, we use the below identity to solve the
equation
mloga x  nloga y  loga xm yn
(ii) Whenever we encounter a term aloga x , it can be removed by fundamental identity.

Illustration 4: Solve for x, log2(3 – x) + log2(1 – x) = 3.

Solution: log2(3 – x) + log2(1 – x) = 3  log2 (3  x) (1  x) = 3  (3  x) (1  x) = 23


 x2  4x – 5 = 0
x = 5,  1
But when x = 5, log2(3 – x) is not defined, So x =  1 is the solution.

Taking logarithms
In some equations, when we take logarithm on both sides the equation converts into a solvable
form.

Illustration 5: Solve for x: (x + 1)log(x + 1) = 100(x + 1).

Solution: Taking log10 on both sides.


log10 (x  1)log(x 1)  log10
100  x 1

log10  x  1  log10  x  1  2  log10  x  1


Now we substitute log10x + 1 = t

Change of variable
We can substitute loga x  t , a new variable and then solve the resulting equation.

2
Illustration 6: Solve for x:  log2 x   log2 x 3  2  0

Solution:  log2 x 2  3log2 x  2  0 .


Now log2 x  t  t2 – 3t + 2 = 0,  t = 1, 2
log2 x  1, log2 x  2  x = 2, x = 4.

P-2022-CBSE-P1-MATHEMATICS-LOGARITHMS
5
Passing to a new base
logb x
We can convert a log equation into a new base by the identity loga x 
logb a

Illustration 7: Solve for x: 1 + log2(x – 1) = logx – 14

Solution: 1 + log2(x – 1) = logx – 14


log2 4 2
logx–14 = 
log2 x  1 log2 x  1
2
Now equation becomes, 1 + log2x – 1 =
log2 x  1
Here we use the changing variable method.
log2 x  1  t  1 + t = 2/t
2
t +t–2=0
t = -2, 1
log2 x  1  2, log2 x  1  1
x = 5/4, 3

Illustration 8: Find the value of x for which log (x + 3) + log (x – 3) = log 27.

Solution: The given equation


log (x + 3) + log (x – 3) = log 27
 log (x + 3)(x – 3) = log 27
 log (x2 – 9) = log 27
 x2 – 9 = 27  x2 = 36  x = 6.

Exercise 2.
log a log b log c
i) If   , then prove that aa bb cc = 1.
bc ca ab
ii) Solve for x: log 2 + log (x + 2)  log (3x  5) = log 3.
 loga x  logb x 
iii) Prove that logabx = .
loga x  logb x
iv) logx+1(x2 + x  6)2 = 4.
v) log5120 + (x – 3)  2 log5(1 – 5x -3) =  log5(0.2 – 5x – 4)

Algorithm for Solving Logarithmic Inequalites:

While solving logarithmic inequalities, it is necessary to remember that the function


y = logax (a > 0, a  1, x > 0) is decreasing when a  (0, 1) and increasing if a  (1, )

Therefore if a  (0, 1), log(f(x)) < log(g(x))  f(x) > g(x)


If a  (1, ), loga(f(x) < logag(x))  f(x) < g(x)
This can also be considered as, when we take antilogarithm to remove a logarithm with base ‘a’,
nature of inequality changes if a  (0, 1) and it remains same if a  (1, )
Based on the above thought, we conclude
(i) If a > 1 loga x  0  x  1

P-2022-CBSE-P1-MATHEMATICS-LOGARITHMS
6
loga x  p  x  ap
loga x  0  0  x  1
loga x  p  0  x  ap
(ii) If 0 < a < 1 loga x > 0  0 < x < 1
loga x > p  0 < x < ap
loga x < 0  x > a
loga x < ap  x > ap

4x  6
Illustration 9: Find the solution set of the inequality log1 / 5 0.
x

Solution: The base lies in (0, 1)


4x  6
Taking antilog 0 < 1
x
Solving this we get x  [-2, -3/2)

Illustration 10: Find the solution set of the inequality log 2 x 3  x 2  log 2 x 3   2x  3  .

3
Solution: Case1: 0 < 2x + 3 < 1  <x<1
2
x2 > (2x + 3)
x2 – 2x – 3 > 0
 (x – 3) (x + 1) > 0  x  (-, -1)  (3, )
But interval in consideration is x  ( 3/2,  1)
Hence x  ( 3/2,  1)

Case 2: 2x+ 3 > 1


 
 x >  1  log 2x 3  x2  log 2x 3   2x  3 
 x < 2x + 3 ( base > 1)  x2 – 2x – 3 < 0
2

 (x – 3) (x + 1) < 0  x  ( 1, 3)
Complete Solution: x  (-3/2,  1)  ( 1, 3)

Exercise 3:
Solve for x:
log 3  x 6   x2  x 
i) 3 log3 x 1
3 3 ii) log0.5  log6 0
 x  4 
 x 1  1 1
iii) log x 6  log2 0 iv) Prove that < log345 <
3 
x2 3 2
loga logb logc
v) If a, b, c are positive reral numbers such that   , then prove that
bc c a ab
b+c c+a a+b a b c
(A) a +b +c 3 (B) a + b + c  3.

Common Logarithms:
In common logarithms the base is always taken as 10.

P-2022-CBSE-P1-MATHEMATICS-LOGARITHMS
7
Note:
p
 Any positive number k in decimal form is written in the form k = m  10 where p
is an integer and 1 < m < 10. This is called the standard form of k.

Miscellaneous Exercise
3/2
i) Show that 3 log 4  2 log 6 + log (18) = log 96 2 .
ii) If log0.3 (x – 1) < log0.09 (x – 1), find the interval in which x lies.
2
iii) Solve for x: x log x ( x  3 )  16 .
 8  2
iv) Find x if it is given by l og8  2   3   log8 x  .
x 
v) Find x if 3 loga x  3  x loga 3  2 .

P-2022-CBSE-P1-MATHEMATICS-LOGARITHMS
8
ANSWERS TO EXERCISES

Exercise 1.
a a2 1
(ii) x=  e (iv)
2 4 4
(v) 5

Exercise 2.
19
(ii) x= (iv) x=1
7
(v) x=1

Exercise 3:
(i) (6, ) (ii) ( 4,  3)  (8, )
(iii) ( 6,  5)  ( 3,  2)

Miscellaneous Exercise

(ii) x>2 (iii) no solution


1
(iv) x= ,2 (v) x = 2 log3 a
8

P-2022-CBSE-P1-MATHEMATICS-LOGARITHMS
9
SOLVED PROBLEMS

Subjective:

Level – 0

Problem 1. Find the value of x satisfying the equation


log5(2x2 + 3x + 2)1/4 = log25(2x).

Solution: Clearly x > 0 and


1 log5  2x  1
log5  2x2  3x  2   = log  2x 
4 log5 25 2
 2x2 + 3x + 2 = (2x)2
or 2x2  3x  2 = 0
or (x  2) (2x + 1) = 0  x = 2.

1 1 1
Problem 2. If n = 1000!, evaluate   ......  .
log2 n log3 n log1000 n

Solution: We have
1 1 1
  ...... 
log2 n log3 n log1000 n
log2 log3 log1000
=   ...... 
logn logn logn
log2  log3  log 4  ......  log1000
=
logn
log2  3  4  .........1000 log(1000)!
= =  1.
logn logn

10
Problem 3. If log 8
x , then find x.
3
10 log x log x 2log x
Solution: Here  log 8
x  =
3 log 8 (1/ 2)log8 log8
 log810 = log x6  810 = x6
 x = (8)10/6 = (8)5/3 = 25 = 32.
 1
log2.5  
Problem 4. Find the value of  0.16  2 .

 1  1
log5 / 2   2log2.5  
Solution: Let A =  0.16  2 =  0.4  2
2 2
 1  1
log  log
 2  2.5  2   5  5 / 2  2 
=   = 
5  2
log5 / 2 22
5
=   = 22 = 4.
 2

P-2022-CBSE-P1-MATHEMATICS-LOGARITHMS
10
Level – I

Problem 5. If ln2. logb625 = log1016. ln10 then find the value of b.

Solution: We have ln2. logb54 = log1024. ln 10


 4 ln2. logb5 = 4log102. ln 10
ln2
 ln2. logb5 = . ln 10
ln10
 logb5 = 1  b = 5.

Problem 6. If in a right angled triangle, a and b are the lengths of sides and c is the length of
hypotenuse and c  b  1, c +b  1, then show that
logc+ba + logcba = 2logc+ba . logcba .

Solution: We know that in a right angled triangle


c2 = a2 + b2
or c2  b2 = a2. ....(1)
1 1
Also logc + ba + logc-ba = 
loga  c  b  loga  c  b 

=
loga  c  b   loga  c  b 
=

loga c 2  b2 
loga  c  b  loga  c  b  loga  c  b  loga  c  b 
loga a2
= (using (1) )
loga  c  b  loga  c  b 
2
=
loga  c  b  loga  c  b 
= 2 logc+ba . log(c b) a = RHS.
Alternative Solution
lna lna
LHS = logc + ba + logc-ba = 
ln(c  b) ln(c  b)
lna  ln(c 2  b2 ) lna  2lna
= =
ln(c  b)ln(c  b) ln(c  b)ln(c  b)
= 2 logc+ba . logc-ba = RHS.

Problem 7. Given a positive term geometric progression a, a1, a2,... and an arithmetic
progression b, b1, b2,.... with positive common difference. Show that there exists
a system of logarithm for which logan  bn=loga  b (for any n). Find the base of
the system.

Solution: Let d be the common difference of the A.P. and r be the common ratio of the
G.P. so that,
n
an = ar , ....(1)
bn = b + nd. ....(2)
Taking logarithms on both sides of (1) to base  (  1,  > 0), we get
log an  log a  nlog r . ....(3)
Operating (3)  (2) leads to

P-2022-CBSE-P1-MATHEMATICS-LOGARITHMS
11
log an  bn  log a  nlog r  b  nd . ....(4)
Now, in order that the RHS of (4) reduces to loga  b, we must have
n logr  nd = 0
or logr = d  r = d   =r1/d.
Hence there exists a system of logarithm to base r1/d such that
logan  bn = loga  b.
Alternative Solution:
Let d be the common difference of the A.P and r be the common ratio of the
G.P., so that
bn = b + nd and an = arn  ln an = ln a + n ln r
b b
= ln a + n lnr
d
= ln a + (bn – b) ln r1/d  logx an = logx a + bn- b, where x = r1/d.

Problem 8. Show that log418 is an irrational number.

Solution: We have log418 = log4(32  2) = 2 log43 + log42


log2 3 1 1
=2   log2 3  .
log2 4 log2 4 2
Let us assume the contrary, that this number log23 is a rational number.
p
 log23 = . Since log23 > 0, both the numbers p and q may be regarded as
q
natural numbers
 3 = 2p/q  2p = 3q.
But this is not possible for natural numbers p and q. The resulting contradiction
completes the proof.

Problem 9. 
Solve for x, log7 log5 x + 5 + x = 0. 
Solution: From the given equation, log5  
x  5  x  70  1  log5 5
 x5  x = 5
or x  5  5  x  x + 5 = 25 + x - 10 x
or x = 2  x = 4.

Problem 10. Solve for x and y, the equations log100|x + y| = 1/2 and log10 y– log10 |x| = log100 4.

Solution: From log100|x + y| = 1/2, we get


1/2
|x + y| = 100 = 10 …(1)
From log10 y – log10 |x| = log100 4 = log10 2, we have
y
 2  y 2  4x 2
|x|
 x2 + y2 + 2xy = 100
or 5x2 + 4x|x| = 100
10 20
Also, x = for x > 0  y =
3 3
and x = 10 for x < 0  y = 20.

P-2022-CBSE-P1-MATHEMATICS-LOGARITHMS
12
Level – II

 8  2
Problem 11. Solve for x, log8  2   3  log8 x  .
x 

Solution: On solving, we have


2 2
log8 8  log8 x 2  3  log8 x   3z + 2z  1 = 0, where z = log8 x; x > 0
 z = log8 x =  1 or 1/3  x = 1/8 or 2.

Problem 12. Find the solution set of the inequality log0.3  x  1 < log0.09  x  1 .

Solution: Given log 0.3(x – 1) < log 0.09 (x – 1)


1
 log 0.3 (x – 1) < log
 0.3 2
 x  1 = log0.3  x  1
2
 2 log0.3  x  1 < log 0.3 (x– 1)
 log 0.3 (x – 1)2 < log 0.3 (x – 1)
 (x – 1)2 > (x – 1) [  base = 0.3 < 1]
 (x – 1) (x – 2) > 0  x < 1 or x < 2
But logarithm function is defined if x – 1 > 0  x > 1.
Hence, the solution is x > 2 or x  (2, ).

Problem 13. Find the value of x for the given equation


6 loga x log10 a  loga 5 log 2 x  log 2 2
a  3log10 x log10 10 = 9 10 2
.
5

6 loga x log10 a  loga 5 log 2 x  log 2 2


Solution: a  3log10 x log10 10 = 9 10 2

5
1 1
6 log10 xloga 5 log10 x  log2 2
 a  3log10 x 1 = 9 2 2
5
6 loga 5  log10 x
 3 10   3 10 
log x 1 log x 1
 a
5
6 log10 x
 3 10   3 10 
log x 1 log x 1
 5
5
 6  5log10 x 1  3 10  1  32 
log x 1

log x 1 1
 5  10 10  5 
     
3 6 3
 log 10 x – 1 = 1
2
 log 10 x = 2  x = 10 = 100.

Problem 14. Find the value of 3 log 2 + log sin x + log cos x + log cos 2x + log cos 4x.

Solution: 3 log 2 + log sin x + log cos x + log cos 2x + log cos 4x
= log (8 sin x cos x cos 2x cos 4x)
= log (4 sin 2x cos 2x cos 4x)
= log (2 sin 4x cos 4x)
= log (sin 8x)

P-2022-CBSE-P1-MATHEMATICS-LOGARITHMS
13
1 1 1 1
Problem 15. Find the value of + + + …+ .
log2 N log3 N log4 N log2005 N

1 1 1 1
Solution:   + …+
log2 N log3 N log4 N log2005 N
 log N 2 + log N 3 + log N 4 + … + log N 2005
 log N(2  3  4  2005)
1
 n =
log 2005 N

yzlog  yz  zx log  zx  xy log  xy 


Problem 16. If = = , then prove that xx = yy = zz.
yz zx xy

yzlog  yz  zx log  zx  xy log  xy 


Solution: = =
yz zx xy
Dividing each term by xyz, we have
yzlog  yz  zxlog  zx  xy log  xy 
= =
xyz  y  z  xyz  z  x  xyz  x  y 
log y  logz log z  log x log x  log y
   = K (say)
x y  z y z  x  zx  y
 log y + log z = K(xy + xz) …(1)
log z + log x = K(yz + xy) …(2)
log x + log y = K(xz + yz) …(3)
Adding we get, 2(log x + log y + log z) = 2K(xy + yz + zx)
i.e. log x + log y + log z = K(xy + yz + zx) …(4)
Subtracting (1) from (4), we get log x = Kyz
 x log x = K xyz
Similarly y log y = K xyz and z log z = Kxyz
 x log x = y log y = z log z
 log xx = log yy = log zz
 xx = yy = zz

P-2022-CBSE-P1-MATHEMATICS-LOGARITHMS
14

Objective:

Level – 0

True / False

1 1 1 n  n  1
Problem 1. The sum of the series    ..... upto n terms is loga2
log2 a log4 a log8 a 2

Solution: True
The given expression can be written as
loga2 + loga4 + loga8 + .... upto n terms
= loga2 + 2 loga2 + 3 loga2 + .... n loga2
n  n  1
= (loga2) [1+ 2 + ... n] = loga 2 .
2

Problem 2. The value of ‘b’ satisfying log 8


b  3 13 is 30

Solution: False
10
We have log 8
b  3 13  log23 / 2 b 
3
2 10
log2 b   log2b = 5  b = 25.
3 3

Problem 3. The value of x, satisfying loga(1 + logb{1 + logc(1 + logpx)}) = 0, is 1

Solution: True
The given equation yields 1 + logb{1 + logc(1 + logpx)} = 1
 logb{1 + logc(1 + logpx)} = 0  1 + logc(1 + logpx) = 1
 logc(1 + logpx) = 0  1 + logpx = 1
 logpx = 0  x = 1.

Problem 4. For 0 < x < 1, the value of log(1 + x) + log(1 + x2) + log(1 + x4) .... to  is logx

Solution: False
2
The given expression = log [(1 + x) (1 + x ) ..... to  ]
 1 
= log[ 1+ x + x2 + x3 + .... to ] = log  
 1 x 
=  log(1  x).

3x -4
Problem 5. The value of x satisfying 184x3 = 54 2   is 6

Solution: True
3x  4
Here 18
4x3

= 54 2  .
Taking log on both sides, we get

P-2022-CBSE-P1-MATHEMATICS-LOGARITHMS
15
(4x  3) log18 = (3x  4) log54 2
3
 (4x  3) log18 = (3x  4) log(18)3/2  (4x  3) = (3x  4).
2
 x = 6.

Fill in the Blanks

Problem 6. Number of solutions of the equation log2(32x2 + 7) = 2 + log2(3x1 + 1) is ______

Solution: 2
Here log2(32x2 + 7) = log24 + log2(3x1 + 1)
x1
= logx 4(3 + 1)
2(x1) x1
 3  4.3 + 3 = 0
x1 x1
 (3  1) (3  3) = 0
 3x1 = 1 = 30 and 3x1 = 3 = 31
 x = 1 and x = 2.

Problem 7. If log303 = c, log305 = d, then log308 is equal to _______

Solution: 3(1 – c – d)
We have log308 = log3023 = 3 log30 30
15
= 3  log30 30  log30  3  5  
= 3[1  c  d]

log 9 36 4/log 7 9
Problem 8. The value of 81(1/log5 3)  27 3 is equal to _____

Solution: 890
81(1/log5 3)  27log9 36  34/log7 9
1
3. log3 36
 3 4 log3 5  3 2  3 4 log9 7
4 3/2 4/ 2
 3log3 5  3log3 36  3log3 7
 5 4  363/2  72  890 .

Level – I

MCQ Single Correct

 16   25   81 
Problem 9. 7log    5log    3log   is equal to
 15   24   80 
(A) 0 (B) 1
(C) log2 (D) log3

Solution: C
 167 255 813 
Given expression = log  7 . 5 . 3   log2 .
 15 24 80 

P-2022-CBSE-P1-MATHEMATICS-LOGARITHMS
16
Problem 10. If log4 5  a and log5 6  b, then log3 2 is equal to
1 1
(A) (B)
2a  1 2b  1
1
(C) 2ab  1 (D)
2ab  1

Solution: D
1
ab  log4 5.log5 6  log4 6  log2 6
2
1
ab  (1  log2 3)  2ab  1  log2 3
2
1
 log3 2  .
2ab  1

Problem 11. If log5 a.loga x  2, then x is equal to


(A) 125 (B) a2
(C) 25 (D) None of these

Solution: C
loge a loge b loge c
log5 a.loga x  2      1  x  52  25 .
loge b loge c loge a

Problem 12. If a2  4b2  12ab, then log(a  2b) is


1 a b
(A) [loga  logb  log2] (B) log  log  log2
2 2 2
1 1
(C) [loga  logb  4log2] (D) [loga  logb  4log2]
2 2

Solution: C
a2  4b2  12ab
 a2  4b2  4ab  16ab  (a  2b)2  16ab
 2log(a  2b)  log16  loga  logb
1
 log(a  2b)  [loga  logb  4log2]
2

Problem 13. If A  log2 log2 log4 256  2log 2


2, then A is equal to
(A) 2 (B) 3
(C) 5 (D) 7

Solution: C
A  log2 log2 log4 256 + 2log2 1/ 2 2
1
 log2 log2 log4 44  2  log2 2
(1/ 2)
 log2 log2 4  4  log2 log2 22  4
 log2 2  4  1  4  5 .

P-2022-CBSE-P1-MATHEMATICS-LOGARITHMS
17
Level – II

Problem 14. If log10 x  y, then log1000 x 2 is equal to


(A) y 2 (B) 2y
3y 2y
(C) (D)
2 3

Solution: D
2 2
log1000 x 2  log103 x 2  2log103 x  log10 x  y .
3 3

Problem 15. If x  loga (bc), y  logb (ca),z  logc (ab), then which of the following is equal to 1
(A) x  y  z (B) (1  x)1  (1  y)1  (1  z)1
(C) xyz (D) None of these

Solution: B
x  loga bc  1  x  loga a  loga bc  loga abc
 (1  x)1  logabc a
 (1  x)1  (1  y)1  (1  z)1  logabc a  logabc b  logabc c
 logabc abc  1 .

Problem 16. If a  log24 12,b  log36 24 and c  log48 36, then 1+abc is equal to
(A) 2ab (B) 2ac
(C) 2bc (D) 0

Solution: C
log12 2log2  log3
a  log24 12  
log24 3log2  log3
3log2  log3
b  log36 24 
2(log2  log3)
2(log2  log3)
c  log48 36 
4log2  log3
2 log2  log3
 abc 
4log2  log3
6log2  2log3 3log2  log3
 1  abc   2.  2bc .
4log2  log3 4log2  log3

Problem 17. If a x  b,b y  c,c z  a, then value of xyz is


(A) 0 (B) 1
(C) 2 (D) 3

Solution: B
a x  b  x loga  logb
logb
 x  loga b
loga

P-2022-CBSE-P1-MATHEMATICS-LOGARITHMS
18
Similarly y  logb c,z  logc a
x lna  ylnb  xy ln(ab)  k (say) xyz  loga b.logb c.logc a  1.

Problem 18. If log10 2  0.30103,log10 3  0.47712, the number of digits in 312  28 is


(A) 7 (B) 8
(C) 9 (D) 10
Solution: C
y  312  28  log10 y  12log10 3  8log10 2
 12  0.47712  8  0.30103
 5.72544  2.40824  8.13368
 Number of digits in y = 8 + 1 = 9.

n
1
Problem 19.  log (a)

n 1 2n

n(n  1) n(n  1)
(A) loga 2 (B) log2 a
2 2
(n  1)2 n2
(C) log2 a (D) None of these
4
Solution: A
n n
1
 log   loga 2n = x  1
n 1 2n
(a) n 1

n(n  1) n(n  1)
= loga 2.  loga 2 .
2 2

Problem 20. The solution of the equation log7 log5 ( x 2  5  x )  0


(A) x  2 (B) x  3
(C) x  4 (D) x  2
Solution: C
log7 log5 ( x 2  5  x )  0  log7 1
 134  6292  [112  ( 13 )2 ]  2.11. 13
 (x 2  5  x)1/2  5
 (x 2  x  5)  25  x 2  x  20  0
 (x  4)(x  5)  0  x  4,  5  x  4 .

Problem 21. log4 18 is


(A) A rational number (B) An irrational number
(C) A prime number (D) None of these
Solution: B
1 1
log4 18  log2 (32.2)  (2log2 3  log2 2)
2 2
1
 log2 3  , which is irrational.
2

P-2022-CBSE-P1-MATHEMATICS-LOGARITHMS
19
log (0.1 0.01 0.001......)
Problem 22. The value of (0.05) 20
is
1
(A) 81 (B)
81
1
(C) 20 (D)
20

Solution: A
 0.1 
2 log20  
log (0.1 0.01......)  1   10.1 
(0.05) 20
 
 20 
2
 20 2 log20 (1/9)  202 log20 9  20log20 9  92  81 .

Problem 23. If a, b, c are distinct positive numbers, each different from 1, such that
[logb alogc a  loga a]  [loga blogc b  logb b] [loga c logb c  logc c]  0, then abc =
(A) 1 (B) 2
(C) 3 (D) None of these
Solution: A
[logb a.logc a  loga a]  [loga b.logc b  logb b] [loga c logb c  logc c]  0
9
 x
4
1
 [(lna)3  (lnb)3  (lnc)3  3lna.lnb.lnc]  0
lna.lnb.lnc
 (lna)3  (lnb)3  (lnc)3  3lna.lnb.lnc  0
 lna  lnb  lnc  0
 ln(abc) = ln 1, [a3  b3  c 3  3abc  0
 a  b  c  0] ,  abc  1 .

Problem 24. If log12 27  a, then log6 16 


3a 3a
(A) 2. (B) 3.
3a 3a
3a
(C) 4. (D) None of these
3a

Solution: C
log27 3log3 2alog2
a   log3 
log12 log3  2log2 3a
log16 4log2 4log2 4(3  a) 3a
log6 16      4. .
log6 log2  log3 2alog2 3  a  2a 3a
log2 
3a

MCQ Multi Correct

Problem 25. If logk x.log5 k  logx 5,k  1,k  0, then x is equal to


1
(A) k (B)
5
(C) 5 (D) None of these

P-2022-CBSE-P1-MATHEMATICS-LOGARITHMS
20
Solution: B, C
logk x.log5 k  logx 5  log5 x  logx 5
1
 logx 5   1  2B  0  logx 5  1
logx 5
1
 x  1  5  x  5, .
5

Problem 26. Solution of log3x = 1 + logx9 are


(A) 9 (B) 2
(C) 1 (D) 1/3

Solution: A, D
log3x = 1 + 2 logx 3
 (log3x)2 – log3x – 2 = 0
log3 = 2, – 1
 x = 9, 1/3.

Problem 27. If log2(32x 2 + 7) = 2 + log2(3x – 1 + 1), then x is


(A) 0 (B) 1
(C) 2 (D) None

Solution: B, C
32x–2 + 7 = 4(3x–1 + 1)
(3x–1)2 – 4.3x–1 + 3 = 0
3x–1 = 1, 3
x = 1, 2

Paragraph Type

Let f(x) = log{x}[x], g(x) = log[x]{x}, h(x) = log{x}{x}


(where [.], {.} denotes greatest integer function and fractional part).

Problem 28. For x  (1, 5), f(x) is not defined at how many points
(A) 5 (B) 4
(C) 3 (D) 2

Solution: C
{x}  0  x  2, 3, 4.

Problem 29. If A = {x: x  domain of f(x)} and B = {x: x  domain of g(x)}, then A – B will be
(A) (2, 3) (B) (1, 3)
(C) (1, 2) (D) none of these

Solution: C
[x]  2  x  [2, 0)
A = {x: x  I+}
B = {x: x  I+  (2, )}
A – B = (1, 2).

Problem 30. Domain of h(x) is


(A) R (B) I
+
(C) R – I (D) R –I

P-2022-CBSE-P1-MATHEMATICS-LOGARITHMS
21
Solution: C
{x}  0  x  R – I.

Matrix Match Type

Problem 31. Match the following


Column – I Column – II
3 5
(A) log3 x 2 log3 x  (p) number of solutions = 2
x4 4  3
(B) log 3  sin x  2 
2 cos x  2,  2  x  2 (q) number of solutions = 3
x2
(C) log0.2 1 (r) number of solutions = 1
x
log2  x  5   6  x
infinite number of
(D) (s)
solutions

Solution: A  q; B  p; C  s; D r
log3x = t  x = 3t
3 5 1
t3  t 2  t
 34 4  32
3 2
 3t + 4t – 5t – 2 = 0
 (3t + 1)(t + 2)(t – 1) = 0
1 1
 x 3 , ,3
3 9
x2 1

x 5

Decimal Type

Problem 32. If log3 (log2 x) + log1/3 (log1/2 y) = 1; xy2 = 9. x, y  R, then x + y is


Solution: 729.11
log3 (log2 x) + log1/3 (log1/2 y) = 1 ……(1)
Now, log1/2 y = (log2 y)  (log1/2 2) = – log2 y ……(2) (since log1/2 2 = –1)
Also, log1/3 (log1/2 y) = log3 (log1/2 y)  log1/3 3
= – log3 (log1/2 y) (since log1/3 3 = –1)
= – log3 (–log2 y) ……(3) (by (2))
Now from (1) and (3), we have
log3 (log2 (x)) – log3 (–log2 y) = 1
  log2 x    log2 x 
 log3   =1   =3
 log2 y   log2 y 
 3 log2 y + log2 x = 0  xy3 = 1
2
also xy = 9
1
From the above we have y = and x = 729, so x + y = 729.11
9

Problem 33. Solve the equation


log3x+4(4x2 + 4x + 1) + log2x+1(6x2 + 11x + 4) = 4
Solution: 0.75
The given equation is

P-2022-CBSE-P1-MATHEMATICS-LOGARITHMS
22
2
ln  2x  1 ln  3x  4  2x  1 
 4
ln  3x  4  ln  2x  1
2ln  2x  1 ln  3x  4 
  1 4
ln  3x  4  ln  2x  1
ln  2x  1
 2t2 − 3t + 1 = 0 where t =
ln  3x  4 
 (t − 1) (2t − 1) = 0  t = 1 or t = 1/2
When t = 1, we have 3x + 4 = 2x + 1  x = −3
When t = 1/2, we have 3x + 4 = (2x + 1)2
 4x2 + x − 3 = 0  (x + 1) (4x − 3) = 0  x = −1, 3/4
But x = −1 and −3 make the base of the original equation negative.
3
 Only solution is x = = 0.75.
4

P-2022-CBSE-P1-MATHEMATICS-LOGARITHMS
23
CHAPTER PRACTICE PROBLEMS

Subjective:

Level – I
1. Given log 2 = .30103, then find the number of ciphers between the decimal point and first
1000
 1
significant digit in   .
 2

log 27  log8  log 1000


2. Find the value of .
log1 2

3. If 0 < x < 1, then find the sum of the series


log(1+ x) + log(1 + x2) + log (1+ x4) + log(1 + x8) + … to n terms

Level – II
4. Find the value of x satisfying the simultaneous equations a2x b3y = m5 and a3x b2y = m10.

5. If a   b and a + b  1, then find the value of x satisfying the equation (a4 – 2a2b2 + b4)x–1
= (a – b)2x (a + b)–2 .

Objective:

MCQ Single Correct

1. The logarithm of 144 to the base 2 3 is


(A) 4 (B) 2
(C) 3 (D) 12
3
36  6  16
2. 3 log + log   – 2 log is equal to
25  27  125
(A) 2 (B) log 2
(C) log 3 (D) log 5

loga logb logc x y z


3. If = = , then a b c is equal to
yz zx xy
(A) xaybzc (B) xyz
(C) abc (D) 1

4
4. On simplifying log 729 3 9 127 4/3 is equal to
2
(A) 2 log 3 (B) log3
3
1
(C) log 3 (D) log 3
3

5. If a, b, c be the pth, qth and rth terms of a G.P., then (q – r) log a + (r – p)log b + (p – q) log
c is equal to
(A) 0 (B) abc
(C) ap + bq + cr (D) none of these

P-2022-CBSE-P1-MATHEMATICS-LOGARITHMS
24
6. If a2 + b2 = 34ab, then 2 log(a + b) – log a – log b is equal to
(A) 0 (B) log 6
(C) 2 log 6 (D) none of these

x
7. If log (x3 y4) = m + 2n and log (x4 y3) = n + 2m, then log   is equal to
y
m
(A) (B) m  n
n
mn
(C) (D) m + n
7

1 1 1
8. If x = log a (bc), y = log b(ca) and z = log c (ab), then   is equal to
x 1 y 1 z 1
(A) 1 (B) log abc
(C) log abc(a + b + c) (D) none of these

a3b3 b3c 3 c 3 d3
9. log + log + log – 3 log b2 c is equal to
c3 d3 a3
(A) 3(log a + log b + log c) (B) 3 log a
(C) –3(log b + log c) (D) 0

a x 1
10. If a, b, c are in G.P. and = b2x, then x is equal to
c x 1
(A) 1 (B) log c b
(C) log ac (D) log b a

MCQ Multi Correct

1. If logk x.log5 k  logx 5,k  1,k  0, then x is equal to


1
(A) k (B)
5
(C) 5 (D) None of these

logx log y logz


2. If   , then which of the following is true
b c c a a b
(A) xyz  1 (B) x a yb zc  1
(C) xb c yc  a za b  1 (D) xyz  xa yb zc

3 5
(log3 x)2  log3 x 
3. If x 4 4  3 then x has

(A) One positive integral value (B) One irrational value


(C) Two positive rational values (D) None of these

Numerical Based

1. The value of (log20.5 4) is

2. The value of log3 4log4 5log5 6log6 7log7 8log8 9 is

P-2022-CBSE-P1-MATHEMATICS-LOGARITHMS
25
ASSIGNMENT PROBLEMS

Subjective:

Level – 0

10
1. Determine b if log 8
b .
3

2. Evaluate:
(A) log2(log381) (B) log927  log279 (C) log6 216 6

3. If log V = log 4 – log 3 + log  + 3log r, write down the formula for V in terms of  and r

4. Prove that
(yz)logy  logz  (zx)logz  logx  (xy)logx  logy = 1

5. Write down the logarithmic equations for each of the following:


PV mm
(A) R  (B) F  1 2 2
T d
l 1 2
(C) T  2 (D) V  r h
g 3

7.  
Find the value of x, satisfying loga 1  logb 1  logc 1  logp x   0 .
1
8. Solve x if log81 3  for x > 0.
x2

9. If characteristic of log10x is 3, find the number of digits in x.

10. Find the value of logba.logcb.logac.

16 25 81
11. Find the value of 7log  5log  3log .
15 24 80

1 1 1
12. Find the value of   .
1  logb a  logb c 1  logc a  logc b 1  loga b  loga c

13. Find the value of x if log3(x1) = 2.

14. Find the value of x for which 23x = log216.

 x2   y2   z2 
15. Prove that log    log    log    0 .
 yz   zx   xy 

P-2022-CBSE-P1-MATHEMATICS-LOGARITHMS
26
ab 1
16. If log     loga  logb  , prove that a = b.
 2  2

17. If log10x = a, find the values of 10a1 in terms of x.

1 1 1 1
18. Show that     .
log2 x log3 x log43 x log43 ! x

19. Find the value of log2log2log216.

20. If log42 + log44 + log4x + log416 = 6, then find the value of x.

21. If logkx.log5k = logx5, k  1, k > 0, then find the value of x.

22. If 2x.3x+4 = 7x, then find the value of x.

 1 1 
23. Find the value of   .
 log3 12 log 4 12 

1 1
24. Prove that  >2
log3  log4 

25. If log1/ 2
sin x  0,x  [0, 4], then the number of values of x which are integral multiples of

, is
4

P-2022-CBSE-P1-MATHEMATICS-LOGARITHMS
27
Level – I

2 loga b
1. If logax, logbx, logcx are in A.P. where x  1, then show that c =  ac  .

1 1
2. If a2 + b2 = 7ab, prove that log  a  b   loga  logb .
3 2

3. If log2x + log2y  6, determine the least possible value of (x + y).

log10  x 1
4. Solve :  x  1 = 100(x + 1).

5. Solve the following equation for x:


log3  log2 x  2
9  log2 x   log2 x   1 .

1 1 1

6. If y = a1loga x and z = a1loga y then prove that x = a1loga z .

7. Find the value of x which satisfies the equation


log(x + 5)  log(3x + 25) = log(x  15)  log17.

8. Prove that xlogy  logz . ylogz  logx . zlogx  logy = 1.

9. If log3 2, log3 (2x – 5), log3 (2x – 7/2) are in A.P., determine the value of x.

10. If A  log2 log2 log4 256  2log 2


2, then A is equal to

11. If a x  b,by  c,c z  a, then value of xyz is

12. If log10 2  0.30103,log10 3  0.47712, the number of digits in 312  28 is

P-2022-CBSE-P1-MATHEMATICS-LOGARITHMS
28
Level – II

1. Find values of x such that log(x2 – 2x – 2)  0.

1 1 1 
log2.5   2  3  
2. Find the value of (0.16) 3 3 3  .

2
3. 
Solve for x : logx 1 x 2  x  6  4.

4. 
Solve for x : log5 x2  11x  43  2 . 
x y  z  x y z  x  y  zx  y  z
5. If   , prove that yzzy = zxxz = xyyx .
log x log y logz

 2x  8 
6. Solve for x : log1.5    0.
 x2 

7. Find all real numbers x which satisfy the equation 2log2 (log2 x)  log1/ 2 (log2 (2 2x))  1 .

8. Solve for x and y, the equations log10 x + log10 x1/2 + log10 x1/4 + ……. to  = y
1  3  5  .......  (2y  1) 20
and  .
4  7  10  .......  (3y  1) 7log10 x

9. Find the solution set of the inequality


3
log|sinx| (x2  8x + 23) > .
log2 sin x

3 5
(log2 x)2  log2 x 
10. Solve the equation x 4 4  2.

P-2022-CBSE-P1-MATHEMATICS-LOGARITHMS
29

Objective:

Level – I

1. The value of x, satisfying the inequality log0.3(x2 + 8) > log0.39x, lies in


(A) 1 < x < 8 (B) 8 < x < 13
(C) x > 8 (D) none these
 1 
2. If a = log1/ 2 0.125 and b = log3   then
 24  17 
(A) a> 0, b > 0 (B) a < 0, b < 0
(C) a > 0, b < 0 (D) a < 0 , b > 0
3. The number log27 is
(A) rational number (B) irrational number
(C) a prime number (D) an integer

4. The value of x, satisfying 3 4log9  x 1  22log2 x  3 , is


(A) x = 0 (B) x = 1
(C) x = 2 (D) x = 3

1 1
5. The value of  is
log3  log4 
(A) 2 (B) less than 2
(C) more than 2 (D) none of these
6. The value of |logba + logab| where a and b are positive numbers is always
(A)  2 (B)  2
(C) = 2 (D) none of these

16 25 81
7. The value of 7log  5log  3log is
15 24 80
(A) log2 (B) log 3
(C) 0 (D) none of these

2
8. If log0.3(x – x – 5)70, then x lies in the interval
(A) (3, -2) (B) (3, 2)
(C) (-3, 2) (D) (-2, 3)

1 1 1 1
9.    .......  is equal to
log2 n log3 n log4 n log43 n
logn 1
(A) (B)
log(43!) log 43!n
(C) log43! n (D) none of these

10. If log0.3(x  1) < log0.09(x  1), then x will lie in the interval
(A) (2, ) (B) (1, 2)
(C) (2, 1) (D) none of these

P-2022-CBSE-P1-MATHEMATICS-LOGARITHMS
30
loga  logb a 
11. The value of is
logb  loga b 
(A) logab (B) logba
(C)  logab (D) none of these

12. The values of x, satisfying the equation for  a > 0


2logx a  logax a  3loga2 x a  0 are
(A) a2, a1 (B) a1/2, a1
(C) a3, a1 (D) a4/3, a1/2

13. The value of x, satisfying the equation



log10 98  x 3  x 2  12x  36  2 is 
(A) 3 (B) 4
(C) 5 (D)  6

14. If 4log9 3  9log2 4  10logx 83 , then x is equal to


(A) 2 (B) 10
(C) 9 (D) none of these

1 1 1
15. If x > 1, y > 1, z > 1 are in G.P, then , , are in
1  ln x 1  ln y 1  ln z
(A) A.P (B) G.P
(C) H.P (D) none of these

16.  
log3 log2 log 3 
81 is equal to

(A) 2 (B) 1
(C) 3 (D) none of these

ln x ln y ln z
17. For   , xyz is equal to
bc c a ab
(A) 1 (B) abc
(C) 0 (D) none of these

18. The value of logba logcb logac is equal to


(A) 0 (B) 1
(C) logaabc (D) 10

log  x  1
19. The value of x in  2 is
logx
5 1
(A) 1 (B)
2
5 1
(C) (D) 2
2

P-2022-CBSE-P1-MATHEMATICS-LOGARITHMS
31
2
20. If 2log(x + 1) – log(x – 1) = log2, then x =
(A) 1 (B) 0
(C) 2 (D) 3

21. The value of x in log101250 + log1080 = x is


(A) 3 (B) 4
(C) 5 (D) 1

Level – II

Multi-choice (Single Correct Answers)

 
1. The value of log7 log7 7 7 7  is
 
(A) 3 log2 7 (B) 1  3 log7 2
(C) 1  3 log2 7 (D) 3 log7 2
log x
2. The number of roots of the equation 3 x log5 2  2 5  64 is
(A) 2 (B) 4
(C) 1 (D) 0

2x 1
3. The point on the graph y  log 2 log6 {2  4} whose y coordinate is 1, is
(A) (12, 1) (B) (6, 1)
(C) (8, 1) (D) (1, 1)

4. The number of solution(s) for the equation 2 logx a  logax a  3 loga2 x a  0 is (are)
(A) one (B) two
(C) three (D) four

5.  
The number of values of x satisfying 1  log5 x 2  1  log5 x 2  4x  1 is 
(A) 1 (B) 2
(C) 3 (D) infinitely many

6. The number of values of x  [0, n ], n  Z, that satisfy the equation


log sin x 1  cos x   2, is
(A) 0 (B) n
(C) 2n (D) none of these

7. If x1, x 2 , x3 ,...... are positive numbers in G.P., then log xn , log xn1, log xn 2 are in
(A) A.P. (B) G.P.
(C) H.P. (D) none of these

x
8. If log2 x  log2  4  0, then x is equal to
16
(A) 4 (B) –4
1
(C) (D) 2
4

P-2022-CBSE-P1-MATHEMATICS-LOGARITHMS
32
9. The sum of series
x  x   x   x   x 
logcos    log  cos 2   log  cos 3   .......  log  cos n   log  sin n  is equal to
2  2   2   2   2 
 1  1 
(A)  n  sin x (B) log  n sin x 
2  2 
1
(C) n cos x (D) none of these
2

10. Which is the correct order for a given number > 1 in increasing order?
(A) log2 , log3 , loge , log10  (B) log10 , log3 , loge , log2 
(C) log10 , loge , log2 , log3  (D) log3 , loge , log2 , log10 

 1
11. The sum of the series 
r 0
log  e 
3
2r
is equal to

(A) loge 2 (B) loge 4


(C) log2 e (D) loge 9

loga n  logb n
12. If a > 0, c > 0, b = ac , a  1, c  1, ac  1 and n > 0, then the value of is
logb n  logc n
equal to
loga n logn a
(A) (B)
logc n logn c
(C) logca (D) none of these

13. Values of x satisfying the equation


  
log 2x 3  6x 2  23x  21  4  log 3x 7  4x2  12x  9 are 
1 1
(A) 1,  (B) 2, 
3 4
1 1
(C) 1,  (D) 2, 
4 3

14. For x > 1, the minimum value of 2 log2x  logx(0.01) is


(A) 2 (B) 4
(C) 1/2 (D) 1/4

15. If log 1/2(x -1) > 2, then x lies is the interval


(A) (3/2,) (B)(-, 3/2)
(C) (1, 3/2) (D) none

Multi-choice (More than one correct answer)

16. 
If S  x : logx 3x , where log3 x  1 , then 
(A) S is a finite set (B) S  
1 
(C) S   0,   (D) S properly contains  ,  
3 

P-2022-CBSE-P1-MATHEMATICS-LOGARITHMS
33

17. If
 2
log2 4x  x  1   1 , then x may be

log2 x  1 2

 2
(A)  ,   (B) 1,  
 3
 2 
(C)   , 0  (D) none of these
 3 

18. If | a || b |, b  a  1 and a, b are the real roots of the equation x 2  |  | x  |  | 0 , then
x
the equation log|b |  1  0 has
a
(A) one root lying in the interval (, a) (B) one root lying in the interval (b, )
(C) one negative root (D) one positive root

1
19. If  log0.1 x  2 , then
2
1 1 1
(A) the maximum value of x is (B) x lies between and
10 100
10
1 1 1
(C) x does not lie between and (D) the minimum value of x is
100 10 100

log x 2  2 logx 9 7
20. If x  1 3   x  1 , then
(A) x = 81 (B) x = 2
1
(C) two real solutions of ‘x’ are possible (D) x 
3
21. The inequation  logx 2  log2x 2  log2 4x   1
(A) has a meaning for all x (B) has a meaning if x> 2
 1
(C) is satisfied in  2 2 , 
 2 
(D) is satisfied in 1, 2 2  
22. Let a 
log100 10  log2 log4 2   log log  256  . Then the value of a is
4 2
2

log4 8  log8 4
1
(A) greater than  (B) 2
2
6 2
(C)  (D)
13 3

23. If x is a positive number different from 1 such that loga x, logb x and logc x are in A.P.,
then
2  loga  logc  ac
(A) logb  (B) b 
loga  logc 2
loga b
(C) b  a.c (D) c 2   a.c 

24. If (logx 2) (logx/16 2) = log x / 64  2


(A) 2 (B) 4
(C) 6 (D) 8

P-2022-CBSE-P1-MATHEMATICS-LOGARITHMS
34
25. The number log27 is
(A) an integer (B) real number
(C) an irrational number (D) a rational number

Reason Assertion

26. Statement-1: The equation log 3 5  4 log 3 x  1 = 2 has only one solution.
log x log 2
Statement-2: 2 3  x 3 ; x  0 .
(A) Statement -1 is True, Statement -2 is True; Statement-2 is a correct explanation for
Statement-1
(B) Statement -1 is True, Statement -2 is True; Statement-2 is NOT a correct explanation
for Statement-1
(C) Statement -1 is True, Statement -2 is False
(D) Statement -1 is False, Statement -2 is True

COMPREHENSION
Read the following write up carefully and answer the following questions:
If f(x) > 0, then {f(x)} 2n = g(x) is equivalent to 2n logaf(x) = logag(x), a > 0 and a  1

27. The no of solutions of log(2x) = 2log (4x – 15) is


(A) 1 (B) 2
(C) 3 (D) infinite

1
28. Solution set of logx = log (x + 1) is
2
1  1 
(A) 
2

2 1 

 (B) 
2
 
5 1 

1 1 1 
(C)  , 
2 3 
(D) 
2
 5 7 


29. Solution set of equation log(8 – 10x – 12x2) = 3log (2x – 1) is
(A) {1} (B) {3, 2}
(C) {5} (D) 

Read the following write up carefully and answer the following questions:

f  x  is said to be an increasing function in [a, b] if f  x  increases as x increases i.e.


    f     f   , where ,   [a, b] . While for a decreasing function in [a, b]
    f     f    , where ,   [a, b] . For a logarithmic function f  x   logz x , increasing and
decreasing behaviour entirely depends on the value of z. If 0  z  1, then f  x  is decreasing in
its domain of definition, on the other hand if z > 1, then f  x  is increasing in its domain of
definition.
5x  x 2
30. If log 0.5  0 , then x belongs to
4
(A) [–4, 0] (B) [1, 4]
(C) (4, 5) (D) [4, 5)

P-2022-CBSE-P1-MATHEMATICS-LOGARITHMS
35
31. The number of solutions of equation loga x  logb x  for some a  1 and 0  b  1 is/ are
(A) 0 (B) 2
(C) 1 (D) 3

 
32. The set of values of x  0  x   satisfying the function f x   log 0.9 cos x  sin x  is
 2
    
(A) 0,  (B)  , 
 4 4 2 
  
(C)  ,  (D) none of these
2 

Matrix Match

33. Solution set of


List  I List  II
(P) 3 log3  x 2
= log6(216) (1) {5}
log8  4x  (2) 
(Q) 2 = log5 (625)
(R) logx 3 + log3 x = 1 – ¾ (3) { 16}
1 (4) {25}
(S) log5x + log25x = 2
2
Codes :
P Q R S
(A) 1 2 3 4
(B) 4 3 1 2
(C) 3 1 4 2
(D) 1 3 2 4

34. Match the following


List–I (Function) List–II (Range)
(P) f(x) = log3(5 + 4x – x2) (1) Function not defined
(Q) f(x) = log3(x2 – 4x – 5) (2) [0, )
(R) f(x) = log3 (x2 – 4x + 5) (3) (– , 2]
(S) f(x) = log3 (4x – 5 – x2) (4) R
Codes :
P Q R S
(A) 3 2 1 4
(B) 4 3 2 1
(C) 3 4 2 1
(D) 4 3 1 2

35. Match the following :


Column I Column II
3 5
log3 x 2 log3 x  (p) number of solutions = 2
(A) x4 4  3
(B) log 3  sin x  2 
2 cos x  2,  2  x  2 (q) number of solutions = 3
x2
(C) log0.2 1 (r) number of solutions = 1
x
(D) log2  x  5   6  x (s) infinite number of solutions

P-2022-CBSE-P1-MATHEMATICS-LOGARITHMS
36
36. Match the following :
Column I Column II
(A) f  x   log10 x 2 8 
(p) f  x   0 for  , 0    ,  
5 
log  x 1 (q) Domain f  x  is x  1 or x  1
(B) f  x  2 2
x5
(C) f  x   2  log2 4  5x (r) f  x  >0,  x   4,  
(D)  2
f  x   log 1 x  6x  12  (s) Domain of f  x  is R
2

(t) f  x  > 0,  x   8,  

37. Matching Type(3 Column & 4 Rows)


Column – 1 Column – 2 Column – 3
(I) logb c log4 3  log4 32 (i) ab = 4 (P) ac = 12
If a  33 3 ...
4
(II) If log6 a  log6 b  log6 c  6 , a, b, c,  N (ii) b = 36 (Q) ac = 6
and a, b, c are in G.P.
(III) if a = 2 log2 3  3 log3 2 , (iii) ab = 1 (R) ac = –6
   
log2 3 log2 3
b  1   2 3 ,
 
c  log2 log2  2 
 
 
(IV) If a = log2 log3 log2(512), b = log10 5  log5 3 (iv) ab = 2 (S) ac = 20
log2 5log5 2
 log3 100 and c = 10 

(I). Which of the following is correct combination


(A) (I) (ii) (Q) (B) (I) (i) (P)
(C) (I) (iii) (R) (D) (I) (iv) (S)

(II). Which of the following is correct combination


(A) (II) (ii) (Q) (B) (II) (i) (P)
(C) (II) (iii) (R) (D) (II) (iv) (S)

(III). Which of the following is correct combination


(A) (III) (i) (P) (B) (III) (ii) (Q)
(C) (III) (iii) (R) (D) (III) (iv) (S)

Numerical Based

38. The value of log9 27  log27 9 is k/6, then k is equal to _________

39. The value of log325832 is ____

40. If log10x + log10y  2 and the smallest possible value of x + y is 4k, then the value of k is

41. If x  logb a, y  logc b, z  loga c , then xyz is


.21
..
9998
42. The value of log2 .log3 ....log100 100 is

P-2022-CBSE-P1-MATHEMATICS-LOGARITHMS
37
Decimal Type

43. Solve the following equation:


2 2
log(2x+3) (6x + 23x + 21) = 4 − log3x+7(4x + 12x + 9).

44.  
The number of ‘n’ such that logn 32013 is an integer is, n  N .

45. If x   log10 k , where A is set of all positive divisors of 108 except 108 itself, then the
kA

value of  x  is, where [.] is greatest integer less than or equal to x

P-2022-CBSE-P1-MATHEMATICS-LOGARITHMS
38
ANSWERS TO CHAPTER PRACTICE PROBLEMS

Subjective:

Level – I
1. 301

3
2.
2

3. –log(1 – x)

Level – II

4. 4logam

log  a  b 
5.
log  a  b 

Objective:

MCQ Single Correct

1. A 2. B 3. D 4. C
5. A 6. C 7. B 8. A
9. D 10. B

MCQ Multi Correct

1. B, C 2. A, B, C, D 3. A, B, C

Numerical Based
1. 2 2. 2

P-2022-CBSE-P1-MATHEMATICS-LOGARITHMS
39
ANSWERS TO ASSIGNMENT PROBLEMS

Subjective:

Level – 0

1. b = 32
5 7
2. (A) 2 (B) (C)
6 2
4 3
3. r
3
5. (A) log R = log P + log V – log T (B) log F = log m1 + log m2 – 2 log d
(C) log T = log 2 + log  + 1/2 [log l – log g] (D) log V = log  + 2 log r + log h – log 3

1 1
6. If a2 + b2 = 7ab, prove that log  a  b   loga  logb .
3 2
7. 1 8. 2
9. 4 10. 1
11. log 2 12. 1
2
13. x = 10 14. x=
3
x
17. 19. 1
10
20. 32 21. 5
4log3
22. 23. 1
log7  log6
24. 2 25. 4

Level – I

3. 16 4. 99, -9/10
5. x = 2, 81 7. x = 20
9. x=3 10. 5
11. 1 12. 9

Level – II

1. x (1 + 3,3)  (-1, 1-3) 2. 4.


3. x=1 4. x  (2, 9)
6. x  (-, 2)(6, ) 7. x=8
 3   3 
8. x = 105, y = 10 9. x  (3, )   ,    , 5
 2   2 
1 -1/3
10. x = 2, ,2
4

P-2022-CBSE-P1-MATHEMATICS-LOGARITHMS
40

Objective:

Level – I

1. A 2. A 3. B 4. B
5. C 6. A 7. A 8. D
9. B 10. A 11. C 12. D
13. B 14. B 15. C 16. B
17. A 18. B 19. C 20. D
21. C
Level – II

1. B 2. C 3. A 4. B
5. D 6. A 7. A 8. A
9. B 10. B 11. D 12. A
13. B 14. B 15. C 16. C
17. A, B 18. C, D 19. A, B, D 20. A, B, C
21. C, D 22. A, C 23. A, D 24. B, D
25. B, C 26. B 27. A 28. B
29. D 30. B 31. C 32. A
33. D 34. C
35. (A)  (q) (B)  (p) (C)  (s) (D)  (r)
36. (A)  (q) (B)  (r) (C)  (p) (D)  (p), (s)
37. (I). B (II). A (III). C
38. 5 39. 6 40. 5 41. 1
42. 1 43. – 0.25 44. 8.00 45. 316.00

P-2022-CBSE-P1-MATHEMATICS-LOGARITHMS
PINNACLE-CBSE

QUADRATIC EQUATION & EXPRESSIONS


CONTENTS SYLLABUS
 Quadratic equation 1

 Exercise 1 4 CBSE: Statement of Fundamental theorem of


 Condition for two quadratic algebra, solution of quadratic equations in the
equations to have a common
root 4
complex number system.
 Exercise 2 5

 Relation between the roots 5 JEE: Quadratic equations with real


of polynomial equations
coefficients, relations between roots and
 Exercise 3 7
coefficients, formation of quadratic equation
 Method of intervals 8

 Exercise 4 10 with given roots, symmetric function of

 Quadratic Expression 10 roots.

 Exercise 5 12

 Intervals in which roots lie 12

 Exercise 6 16

 Miscellaneous Exercise 17

 Answers to Exercises 18

 Solved Problems 19

 Chapter Practice Problem 38

 Assignment Problems 41
 Answers to Chapter 53
Practice Problems

 Answers to Assignment 54
problems
QUADRATIC EQUATION
2
An equation of the form ax + bx + c = 0, where a  0, is called a quadratic equation. The
numbers a, b, c are called the coefficients of the quadratic equation. A root of the quadratic
equation is a number  (real or complex) such that a2 + b + c = 0.
b  b2  4ac
The roots of the given quadratic equation are given by x = .
2a
Proof: Dividing the given equation by a, we get
b c
x2 + x   0
a a
2 2
2  b   b  c  b 
or x + 2   x         0
 2a   2a  a  2a 
2
 b  b2  4ac
or  x   
 2a  4a2
b  b2  4ac b  b2  4ac
x+  x= .
2a 2a 2a

Basic Results:
The quantity D (D = b2 – 4ac) is known as the discriminant of the quadratic equation. For a, b, c
real,
 The quadratic equation has real and equal roots if and only if D = 0 i.e. b2 – 4ac = 0.
 The quadratic equation has real and distinct roots if and only if D > 0 i.e. b2 – 4ac > 0.
 The quadratic equation has complex roots with non–zero imaginary parts if and only if
D < 0 i.e. b2 – 4ac < 0.
 If p + iq (p and q being real) is a root of the quadratic equation where i = 1 , then
p – iq is also a root of the quadratic equation.
 If p + q is an irrational root of the quadratic equation, then p – q is also a root of the
quadratic equation provided that all the coefficients are rational.
 The quadratic equation has rational roots if D is a perfect square and a, b, c are
rational. If a = 1 and b, c are integers and the roots of the quadratic equation are
rational, then the roots must be integers.

Proof: Let b be even i.e. b = 2m. Then D = 4m2  4c is even if D is a perfect square
b  D
 ( 2m  D) is even  is an integer.
2
If b is odd i.e. b = 2m + 1 then D = 4m2  4  1  4c is odd if D is a perfect square
b  D
 ((2m + 1)  D) is even  is an integer.
2
 If the quadratic equation is satisfied by more than two numbers (real or complex), then it
becomes an identity i.e. a = b = c = 0.

P-2022-CBSE-P1-MATHEMATICS-QEE
2
2
Proof: Let , ,  be three different roots of the quadratic equation ax + bx + c = 0. Hence
a2 + b + c = 0, … (1)
2
a + b + c = 0, … (2)
a2 + b + c = 0. … (3)
Subtract (2) and (3) from (1) and get
a (2  2) + b (  ) = 0 and a (2  2) + b (  ) = 0. … (4)
Since     , we get
a ( + ) + b = 0 and a ( + ) + b = 0. … (5)
Again subtracting one equation of (5) from the other, we have
a (  ) = 0  a = 0, as   
and hence from (5), b = 0, which in turn gives that c = 0
from (1), (2) or (3).

Factorization Method:
In the factorization method we write the coefficient of x as a sum of two numbers l and m such
that lm = ac i.e. we write
b = l + m with lm = ac
so that the equation ax2 + bx + c = 0 becomes
lm
ax2 + (l + m) x + 2 2
 0 or a x + a (l + m) x + lm = 0
a
or (ax + l) (ax + m) = 0  ax + l = 0 or ax + m = 0.
l m
Hence the roots are x = and x = . In order to obtain l and m, we write
a a
lm= (l  m)2  4lm  b2  4ac .
With l + m = b, we get
b  b2  4ac b  b2  4ac
l= and m =
2 2
b  b2  4ac
so that the two roots are .
2a
b
 Let  and  be two roots of the given quadratic equation. Then  +  = – and
a
c
 = .
a
A quadratic equation, whose roots are  and  can be written as (x – ) (x – ) = 0
i.e., ax2 + bx + c  a(x – ) (x – ).

Illustration 1. Solve the following quadratic equations


(i) x2  5x + 6 = 0,
(ii) x2 + x + 1 = 0,
2 2 2
(iii) pqx + (p  q )x  pq = 0.

Solution: (i) We have a = 1, b =  5, c = 6, so that


D = b2  4ac = 25  24 > 0.
Hence we obtain two real and distinct roots given by

P-2022-CBSE-P1-MATHEMATICS-QEE
3
b  D 5  1 b  D 5  1
=   3,     2.
2a 2 2a 2
(ii) We have a = 1, b = 1, c = 1, so that
D = b2  4ac = 1  4 =  3 < 0.
Hence we obtain the pair of complex roots as
b  i D 1  i 3 1  i D 1  i 3
=  and  =  .
2a 2 2a 2
2 2
(iii) We have a = pq, b = p  q , c =  pr so that
D = b  4ac = (p  q )  4pq ( pq) = (p2  q2)2 + 4p2q2 = (p2 + q2)2 > 0.
2 2 2 2

Hence, we have two real roots


b  D (p2  q2 )  p2  q2 2q2 q
=   
2a 2pr 2pq p
b  D (p2  q2 )  (p2  q2 ) 2p2 p
and  =    .
2a 2pq 2pq q

Illustration 2. Prove that the roots of the quadratic equation ax2 – 3bx – 4a = 0 are real and
distinct for all real a and b.

Solution: D = (–3b)2 – 4(–4a) (a) = 9b2 + 16a2 which is always positive.


Hence the roots will be real and distinct.

Illustration 3. Find the value of m for which the equation


(1 + m)x2 –2(1 + 3m)x + (1 + 8m) = 0 has equal roots.

Solution: Given equation is (1 + m)x2 –2(1 + 3m)x + (1 + 8m) = 0.


If the roots are equal, then discriminant = 4(1 + 3m)2 –4(1 + m) (1 + 8m) = 0
 m2 –3m = 0 or m = 0, 3.

Problems based on value of symmetrical functions of  and , where  and  are roots of a
quadratic equation.

Illustration 4. If  and  be roots of the equation ax2 + bx + c = 0,


2  2    
then prove that a     b    = b.
      

b c
Solution: Here  +  = – and  = .
a a
  2 2 
Now, a 
  
a  3  3  b  2   2  
 
   b   =
     
3 2
a       3        b       2 
   
=

 b  3 c  b    b 2 c
a     3      b      2  bc
 a  a  a    a  a 
= = a b.
c/a c/a

P-2022-CBSE-P1-MATHEMATICS- QEE
4
Exercise 1.

i) Show that if the roots of the equation (a2 + b2)x2 + 2x (ac + bd) + c2 + d2 = 0 are real,
they will be equal.
2 2 2 2 2
ii) Show that the roots of the equation (a + b) x – 2(a – b )x + (a – b) = 0 are equal.
iii) Prove that the roots of the equation bx2 + (b –c)x + (b –c –a) = 0 are real if those of
ax2 + 2bx + b = 0 are imaginary and vice versa.
2
iv) If ,  are the roots of the equation x – px + q = 0 and 1, 1 the roots of equation
1 1 1 1
x2–qx+p=0, form the quadratic equation whose roots are  and  .
1  1 1 1
v) Find the sum of real roots of the equation x2 + |x|  6 = 0.

Condition for two Quadratic Equations to have a Common Root


Let ax2 + bx + c = 0 and dx2 + ex + f = 0 have a common root (say). Then a2 + b + c = 0 and
d2 + e + f = 0.
2  1
Solving for 2 and , we get  
bf  ce dc  af ae  bd
2 bf  ce dc  af
i.e.  = and  =  (dc – af)2 = (bf – ce) (ae – bd),
ae  bd ae  bd
which is the required condition for the two equations to have a common root.

Illustration 5. If ,  are the roots of x2 + px + q = 0, and ,  are the roots of x2 + rx + s = 0,


evaluate ( – )( – )( – )( – ) in terms of p, q, r and s. Deduce the condition
that the equations have a common root.

Solution: We have  +  = –p,  = q,  +  = – r,  = s.


Hence ( – )( – )( – )( – ) = [2 – ( + ) + ] [2 – ( + ) + ]
= [2 + r + s] [2+ r + s] = 22 + r( + ) + r2 + s[2+ 2]+ s2+ sr( + )
= q2 – rpq + r2q + s(p2 – 2q) + s2 – srp = (q – s)2 + (p – r)(sp – rq).
For a common root, R.H.S. = 0  (q – s)2 = (p – r)(rq – sp), which is the required
condition.

Illustration 6. Find the value of p if the equations 3x2 – 2x + p = 0 and 6x2 – 17x + 12 = 0 have
a common root.
2
Solution: Let  be the common root. Then 3 –2 + p = 0, ....(1)
and 62 –17 + 12 = 0 ....(2)
2
  1 1 24  17p
     2 =
24  17p 6p  36 51  12 39 39
2 2
36  6p 24  17p  36  6p   12  2p 
and  =  =    
39 39  39   13 
144  48p  4p2
=  312 –221p = 432 –144p + 12p2
169
2 15 8
 12p + 77p + 120 = 0  (4p + 15) (3p + 8) = 0  p = – ,  .
4 3

P-2022-CBSE-P1-MATHEMATICS-QEE
5
3 2 2
Illustration 7. If x  3x  3 x  2  0 and ax  bx  c  0 , a, b, c  R have two common roots
then the roots of the equation ax2 + (a + b)x + c = 0 are
(A) imaginary (B) real and unequal
(C) real and equal (D) None of these

Solution:  
x 3  3x 2  3x  2   x  2  x 2  x  1
2
the common roots are , 
 a = b = c = 1.
Now, ax2 + (a + b)x + c = 0
2
 x + 2x + 1 = 0
2
(x + 1) = 0
 roots are real and equal.

Illustration 8. The value of ‘a’ for which the equation x3 + ax + 1 = 0 and x4 + ax + 1 = 0 have a
common root, is
(A) 2 (B) –2
(C) 3 (D) –3

Solution: Let  be a common root.


3 + a + 1 = 0
4 + a + 1 = 0
 4 – 3 = 0  3( – 1) = 0.
Since  can not be zero.
=1
 1 + a + 1 = 0  a = –2.

Exercise 2.
i) If two equations x2 + cx + ab = 0 and x2 + bx + ca = 0 have a common root, show
that a + b + c = 0.
ii) Prove that equations (q –r)x2 + (r –p)x + p –q = 0 and (r –p)x2+ (p –q)x + q –r = 0
have a common root.
iii) If the equations x2  x  p = 0 and x2 + 2px  12 = 0 have a common root, find it.
iv) If the equations x2 + px + q = 0 and x2 + px + q = 0, (p  p, q  q) have a common
pq   p q q  q
root, prove that it is equal to or .
qq  p  p
v) If equations ax2 + bx + c = 0 and cx2 + bx + a = 0 have a negative common root,
then find the value of (a  b + c).

Relation between the Roots of a Polynomial Equation of Degree n


Consider the equation
n n–1 n–2
anx + an – 1x + an – 2x + …. + a1x + a0 = 0 . . . . (1)
(a0, a1…., an are real coefficients and an  0).
Let 1, 2,….,n be the roots of equation (1). Then
n n–1 n–2
anx + an – 1x + an – 2x + ….. + a1x + a0  an(x – 1) (x – 2) ….. (x – n).
Comparing the coefficients of like powers of x, we get
a
1 + 2 + 3 + …. + n = – n1 ,
an

P-2022-CBSE-P1-MATHEMATICS- QEE
6
an2
12 + 13 + 14 + …. + 23 + … + n – 1n = ,
an
………………………………
r anr
12 . . . . . r + …. + n–r+1n–r+2 … n = ( –1) ,
an
…….…………………………
a0
12 … n = (–1)n a .
n

4 3 2
e.g. If , ,  and  are the roots of ax + bx + cx + dx + e = 0 then
 +  +  +  = –b/a,
 + +  +  +  +  = c/a,
 +  +  +  = –d/a,
 = e/a.

Notes:
 A polynomial equation of degree n has n roots (real or imaginary).
 If all the coefficients are real then the imaginary roots occur in pairs i.e. number of
complex roots is always even.
 If the degree of a polynomial equation is odd then the number of roots will also be odd.
It follows that at least one of the roots will be real.
 If  is a repeated root (repeating r times) of a polynomial equation f(x) = 0 of degree n
i.e. f(x) = (x – )r g(x) , where g(x) is a polynomial of degree n – r and g()  0, then
f() = f() = f() = ... = f (r–1)() = 0 and f r ()  0 .
 Remainder Theorem: If we divide a polynomial p(x) by x – , the remainder obtained is
p(). Note that if p() = 0, then x –  is a factor of p(x).
 If a polynomial equation of degree n has n + 1 roots say x1,...xn + 1, (xi  xj if i  j), then
the polynomial is identically zero. ie. p(x) = 0, x  R.
(In other words, the coefficients a0, .... an are all zero).
 If p(a) and p(b) (a < b) are of opposite sign, then p(x) = 0 has odd number of roots in (a,
b), i.e. it have at least one root in (a, b).
 If coefficients in p(x) have `m’ changes in signs, then p(x) = 0 have at most `m’ positive
real roots and if p(–x) have `t’ changes in sign, then p(x) = 0 have at most `t’ negative
real roots. By this we can find maximum number of real roots and minimum number of
complex roots of a polynomial equations p(x) = 0.

Illustration 9. Let a and b be two roots of the equation x3 + px2 + qx + r=0 satisfying the relation
ab + 1=0. Prove that r2 + pr + q + 1=0. (r  0)

Solution: Given equation is x3 + px2 + qx + r = 0.


Let the third root be c, so that abc = – r. ...(1)
Given ab = –1
 from (1), c = r, which is a root of the given equation.
 (r)3 + p(r)2 + q(r) + r = 0  r2 + pr + q + 1 = 0.

P-2022-CBSE-P1-MATHEMATICS-QEE
7
2 3 2
Illustration 10. If b < 2ac, then prove that ax + bx + cx + d = 0 has exactly one real root.

Solution: Let , ,  be the roots of ax3 + bx2 + cx + d = 0.


b c d
Then  +  +  =  ,  +  +  = ,  =
a a a
2 2 2 2 b2 2c b2  2ac
  + + = ( +  + ) – 2( +  + ) =  
a2 a a2
 2 +2 + 2 < 0 , which is not possible if all , ,  are real
 atleast one root is non–real. But complex roots occur in pairs. Hence the given
cubic equation has two non–real and one real roots.
Alternative Solution:
Let f(x) = ax3 + bx2 + cx + d and f(x) = 0 have all the roots real
 f (x) = 3ax2 +2bx +c = 0 has two real roots.
But its discriminant
= (2b)2 – 4.3.ac = 4 (b2 – 2ac) – 4ac < 0 (as b2 < 2ac, both a and c are of the
same sign)
which is a contradiction  f(x) =0 will not have all the roots real.

Illustration 11. Total number of roots of (x2 + x + 1)2 + 2 = (x2 + x + 1) (x2 – 2x – 6), belonging to
(–2, 4) is
(A) 2 (B) 3
(C) 1 (D) none of these

Solution: Given equation is (x2 + x + 1)(3x + 7) + 2 = 0.

Illustration 12. If (x2 –5x + 4) (y2 + y + 1) < 2y for all y  R, then


(A) x  (3, 4) (B) x  (2, 4)
(C) x  (2, 3) (D) none of these

2y
Solution: (x2 –5x + 4) (y2 + y + 1) < 2y  y  R  x2 –5x + 4 < 2
y  y 1
2
Now least value of RHS is equal to –2  x –5x + 4 < –2  x  (2, 3).

Exercise 3.
2 1 1
i) Find the roots , ,  of x3 –11x2 + 36x –36 = 0 if   .
  
ii) If the roots of x3 + px2 + qx + r = 0 are in G.P., find the relation between p, q, r.
iii) If 2 and 3 are the roots of the equation 2x3 + mx2  13x + n = 0, find the third root.
iv) Solve the equation x3  11x2 + 38x  40 = 0, given that the ratio of two of its roots is
2 : 1.
v) If ,  and  are the roots of the equation x(1 + x2) + x2(6 + x) + 2 = 0, then find the
value of (1 + 1 + 1).

P-2022-CBSE-P1-MATHEMATICS- QEE
8
The Method of Intervals (Wavy Curve Method)
The Method of intervals (or wavy curve method) is used for solving inequalities of the form
n1 n2 nk
 x  a1   x  a2  ....  x  ak 
f(x) = mp
> 0 ( < 0,  0, or  0) where n1, n2,…, nk , m1, m2, ...mp
 x  b1 m  x  b2 m
1 2
....  x  bp 
are natural numbers and the numbers a1, a2, … , ak ; b1, b2,…bp are any real numbers such that
aI  bj , where i = 1, 2, 3,…, k and j = 1, 2, 3,…, p .
It consists of the following steps:
 All zeros1 of the function f(x) contained on the left hand side of the inequality should be
marked on the number line with inked (black) circles.
2
 All points of discontinuities of the function f(x) contained on the left hand side of the
inequality should be marked on the number line with un–inked (white) circles.
 Check the value of f(x) for any real number greater than the right most marked number
on the number line.
 From right to left, beginning above the number line (in case the value of f(x) is positive
in step (iii), otherwise, from below the number line), a wavy curve should be drawn to
pass through all the marked points so that when it passes through a simple point3, the
curve intersects the number line, and, when passing through a double point4, the curve
remains located on one side of the number line.
 The appropriate intervals are chosen in accordance with the sign of inequality (the
function f(x) is positive whenever the curve is situated above the number line, it is
negative if the curve is found below the number line). Their union represents the
solution of the given inequality.

C1 C2 C3 Cm+n-2 Cm+n-1 Cm+n


......

Remark:
(i) Points of discontinuity will never be included in the answer.
(ii) If you are asked to find the intervals where f(x) is non–negative or non–positive then
make the intervals closed corresponding to the roots of the numerator and let it remain
open corresponding to the roots of the denominator.
1
The point for which f(x) vanishes (becomes zero) are called function zeros e.g. x = ai.
2
The points x = bj are the point of the discontinuity of the function f(x).
3
If the exponent of a factor is odd then the point is called a simple point.
4
If the exponent of a factor is even then the point is called a double point.

 x  3  x  2  x  5 
Illustration 13. Let f(x) = . Find intervals, for which f(x) is positive or
 x  1 x  7 
negative.

Solution:

-5 -2 -1 3 7

P-2022-CBSE-P1-MATHEMATICS-QEE
9
f(x) > 0  x  (–5, –2)  (–1, 3)  (7, )
and f(x) < 0  x  (– ,–5)  (–2, –1)  (3, 7).

2x 1
Illustration 14. Find the set of all x for which 2
 
2x  5 x  2 x 1

2x 1 3x  2
Solution: We have 2
 0  0
2x  5x  2 x 1 (x  1)(2x  1)(x  2)
3x  2 2
 0
(x  1)(2x  1)(x  2)(3x  2)
(3x  2)2
 0. …..(1)
1 2
2.3(x  )(x  )(x  1)(x  2)
2 3
There are five intervals x < –2, –2 < x < –1, –1 < x < – 2/3, – 2/3<x<–1/2, x>–1/2.
The inequality (1) will hold for –2 < x < –1 and for –2/3 < x < –1/2.
Hence –2 < x < –1 and –2/3 < x < –1/2.

x3
Illustration 15. The greatest integer x for which the inequality 2
 0 is satisfied, is
x  9x  22
equal to
(A) – 12 (B) –11
(C) 2 (D) 3

x 3
0
x  3 0
Solution: 2

x  11x  2x  22  x  11  x  2 

Largest integral value of x = –12

x2  x  6
Illustration 16. If  1 , then x belongs to the set
x 2  2 x  3
 3
(A) (2, 3) (B)  1, 
 2
 3
(C) R (D) R-  1, 
 2

Solution: x2 + 2x – 3 < 0,  x, so the given inequality reduces to x2 + x – 6 < x2 + 2x  3


 2x2 – x – 3 < 0  (2x  3) (x + 1) < 0
 3
 x  1, 
 2

P-2022-CBSE-P1-MATHEMATICS- QEE
10
Exercise 4.
x 1
i) Solve the inequality 2
< 1.
x  4x  3
3x2  7 x  8
ii) Solve 1  2.
x2  1
x  2 2x  3
iii) Find x  R, for which  .
x  2 4x  1
x2  3x  2
iv) Let f (x) = . Find the intervals for which f (x) is negative.
x2  1
 x2  4 
v) Let f(x) =  3  , then find the interval for which f(x) is positive.
 x 1 

Quadratic Expression
The expression ax2 + bx + c is said to be a real quadratic expression in x where a, b, c are real
and a  0. Let f(x) = ax2 + bx + c where a, b, c,  R (a  0).
 2 2
b  4ac  b2   b  D  2
f(x) can be rewritten as f(x) = a  x     =a   x     , where D = b –4ac
2a 2 2a 2
  4a     4a 
is the discrimnant of the quadratic expression. Then y = f(x) represents a parabola whose axis is
 b D 
parallel to the y – axis, with vertex at A   ,  .
 2a 4a 
Depending on the sign of a and b2 – 4ac, f(x) may be positive, negative or zero. This gives rise to
the following cases:

(i) a > 0 and b2 – 4ac < 0


 f(x) > 0  x  R.
In this case the parabola always remains
above the x – axis.

x x
2
(ii) a > 0 and b – 4ac = 0
 f(x)  0  x  R.
In this case the parabola touches the
x – axis at one point and remains concave
upwards. x x
(iii) a > 0 and b2 – 4ac > 0.
Let f(x) = 0 have two real roots  and (<).
Then f(x) > 0  x  (–, )(, ),
and f(x) < 0  x (, ).
x   x

x x
2
(iv) a < 0 and b – 4ac < 0
 f(x) < 0  x  R.
In this case the parabola always remains below
the x–axis.

P-2022-CBSE-P1-MATHEMATICS-QEE
11
2
(v) a < 0 and b – 4ac = 0
x x
 f(x)  0  x  R.
In this case the parabola touches the
x – axis and lies below the x – axis.

2
(vi) a < 0 and b – 4ac > 0  
Let f(x) = 0 have two real roots  and x x
 ( < ). Then f(x) < 0  x (–, )(, )
and f(x) > 0  x (, ).

Illustration 17. Let f (x) be a quadratic expression which is positive  x  R.


If g(x) = f (x) + f (x) + f (x), then prove that g (x) > 0  x  R.
2
Solution: Let f (x) = ax + bx + c.
Then g (x) = f (x) + f (x) + f (x)
= ax2 + bx + c + (2ax + b) + 2a = ax2 + (b + 2a)x + c + b + 2a.
Given f (x) = ax2 + bx + c > 0  x  R
 b2 – 4ac < 0 and a > 0. ......(1)
Now discrminant of equation g (x) = 0 is
D = (b + 2a)2 – 4a (c + b + 2a) = b2 + 4a2 + 4ab – 4ac – 4ab – 8a2
= b2 – 4a2 – 4ac = (b2 – 4ac) – 4a2 < 0 and a > 0 from (1)
 g (x) > 0  x  R.

Illustration 18. If f (x) = (a1x + b1)2 + (a2x + b2)2 + ......+ (anx + bn)2,
then prove that (a1b1 + a2b2 + ....+anbn)2  a12  a22  ...  an2   b 2
1 
 b22  ...  bn2 .
2 2 2
Solution: Given f (x) = (a1x + b1) + (a2x + b2) + ......+ (anx + bn) ...(1)
or f(x)=  a12  a22  ...  an2 x 2
 2  a1b1  a2 b2  ...  an bn  x


 b12  b22  ...  bn2 .  …(2)
From (1), f (x)  0  x  R
 from (2)
a 2
1 
 a22  .....  an2 x 2  2  a1b1  a2 b2  .....  an bn  x

 
 b12  b22  .....  bn2  0,  x  R.
Hence its discriminant D  0 [  coefficient of x2 is positive]
2
 4  a1b1  a2 b2  .....  an bn   4 a12  a22  .....  an2   b  b  .....  b 
2
1
2
2
2
n
2
  a1b1  a2 b2  .....  an bn   a12  a22  .....  an2   b  b  .....  b  .
2
1
2
2
2
n

2
Illustration 19. If ax – bx + 5 = 0 does not have 2 distinct real roots, then find the minimum
value of 5a +b.
Solution: Let f(x) = ax2 –bx + 5.
Since f(x) = 0 does not have 2 distinct real roots, we have either
f(x)  0  x  R or f(x)  0  x  R.
But f(0) = 5 > 0  f(x)  0  x  R.

P-2022-CBSE-P1-MATHEMATICS- QEE
12

In particular f(–5)  0
 25a + 5b + 5  0  5a + b  – 1.
Hence the least value of 5a + b is – 1.

2
Illustration 20. If  is a positive integer and the roots of the equation 6x –11x +  = 0 are
rational numbers, then the smallest value of  is
(A) 4 (B) 5
(C) 6 (D) none of these

Solution: Since roots of 6x2 –11x +  = 0 are rational and coefficients are rational
2
 121 –24 = k  k  I
Clearly k is odd, let k = 21 + 1
1( 1  1)
=5– , for minimum value of , 1 = 3  min = 3.
6

Exercise 5.
i) a, b, c are three distinct real numbers and they are in G.P. If a + b + c = xb, then
prove that x < –1 or x > 3.
x 2  bc
ii) Show that if x is real, the expression has no value between b and c.
2x  b  c
x  1 
iii) If x is real, show that 2 always lies in the interval   , 1 .
x  5x  9  11 
c b
iv) If the equation ax2 + bx + c = 0 has non-real roots, prove that 1    0
a a
v) If the roots of the equation x2  2ax + a2 + a  3 = 0 are less than 3 then find the set
of all possible values of a.

Interval in Which the Roots Lie


In some problems we want the roots of the equation ax2 + bx + c = 0 to lie in a given interval. For
this we impose conditions on a, b and c. Let f(x) = ax2 + bx + c.
 If both the roots are positive i.e. they lie in (0, ), then the sum of the roots as well as
the product of the roots must be positive
b c 2
  +  = –  0 and  =  0 with b – 4ac  0.
a a
Similarly, if both the roots are negative i.e. they lie in (– , 0) then the sum of the roots
will be negative and the product of the roots must be positive
b c
i.e.  +  = – < 0 and  =  0 with b2 – 4ac  0.
a a
 Both the roots are greater than a given number k if the following three conditions are
b
satisfied: D 0, –  k and a.f(k) > 0.
2a
 Both the roots will be less than a given number k if the following conditions are satisfied:
b
D  0, – < k and a.f(k) > 0.
2a
 Both the roots will lie in the given interval (k1, k2) if the following conditions are satisfied:
b
D  0 k1 < –  k 2 and a. f(k1) > 0, a.f(k2) > 0.
2a

P-2022-CBSE-P1-MATHEMATICS-QEE
13
 Exactly one of the roots lies in the given interval (k1, k2) if f(k1) . f(k2) < 0.
 A given number k will lie between the roots if a.f(k) < 0.
In particular, the roots of the equation will be of opposite signs if 0 lies between the roots
 a.f(0) < 0. It also implies that the product of the roots is negative.

Illustration 21. Find the values of the parameter a for which the roots of the quadratic equation
2
x + 2(a – 1)x + a + 5 = 0 are
(i) real and distinct, (ii) equal,
(iii) not real, (iv) opposite in sign,
(v) equal in magnitude but opposite in sign,
(vi) positive, (vii) negative,
(viii) such that one root is greater than 3, and the other is smaller than 3,
(ix) greater than 3, (x) smaller than 3,
(xi) such that exactly one root lies in the interval (1, 3),
(xii) such that both the roots lie in the interval (1, 3),
(xiii) such that one root is greater than 3 and the other root is smaller than 1.

Solution: Let f(x) = x2 + 2(a – 1)x + a + 5 = Ax2 + Bx + C (say)


 A = 1, B = 2(a – 1), C = a + 5.
Also D = B2 – 4AC= 4(a – 1)2 – 4(a + 5) = 4(a + 1) (a – 4).
(i) D > 0  (a + 1) (a – 4) > 0
 a  (–, –1)  (4, ).
(ii) D = 0  (a + 1) (a – 4) = 0  a = –1, 4.
(iii) D < 0  (a + 1) (a – 4) < 0  a  (–1, 4).
(iv) This means that 0 lies between the roots of the given equation.
 f(0) < 0 and D > 0 i.e. a  (–, –1)  (4, )
and a + 5 < 0  a < –5
or a  (–, –5).
(v) This means that the sum of the roots is zero
 – 2(a – 1) = 0 and D > 0 i.e. a  (–, –1)  (4, ) and a = 1
which does not belong to (–, –1)(4, )
 a is null set .
(vi) This implies that both the roots are greater than zero
B C
 – > 0, > 0, D  0
A A
 –(a – 1) > 0, a + 5 > 0, a  (–, –1]  [4, )
 a < 1, –5 < a, a  (–, –1]  [4, )  a  (–5, –1].
(vii) This implies that both the roots are less than zero
B C
 – < 0, > 0, D  0
A A
 –(a – 1) < 0, a + 5 > 0, a  (–, –1]  [4, )
 a > 1, a > –5, a  (–, –1]  [4, )  a  [4, ).
(viii) This means that 3 lies between the roots
 f(3) < 0 and D > 0
 9 + 2(a – 1)(3) + a + 5 < 0 and a  (–, –1)  (4, )

P-2022-CBSE-P1-MATHEMATICS- QEE
14
 a < –8/7 and a  (–, –1) (4, )
 8
 a   ,   .
 7
(ix) In this case
B
–  3 , A.f (3) > 0 and D  0
2A
 –(a – 1) > 3, 7a + 8 > 0 and a  (–, –1]  [4, )
 a < –2, a > –8/7 and a  (–, –1]  [4, ).
Since no value of a can satisfy these conditions simultaneously, there can be no
value of a for which both the roots will be greater than 3.
(x) In this case
B
– < 3, A.f(3) > 0 and D  0
2A
8  8 


 a > –2, a > – and a  (–, –1]  [4, )  a    , 1  4,   .
7 7 
(xi) In this case f(1).f(3) < 0 and D  0
 4 8
 (3a + 4)(7a + 8) < 0 and a  (–, –1]  [4, )  a    ,   .
 3 7
B
(xii) In this case 1 < – < 3, A.f(1) > 0, A.f(3) > 0, D  0
2A
 1 < –1(a –1) < 3, 3a + 4 > 0, 7a + 8 > 0, a  (–, –1]  [4, )
 –2 < a < 0, a > –4/3, a > –8/7, a  (–, –1]  [4, )
 8 
 a    , 1 .
 7 
(xiii) In this case D > 0 and f(1) < 0 i.e. 3a + 4 < 0
 a < –4/3
and f(3) < 0 i.e. 7a + 8 < 0
 a < –8/7 and a  (–, –1)  (4, ).
Combining we get, a  (–, –4/3).

Illustration 22. If  is a root of the equation ax2 + bx + c = 0 and  is a root of the equation
–ax2 + bx + c = 0, then prove that there will be a root of the equation
a 2
x  bx  c  0 lying between  and .
2

a 2
Solution: Let f(x) = x  bx  c
2
a a
 f() = 2 + b + c = a2 + b + c – 2
2 2
a 2
=–  (As  is a root of ax2 + bx + c = 0).
2
a 2 2 3 2
Also f() =  + b+ c = – a + b + c + a
2 2
3
= a2 (As  is a root of – ax2 + bx +c = 0).
2

P-2022-CBSE-P1-MATHEMATICS-QEE
15
3 2 2 2
Now f(). f() = a   < 0  f(x) = 0
4
has one real root between  and .

9
Illustration 23. Find the values of ‘a’ for which 4t – (a – 4) 2t + a < 0  t  (1, 2).
4

t 2 9
Solution: Let 2 =x and f(x) = x – (a – 4) x + a.
4
We want f(x) < 0  x  (21, 22) i.e.  x  (2, 4).
(i) Since coefficient of x2 in f(x) is positive, f(x) < 0 for some x only when roots of
f(x) = 0 are real and distinct
 D > 0  a2 –17a +16 > 0  a < 1 or a > 16. ... (1)
(ii) Since we want f(x) < 0  x  (2, 4), one of the roots of f(x) = 0 should be less
than 2 and the other must be greater than 4 i.e. f(2)  0 and f(4)  0.
a  –48 and a  128 /7, which is not possible.
Hence no such ‘a’ exists.

Illustration 24. Find all the values of the parameter a for which the inequality
4x –a2x – a +3  0 is satisfied by at least one real x.

Solution: 4x – a2X + (3 – a)  0. Let 2x = t > 0 so that t2 – at + (3 – a)  0.


Let f(t) = t2 – at + (3 – a).
Note that graph of the function f is an upward parabola. Now we want some
positive values of t for which f(t)  0. This implies that roots of the equation
should be real and atleast one of them should be positive.
Condition for both the roots to be real is that a2 – 4 (3 – a)  0
 a2 + 4a – 12  0  (a + 6) (a – 2)  0  a  (–, –6]  [2, + ).
Condition for both the roots to be negative is that  +  < 0 and    > 0
 a < 0 and 3 – a > 0  a < 0 and a < 3  a < 0.
So the condition for atleast one root to be positive is that a  0.
Hence the required range of a is [2,  ).

Illustration 25. Let S denote the set of all possible values of parameter a for which
x+ x2  a  a posses solution, then find set S.

Solution: x+ x2  a  a …(1)
2 2 2
 x + a = x  2ax + a
 2x = a  1
Putting it in (1), we get (a  1) + |a + 1| = 2a  a   1
 S = [1, ).

P-2022-CBSE-P1-MATHEMATICS- QEE
16

Exercise 6.
i) For what values of a  R, the quadratic equation
(a2 + 1)x2 –(a + 1)x + (a2 –a –2) = 0 will have roots of opposite sign.
2
ii) For what values of a, does the equation ax –(a + 1)x + 3 = 0, have roots lying
between 1 and 2.
x 2  mx  2
iii) For what value of m,  3   2 for all x  R.
x2  x  1
2
iv) If the roots of the equation x + 2(a  3)x + 9 = 0 lies between  6 and 1, find the
integral part of a.
v) Find the least integral value of k for which (k  2)x2 + 8x + k + 4 > 0  x  R.

Cube Root of Unity:


Consider the equation x3 = 1  x3 – 1 = 0. We note that x = 1 is one of the roots of this equation,
so that
0 = x3  1 = (x  1) (x2 + x + 1)
1  i 3
 x = 1 or x2 + x + 1 = 0  x= .
2
1 i 3 1 i 3
Hence the cube roots of unity are: x = 1, x =  + ,x=  .
2 2 2 2
1 3
Alternatively, x = 1, , 2 where  =   i .
2 2

P-2022-CBSE-P1-MATHEMATICS-QEE
17

Miscellaneous Exercise

i) If the constant term in a quadratic equation is zero, prove that one of its roots is
zero.
2
ii) If r is the ratio of the roots of the quadratic equation ax + bx + c = 0, prove that
(1 + r)2 ac = b2r.
x 2  bx m  1
iii) For what value of m ( 1) will the equation  have roots equal in
ax  c m 1
magnitude but opposite in sign.
2 2
iv) If the roots of the equation x  lx + m = 0 differ by 1, prove that l = 4m + 1.
3
v) Prove that the roots of the equation 2x2  x  = 0 are irrational.
2

vi) Prove that 20  20  20  ....  5 .

a x  ax a
vii) Solve for x:  .
ax  ax x
x+1
viii) Solve for x: 7 + 71x = 50.
ix) Show that the equation (x  a) (x  b) = c2 has real roots.
x) Prove that the roots of the equation (x  a) (x  b) + (x  b) (x  c) + (x  c) (x  a) = 0
are equal if and only if a = b = c.
xi) Let a, b, c be real numbers such that a + 2b + c = 4. Find max(ab + bc + ca) .
xii) If  and  are the roots of the equation ax2 + bx + c = 0 express the roots of the
equation a3x2 –ab2x + b2c = 0 in the terms of  and .
xiii) If |2x –3| + |2x +3| = 6, then find x.
xiv) Find the values of the parameter a  0 for which one of the roots of the quadratic
equation x2 –x + 3a = 0 is double of one of the roots of the equation x2 –x + a = 0.
xv) If the roots of the equation x2 + px + q = 0 differ from the roots of the equation
2
x + qx + p = 0 by the same quantity, then show that p + q + 4 = 0.

P-2022-CBSE-P1-MATHEMATICS- QEE
18
ANSWERS TO EXERCISES

Exercise 1:

(iv) p2q2x2 – p2q2x + p3 + q3 – 4pq = 0


(v) 0

Exercise 2:
(iii) 2 (v) 0

Exercise 3:
(i) 6, 3, 2
3 3
(ii) rp = q
5

(iii) 2
(iv) 4, 2, 5
(v) 1/2

Exercise 4:
(i) (–, 1)  (1, 3)  (4, )
(ii) [1, 6]
1 
(iii) ( ,  2)  , 1  (4, )
4 
(iv) ( 1, 2)  { 1}
(v) (2, 1)  (2, )

Exercise 5:
(v) (, 2)

Exercise 6:
(i) –1 < a < 2
(ii) no real values
(iii) 1m2
(iv) 6
(v) 5

Miscellaneous Exercise:
a b
(iii) , (vii) x =  a, (viii) x = 1, 1.
ab

xi) 4 xii) 2 +  or 2 + .

3 3
xiii) − x xiv) a = –2 xv) p+q+4=0
2 2

P-2022-CBSE-P1-MATHEMATICS-QEE
19
SOLVED PROBLEMS

Subjective:

Level – 0

Problem 1. The real numbers x1, x2, x3 satisfying the equation x3 – x2 + x +  = 0 are in A.P.
Find the intervals in which  and  lie.

Solution: Given that x2 – x1 = x3 – x2


and x3 – x2 + x +  = 0. …(1)
x1, x2 and x3 are roots of equation (1), then
x1 + x2 + x3 = 1 = 3x2  x2 = 1/3 and x1 + x3 = 2/3.
x x
Also x1x2 + x2x3 + x1x3 =  i.e. 1  3 + x1x3 = . …(2)
3 3
x1x 3
Again x1x2x3 = –     x1x3 = –3 …(3)
3
2 
 x1   x1  = – 3  3x12 – 2x1 – 9 = 0.
3 
1
For real value of x1, (–2)2 + 108  0     .
27
From equation (2), we have
2
x1 + x3 + 3x1x3 = 3   =  3 .
9
1 2 3 1
Since    ,   .
27 9 27 3

Problem 2. Find the value of ‘a’ for which ax2 + (a – 3)x + 1<0 for at least one positive real x.

Solution: Let f(x) = ax2 + (a – 3)x + 1


Case (i) If a < 0, then f(x) will be positive only for those values of x which lie
between the roots of the equation f (x) = 0.
As the interval between the roots can not cover all the positive real numbers
implies f(x) < 0 for at least one positive real x
Case(ii) If a > 0, then f(x) will be negative only for those values of x which lie
between the roots of f (x) = 0. From the graphs we can see that f(x) will be less
than zero for at least one positive real x, when f(x) = 0 has distinct roots and at
least one of these roots is positive real root.
 y
 

x x
x x x Fig. 2 x
Fig. 1 Fig. 3

y
  y

x x x x
Fig. 4 Fig. 5
2
For this D > 0, i.e. (a – 3) – 4a > 0  a < 1 or a > 9. … (1)

P-2022-CBSE-P1-MATHEMATICS- QEE
20
Both the roots are non–positive  sum  0 and product  0
a 3 … (2)
 at least one roots is positive if a < 3, and (1) is satisfied.
Combining (1) and (2) , we get a < 1 so that, 0 < a< 1.
Case (iii) If a = 0, the given inequality is satisfied for at least one x > 0.
Hence the required set of values of a is (–, 1).

Problem 3. Find the conditions on a, b, c, d such that the equations 2ax3 + bx2 + cx + d = 0
and 2ax2 + 3bx + 4c = 0 have a common root.

Solution: Let ‘’ be a common root of the given two equations.


3 2
Than 2a + b + c + d = 0 … (1)
and 2a2 + 3b + 4c = 0 … (2)
Multiply (2) with  and then subtract (1) from it, we get
2b2 + 3c  d = 0 … (3)
Now (2) and (3) are quadratic having a common root , so
2  1
2
 
3bd  12c 8bc  2ad 6ac  6b2
bd  4c 2 4bc  ad
2 = , =
2
2b  2ac 3ac  3b2
Eliminating  from these two equations we get,
9
(4bc + ad)2 = (bd + 4c2 )( b2  ac) which is the required condition.
2

x 2  34 x  71
Problem 4. Find the range in which the value of the function lies for all real
x 2  2x  7
values of x.

x2  34x  71
Solution: Let =y
x 2  2x  7
 x2(y – 1) – 2x(17 – y) + 71 – 7y = 0.
For real values of x, (17 – y)2 – (y – 1)(71 – 7y)  0
 8y2 – 112y + 360  0  y2 – 14y + 45  0
 (y – 9)(y – 5)  0  y  5 or y  9.
Thus for real values of x, the given expression is either  5 or  9.

Level – I

Problem 5. Let a, b, c be real numbers with a  0 and let ,  be the roots of the equation
ax2 + bx + c = 0. Express the roots of a3x2 + abcx + c3 = 0 in terms of , .

b c
Solution: Here +  =  and  = so that
a a
a3x2 + abcx + c3 = 0 leads to
3
2 b c c 
x +  x  0
a a a
or x2  ( + ) x + 33 = 0
i.e. x2  (2 + 2)x + 33 = 0
or (x  2) (x  2) = 0
 x = 2, 2.

P-2022-CBSE-P1-MATHEMATICS-QEE
21
Alternative Solution:
Dividing by c2, the second equation can be written as
2
a  a 
a  x  b x  c  0
c  c 
ax c c
so that  ,  or x = ,  = 2, 2.
c a a

Problem 6. Find the values of ‘a’ for which the equation


(x2 + x + 2)2 – (a – 3)(x2 + x + 2)(x2 + x + 1) + (a – 4)(x2 + x + 1)2 = 0 has at least
one real root.

Solution: The given equation can be rewritten as


2
 x2  x  2   x2  x  2 
 2   (a  3)  2   (a  4)  0 .
 x  x 1  x  x 1
x2  x  2 1
Let 2
= t or, t = 1  2
.
x  x 1 x  x 1
2
 1 3
Since x2 + x + 1 =  x    ,
 2  4
2 3  7
x +x+1  t   1, .
4  3 
Now the given equation reduces to
t2 – (a –3)t + (a – 4) = 0.
 7
At least one root of this equation must lie in  1, .
 3 
(a  3)  (a  3)2  4(a  4)
Now, t =  t = a – 4, 1.
2
 7 7 19
For one root to lie in  1,  , we must have 1 < a – 4  5<a .
 3 3 3
Alternative Solution:
Putting x2 + x +1 = , we get
( + 1)2 – (a – 3)( + 1) + (a – 4)2 = 0
1 1
 (5 – a) + 1 = 0   =   x2 + x + 1 =
5a a5
1
  3/4  a – 5 > 0 and a – 5  4/3  5 < a  19/3.
a5

Problem 7. Given real numbers a, b, c and a  0. If  is a root of a2x2 + bx + c = 0,  is a root


of a2x2 – bx – c = 0, and 0 <  < , then show that the equation a2x2 + 2bx + 2c =
0 has a root  that always satisfies  <  < .
Solution:  is a root of a2x2 + bx + c = 0
2 2 2 2
 a  + b + c = 0  b +c = – a  . …(1)
2 2 2 2
 is a root of a x – bx – c = 0  b + c = a  . …(2)
Let f(x) = a2 x2 + 2bx + 2c. Then
f() = a22 + 2(b + c) = a2 2 – 2a2 2 = –a22  0, (from (1))
and f() = a2 2 + 2b +2c = 3a2 2 > 0. (from (2))
Thus, f(x) is a polynomial such that f() < 0 and f() > 0. Therefore there exits 
satisfying  <  <  such that f() = 0 .

P-2022-CBSE-P1-MATHEMATICS- QEE
22
2
Problem 8. Let a, b, c be real. If ax + bx + c = 0 has two real roots  and  where
c b
 < – 1 and >1, then show that 1    0.
a a

Solution:   – 1   + E1 = – 1 where E1  0.
Also   1   – E2 = 1 where E2  0.
c b
Now 1+   1        1  1  E1 1  E 2   E2  E1
a a
= 1 – 1 – E1 – E2 – E1E2 + E2 – E1
= – E1 –E2 – E1E2 + E2 – E1 if E2  E1.
= – E1 – E2 – E1E2 + E1 – E2 if E1  E2.
Hence L.H.S = – 2E1 – E1E2 or – 2E2 – E1E2.
c b
In both the cases 1    0. (E1, E2 > 0)
a a
Alternative Solution:
b c  
Let f(x) = x2 + x + .
a a -1 1
From graph, f (– 1) < 0 and f (1) < 0
c b c b c b
 1+  < 0 and 1 + + <0 1+ + < 0.
a a a a a a

Problem 9. If , are the roots of x2 + px + q = 0, and also of x2n + pnxn + qn = 0, and if is a

root of xn + 1 + (x + 1)n = 0, then prove that n must be an even integer,
where n  n.

Solution: ,  are the roots of x2 + px + q = 0   +  = – p. . . . (1)


Also ,  are the roots of x2n+pnxn+qn=0
 2n + pnn + qn = 0, ... (2)
2n + pnn + qn = 0. ... (3)
Subtract (3) from (2). We get 2n – 2n + pn(n – n) = 0
 n + n = –pn. (as n  n) ... (4)
n
 
Now is a root of xn  1  (x  1)n  0     1  ( /   1)n  0
 
n n n n n
  +  + ( + ) = 0  – p + (–p) = 0
from (1) and (4),
 –pn+(–1)npn = 0. This is true only if n is an even integer.

Problem 10. For what value of , 1 lies between the roots of the quadratic equation
3x2 – 3 sin x – 2 cos2 = 0.

Solution: Let f(x) = 3x2 – 3 sin x – 2 cos2.


2
The coefficient of x > 0  f(1) < 0
2 2
 3 – 3sin – 2cos  < 0  2sin  – 3sin + 1 < 0
 (2sin – 1) (sin – 1) < 0
1  
 < sin < 1  2n + <  < 2n + .
2 6 2

P-2022-CBSE-P1-MATHEMATICS-QEE
23
Level – II

Problem 11. Prove that for any value of a the inequality


(a2 + 4)x2 + (a + 2) x − 4 < 2 is true for at least one negative x.
2 2
Solution: Let f(x) = (a + 3) x + (a + 2) x − 6
First of all
>0
(a + 2)2 + 24(a2 + 3) > 0
i.e. for all values of a roots are real and distinct. The graph of the function is
concave upwards so f(x) < 0 only between the roots of the equation.
6
Product of the roots of f(x) = 0 is − 2 < 0 for all values of a, i.e. roots are
a 3
always of opposite signs.
So f(x) < 0 is satisfied for all negative values of x between the negative root and
zero irrespective of value of a. Hence f(x) < 0, for at least one negative x, for all
real values of a.

Problem 12. Let x, y, z be real variables satisfying the equations x + y + z = 6 and


xy + yz + zx = 7. Then find the range in which variables can lie.

Solution: x+y+z=6
xy + yz + zx = 7 ….(1)
(1)  z = 6 – x – y ….(2)
Putting the value of z in (2)
xy + y(6 – x – y) + x(6 – x – y ) = 7
or, y2 + y(x –6) + x2 – 6x + 7 = 0
since y is real, so
(x – 6)2 – 4 (x2 – 6x + 7)  0
or, 3x2 – 12x – 8  0
6  2 15 6  2 15
 x
3 3
since (1) and (2) are symmetrical in x, y and z. So all the variables lie in the
interval
 6  2 15 6  2 15 
 , 
 3 3 

3 2 2
Problem 13. If equations ax + 2bx + 3cx + 4d = 0 and ax + bx + c = 0 have a non-zero
2 2
common root, then prove that (c – 2bd) (b – ac)  0.

Solution: Let  be the non-zero common root


so, a3 + 2b2 + 3c + 4d = 0 ….(1)
a2 + b + c = 0 ….(2)
(1) − II gives
b2 + 2c + 4d = 0 ….(3)
by (2) and (3)
2  1
 
2c 2  4bd 4ad  bc b2  2ac
(4ad  bc)2 2c 2  4bd
  2
(b2  2ac)2 b  2ac
 (4ad  bc)2 = 2(b + 2ac) (c2 – 2bd)
 (b2 – 2ac) (c2 – 2bd)  0

P-2022-CBSE-P1-MATHEMATICS- QEE
24
Problem 14. If the equation x4 – 4x3 + ax2 + bx + 1 = 0 has four positive roots, then find a
and b .
Solution: Let the roots be x1, x2, x3 and x4 . then
x1+ x2 + x3 + x4 = 4 and x1 x2 x3 x4 = 1
 A.M. = G.M ( for roots)  x1= x2 = x3 = x4
 xi = 1,  i  x4 – 4 x3 + ax2 + bx +1 = (x – 1)4
a= 6, b=-4.
2 2
Problem 15. Find the range of real values of x and y if 2x + 6xy + 5y = 1
Solution: 2x2 + 6xy + 5y2 = 1 …. (1)
Equation (1) can be rewritten as
2x2 + (6y) x + 5y2 – 1 = 0
Since x is real
2 2
36y – 8(5y –1)  0
2
 y  2   2  y  2
Equation (1) can also be rewritten as 5y2 +(6x)y +2x2-1 = 0
Since y is real, 36x2 – 20 ( 2x2 –1)  0
 36x2  40x2 + 20  0
 4x2   20  x2  5   5  x  5 .

Problem 16. If a, b, c are in G.P, then the equations ax2 + 2bx + c = 0 and dx2 + 2ex + f = 0
d e f
have a common root, then prove that , , are in A.P.
a b c

Solution: a, b, c are in G. P  b2 = ac.


Now the equation ax2 + 2bx + c = 0 can be rewritten as
c
ax2 + 2 ac x + c = 0  ( a x + c )2 = 0  x = – .
a
If the two given equations have a common root, then this root must be
c c c
– . Thus d  2e f 0
a a a
d f 2e c 2e 2e d e f
      , , are in A. P.
a c c a ac b a b c

Problem 17. Prove that roots of the quadratic equation


x 2
  2

 4 x  3   x  6 x  8  0,   R are real

Solution: x 2
  
 4x  3   x 2  6x  8  0
x 2 1     2x  2  3    3  8   0
2
Discriminant. D  4  2  3   4 1    3  8 


D  4 2    1 
if  R then D  0
so root of given quadratic always real

Problem 18. If x 2  2ax  b  c, x  R , then prove that b  c  a2

Solution: x 2  2ax  b  c x  R
2
x  2ax  b  c  0 , D  4a2  4  b  c   0  a2  b  c

P-2022-CBSE-P1-MATHEMATICS-QEE
25

Objective:

Level – 0

True / False

Problem 1. If all the real solutions of the equation 4 x   a  3  2 x   a  4   0 are non


positive, then a lies between (4, 5].

Solution: True
4x   a  3  2x   a  4   0
x  0, Let y  2x
y2  a  3  y  a  4   0
The roots of Quadratic must lie b/w (0, 1]

Problem 2. If the equation 5 x 2  10x  log1 / 5 a  0 has real roots then the minimum
1
value of a is .
56

Solution: False
5x 2  10x  log1/ 5 a  0 has real roots.
100  20log1/ 5 a  0
5
 1
log1/ 5 a  5  log1/ 5  
5
1
a  0 and a  5
5
1
minimum value of a 
55

Problem 3. If a, b  R, a  0 and the quadratic equation ax 2  bx  1  0 has imaginary


roots, then a + b + 1 is positive.
Solution: True
f(x)  ax 2  bx  1
f(0) = 1 (positive)
2
i.e. f(x)  ax2  bx  1  0 ( ax – bx + 1 = 0 has imaginary roots)
f(-1) = a + b + 1  0
Problem 4. The set of values of ‘a’ for which the equation x 3  3 x  a  0 has three
distinct real roots , is (2, 2).
Solution: True
f(x)  x3  3x  a
f '(x)  3x 2  3 1
-1
f’(x) = 0 at x =  1
coefficient of x 3 is positive
f(x) has three distinct real roots is

P-2022-CBSE-P1-MATHEMATICS- QEE
26
i.e. f(1)  0 and f(1)  0
1 + 3 + a  0 and 1 – 3 + a  0
a  2 and a  2
a  (2, 2)

Fill in the Blanks

 1
Problem 5. If the expression  mx  1   is non-negative for all positive real x ,then the
 x
minimum value of m is ______

Solution: 1/4
We know that ax2 + bx + c  0 if a > 0 and b2 – 4ac  0.
1 mx 2  x  1
So, mx – 1 + 0 0
x x
 mx2 – x + 1  0 as x > 0.
Now, mx2 – x + 1  0 if m > 0 and 1 – 4m  0
or if m > 0 and m  1/4.
Thus, the minimum value of m is 1/4.

Problem 6. If x2 – 4x + log1/2a = 0 does not have two distinct real roots, then maximum value
of a is _______

Solution: 1/16
Since x2 – 4x + log1/2a = 0 does not have two distinct real roots,
discriminant  0
 16 – 4 log1/2 a  0  log1/2 a  4  a  1/16.

x2  1
Problem 7. The largest negative integer which satisfies  0 is ______
 x  2  x  3 

Solution: –2
By wavy curve method .
x2  1
 0  x  (–, –1)  (1, 2)  (3, ).
 x  2  x  3 
Therefore largest negative integer is –2.

Level – I

MCQ Single Correct

Problem 8. If  and  are the roots of the equation 2x2 – 3x – 6 = 0, then equation whose
2 2
roots are  + 2,  + 2 is
(A) 4x2 + 49x + 118 = 0 (B) 4x2 – 49x + 118 = 0
2
(C) 4x – 49x – 118 = 0 (D) x2 – 49x + 118 = 0

Solution: B

P-2022-CBSE-P1-MATHEMATICS-QEE
27
Here  +  = 3/2,  = –6/2 = –3 so that
2 2 2 49
S =  +  + 4 = ( + ) – 2 + 4 = ,
4
2 2 2 2 2 2 2 118
P =   + 2( +  ) + 4 =   + 4 + 2[( + ) – 2 ] = .
4
2  49   118  2
Therefore, the equation is x –  x    0  4x – 49x + 118 = 0.
 4   4 
2 2
Alternate: Let y = x +2, then 2x – 3x – 6 = 0
 (3x)2 = (2x2 – 6)2  [2(y – 2) – 6]2 = 9(y – 2)  4y2 – 49y + 118 = 0.

Problem 9. If the roots of the equation x2 – px + q = 0 differ by unity then


(A) p2 = 1 – 4q (B) p2 = 1 + 4q
2 2
(C) q = 1 – 4p (D) q = 1 + 4p

Solution: B
Suppose the equation x2 – px + q = 0 has the roots  + 1
and  then  + 1+  = p  2 = p – 1 . . . . (1)
and (+1)  = q  2 +  = q. . . . .. (2)
Putting the value of  from (1) in (2), we get
2
p  1 p 1 2
 q  (p – 1) + 2(p – 1) = 4q
4 2
 p2 – 1 = 4q  p2 = 4q + 1.
Alternative: Let  and  be the roots. | – | = 1 ( + )2 – 4 = 1
 p2 – 4q = 1, or p2 = 1+ 4q.

Problem 10. The quadratic equation whose roots are A.M. and H.M. between the roots of the
equation ax2 + bx + c = 0 is
(A) abx2 + (b2 + ac)x + bc = 0 (B) 2abx2 + (b2 + 4ac)x + 2bc = 0
(C) 2abx2 + (b2 + ac)x + bc = 0 (D) none of these
Solution: B
Let (, ) be the roots of the given equation, then
b c
 +  = – ,  =
a a
2     2        2 
Required equation is x – x      =0
 2      2     
 2abx2 + (b2 + 4ac)x + 2bc = 0.

Problem 11. The expression ax2 + by2 + cz2 + 2ayz + 2bzx + 2cxy can resolved into rational
factors, then a3 + b3 + c3 is
(A) abc (B) a + b + c
(C)  1 (D) none of these

Solution: D
The given expression is ax2 + by2 + cz2 + 2ayz + 2bzx + 2cxy

P-2022-CBSE-P1-MATHEMATICS- QEE
28

  x 2 y
2
y x  x  y  
= z2 a   b   c  2a   2b   2c   
  z  z z z  z  z  
x y
= z2 [aX2 + bY2 + 2cXY + 2bX + 2aY + c] … (1) (where X = ,Y  )
z z
The given expression can be resolved into rational factors if the expression within
brackets in (1) is expressible into rational factors, the condition for which is abc +
2abc  a. a2  b. b2  c. c2 = 0  a3 + b3 + c3 = 3abc.
2
Problem 12. Let p and q be the roots of the equation x – 2x + A = 0 and let r and s be the
2
roots of the equation x – 18x + B = 0. If p < q < r < s are in arithmetic
progression then the values of A and B are given by
(A) A = 3, B = 77 (B) A = 3, B = 7
(C) A = –3, B = 77 (D) A = 3, B = –7
Solution: C
Let the four numbers in A.P. be a – 3d, a – d, a + d and a + 3d corresponding to
the numbers p, q, r and s respectively.
We have p + q = 2, pq = A for x2 – 2x + A = 0
and r + s = 18, rs = B for x2 – 18x + B = 0.
Therefore p + q +r + s = 4a = 20  a = 5.
Also p + q = 2  10 –4d = 2  d = 2.
Hence the numbers are –1, 3, 7, 11.
Thus we have pq = A = –3 and rs = B = 77.

Level – II

Problem 13. If roots of the equation 2 x 2  4x  2 sin   1  0 are of opposite sign, then 
belongs to
  5      5 
(A)  ,  (B)  0,   , 
6 6   6  6 
 13 17  
(C)  ,  (D) none of these
 6 6 

Solution: B
2x 2  4x  2 sin   1  0
coefficient of x 2 is positive
f(0)  0
2sin   1  0
1
sin  
2
    5 
   0,    ,  
 6  6 

O /6 5/6  13/6 17/6

P-2022-CBSE-P1-MATHEMATICS-QEE
29
2
Problem 14. The set of values of p for which the roots of the equation 3x + 2x + p(p – 1) = 0
are of opposite sign is
(A) (–, 0) (B) (0, 1)
(C) (1, ) (D) (0, )

Solution: B
Since the roots of the given equation are of opposite signs, product of the
roots< 0
p  p  1
  0  p(p – 1) < 0  0 < p < 1.
3
For real roots, 4  12p(p  1)  0  3p2  3p  1  0
1 7 1 1 7
 1 p   0 < p < 1.
2 3 2 2 3

mx 2  3x  4
Problem 15. If the inequality  5 is satisfied for all x  R, then
x2  2 x  2
(A) 1 < m < 5 (B) –1 < m < 5
71
(C) 1< m < 6 (D) m <
24

Solution: D
We have x2 + 2x + 2 = (x + 1)2 + 1 > 0,  x  R.
mx2  3x  4 2 2
Therefore, 2  5  mx + 3x + 4 < 5(x + 2x + 2)
x  2x  2
 (m – 5)x2 – 7x – 6 < 0,  x  R.
This is possible if D = b2 – 4ac = 49 + 24(m – 5) < 0 and m – 5 < 0
71
m< .
24

Problem 16. The value of p, for which both the roots of the equation 4x2  20px + (25p2 + 15p
 66) = 0 are less than 2, is
4   4 
(A)  , 2  (B)  1, 
5   5 
(C) (2, ) (D) ( ,  1)

Solution: D
22
(i). D  0  15p – 66  0 p  …(1)
5
2
(ii) 4 f (2) > 0  p – p – 2  0  p  (–, –1)  (2, ) …(2)
b 20p 4
(iii) – <2 <4p< …(3)
2a 4 5
Taking intersection of (1), (2) and (3), we get p  (–, –1).

Problem 17. Sum of the real roots of the equation x2 + 5|x| + 6 = 0


(A) equals 5 (B) equals 10
(C) equals –5 (D) does not exit

P-2022-CBSE-P1-MATHEMATICS- QEE
30
Solution: D
Since x2, 5|x| and 6 are positive so x2 + 5|x| + 6 = 0 does not have any real root.
Therefore sum does not exist.

Problem 18. If c > 0 and 4a + c < 2b then ax2 – bx + c = 0 has a root in the interval
(A) (0, 2) (B) (2, 4)
(C) (0, 1) (D) (–2, 0)

Solution: A
Let f(x) = ax2 – bx + c
f(0) = c > 0 and f(2) = 4a –2b + c < 0 so that f(x) = 0 has a root in the interval
(0, 2).

Problem 19. Let a,b,c,a1 ,b1 ,c1 , R and ax 2  bx  c  0 x  R


and a1 x 2  b1 x  c1  0 x  R . Then
(A) aa1 x 2  bb1 x  cc1  0 x  R
(B) aa1 x 2  bb1 x  cc1  0 x  R
(C) aa1 x 2  bb1 x  cc1  0, will have real roots.
(D) Nothing can be said in general about the nature of roots of,
aa1 x 2  bb1 x  cc1  0,

Solution: We have ax 2  bx  c  0 x  R
2
a1x b1x  c1  0 x  R
2
So, b  4ac  0 , a  0, c  0
and b12  4a1c1  0 , a1  0, c1  0
b2  4ac and b12  4a1c1
b2b12  16a1c1ac
2
 bb1   4a1acc1  12a1ac  c1 (positive)
b2b12  4aa1cc1 is discriminate of aa1x2  bb1x  cc1  0
If may be negative, zero, or positive that way we can’t say nature of the root.

MCQ Multi Correct


Problem 20. If 4ac > b2 and a + c > b for real numbers a, b and c, then which of the following
is true?
(A) a > 0 (B) c > 0
(C) a + b + c > 0 (D) 4a + c > 2b
Solution: A, B, C, D
Let f(x) = ax2 +bx +c
2
Its discriminant D = b - 4ac < 0
And f(-1) = a- b +c > 0
So, f(x) > 0 for all x  R
So, a > 0
f(0) >0  c > 0
f(1) > 0  a +b +c > 0
f( -2) > 0  4a – 2b + c > 0 .

P-2022-CBSE-P1-MATHEMATICS-QEE
31

Problem 21. 5
 2
log5 x  9x  24
 x 1

(A) x  R (B) x  ( 0, )
(C) x  ( -, 0) (D) none of these
Solution: A, B, C
5
5 
log x2  9x  24 
 x  1  x – 9x +2 4 > x –1
2

2 2
 x – 10x + 25 > 0  ( x – 5) > 0.
Which is true for  x  R.
Problem 22. Let f(x) is a quadratic expression with positive integral coefficients such that for

every ,   R,  > ,  f  x  dx > 0. Let g(t) = f(t) f(t), and g(0) = 12, then

(A) 16 such quadratics are possible
(B) f(x) = 0 has either no real root or distinct roots
(C) minimum value of f(1) is 6
(D) maximum value of f(1) is 11
Solution: A, C, D

Let f(x) = ax2 + bx + c. Given  f  x  dx > 0 for all  > , so f(x) = 0 has no real

roots.
Now g(0) = f(0) f(0) = 2ac  ac = 6.
 a and c are positive integers. So the possible values are a = 6, c = 1; a = 1,
c = 6; a = 2, c = 3; a = 3, c = 2.
Again b2 < 4ac or b2 < 24. So b can be 1, 2, 3, 4.
 In all 4  4 = 16 such quadratic are possible.
Now f(1) = a + b + c, then maximum value of (1) = 7 + 4 = 11 and minimum value
of f(1) = 5 + 1 = 6.

Problem 23. 0 < c < b < a and the roots ,  of the equation cx2 + bx + a = 0 are imaginary,
then
|||| 1 1
(A) = || || (B) 
4 || ||
1 1
(C)  2 (D) none of these
|| ||

Solution: B, C
Since roots are imaginary.
So, discriminant < 0
b  i 4ac  b2
=
2c
b  i 4ac  b2
=
2c
b2 4ac  b2 a
|| = || = 2
 2
  1.
4c 4c c

P-2022-CBSE-P1-MATHEMATICS- QEE
32
Assertion/Reasoning
Direction: Read the following carefully and give the answers as follows:
Option (A) if both the statements are TRUE and STATEMENT-2 is the correct explanation of
STATEMENT-1
Option (B) if both the statements are TRUE but STATEMENT-2 is NOT the correct
explanation of STATEMENT- 1
Option (C) if STATEMENT-1 is TRUE and STATEMENT-2 is FALSE.
Option (D) if STATEMENT-1 is FALSE and STATEMENT-2 is TRUE.
Problem 24. Statement–1: Least degree of polynomial expression with integer coefficients
1/7 1/7
3 5
one of whose zeros is      is 7.
5 3
2
Statement–2: x (3y – 5) = y + 1 has no integral solution.
Solution: C
1/7
5
For A, Let    t
3
and T = t + t –1
 t 3  t 3  T 3  3T
 
t 5  t 5  T 5  5 T 3  3T  10T  T 5  5T3  5T

and 7
t t 7
T 7
 7 T 5
  
 5T 3  5T  21 T 2  3T  35T
7 5 3
 T  7T  14T  7T
where T is a zero of the polynomial.
3 5
t 7  7t 5  14t 3  7t     and multiplying it by 15 we get the required result.
5 3
 deg.  7
 A is true.
In R. one of solution (1, 1) sin a(1, 1) satisfy it
R is false.

Problem 25. Statement–1: If f(x) = x4 + ax3 + bx2 + cx + 8, a, b, c  R and f(x) = 0 has 4 real
roots, then f(x) = 0 has then real roots.
Statement–2: If f(x) = 0 of n degree has n real roots then
f ’ (x) = 0 (n – 1) real roots.

Solution: A
A and R are correct and R is the correct explanation of A.

Comprehension–I
If a group of p consecutive terms is missing from the polynomial equation f(x) = 0,
then
(i) If p is even, the polynomial equation f(x) = 0 has at least p imaginary roots.
(ii) If p is odd, there are at least (p + 1) or at least (p – 1) imaginary roots of f(x)
= 0, according as the terms which immediately preceed and follow the
group, have like or unlike signs (1 – 4):
18 17 2
Problem 26. The no. of at least imaginary roots of the equation x + 2x – 7x + 13x + 5 = 0
is
(A) 15 (B) 14
(C) 16 (D) 13

P-2022-CBSE-P1-MATHEMATICS-QEE
33
Solution: B
14 Terms are missing between the terms 2x17 and -7x2. Hence at least no. of
imaginary roots = 14.
Problem 27. The no. of maximum real roots of the equation 2x88 + 3x87 – 13x2 + 5x + 9 = 0 is
(A) 3 (B) 5
(C) 4 (D) 2.
Solution: D
At least no. of imaginary roots = 84.
 Max. real roots = 88 – 84 = 4.
Problem 28. The no. of at least imaginary roots of the equation x8 + 7x2 + 43x + 7 = 0 is
(A) 4 (B) 6
(C) 5 (D) 2.
Solution: B
No. of missing terms between x8 and 7x2 is 5. Sign of x8 and 7x2 are same.
 at least no. of imaginary roots = (5 + 1) = 6.
Problem 29. The no. of maximum real roots of the equation
3x17 + 4x16 + 3x15 – x3 + 5x2 + 6x + 8 = 0 is
(A) 7 (B) 8
(C) 10 (D) 12.
Solution: A
No. of missing terms between 3x15 and –x3 is 11. But sign of 3x15 and –x3 are
different. Therefore at least no. of imaginary roots = 11 – 1 = 10.
 No. of max. real roots = 17 – 10 = 7.

Comprehension–II

If f(x) = (x  )n g(x), then we always have


f() = f() = f() = … = f n1() where f(x) and g(x) are polynomial functions.
Provided that f(x) has rational coefficients.

Problem 30. If f(x) is of degree 4 and touches x-axis at  


3 , 0 , then
(A) sum of the roots of f(x) is 0
(B) product of the roots of f(x) is 27
(C) sum of the product of the roots taken three at time is 12 3
(D) none of these

Solution: A
If f(x) touches x-axis at   
3, 0 and also touches at  3, 0 
 roots of f(x) are 3, 3,  3,  3 .

Problem 31. Suppose f(x) touches x-axis at only one point then the point of touching is
(A) always a rational quantity
(B) may or may not be a rational number
(C) never a rational number
(D) none of these

Solution: A

P-2022-CBSE-P1-MATHEMATICS- QEE
34
f(x) touches x-axis at only one irrational point then f(x) = (x  )2 g(x) where  is
irrational
 coefficients of f(x) cannot be rational
 If f(x) is moving rational coefficients then point of touching is also rational.

Matching Type

Problem 32. Match the following:


Column – I Column – II
(A). If all the solutions ‘x’ of acosx + a–cosx = 6 (a > 1) are (p) ( 2, 0)
real, then set of values of a is
2
(B). If both the distinct roots of the equation |sin x| + |sin
(q) [3+2 2 , )
x| + b = 0 in [0, ] are real, then values of ‘b’ are
(C). The set of values of ‘a’ for which all the solutions of (r) [–3, 3 ]
the equation 4sin4 x + asin2x + 3 = 0 are real and
distinct is
(D). If all roots of equation z3 + az2+ bz + c = 0 (a, b, c  (s) [-7, -4 3 )
R) are of unit modulus, then values of ‘a’ is

Solution: (A)  (q); (B)  (p); (C)  (s); (D)  (r)


(A). Let acosx = t
y = logax
1 +1
t+ =6
t y = loga(3+ 2 2 )
2 a>1
 t – 6t + 1 = 0
6  36  4
t = =32 2 O
2
cosx
a =32 2 -1 y = loga(3 – 2 2 )
 cosx = loga(3  2 2 )
since a > 1 , for all the roots to
be real,
we must have loga(3 + 2 2 )  1 and loga(3 - 2 2 )  -1,

(B). Given that |sin x|2 + |sin x| + b = 0


1  1  4b 1  1  4b
 |sin x| = 0 < 1   2 < b < 0.
2 2
2 2
(C). Put sin x = t and f (t) = 4t + at + 3
For required condition both the roots of the equation f (t) = 0 should be
distinct and should lie in the interval [0, 1]
2
for this, D > 0  a –48 > 0  a < -4 3 , a > 4 3
f (0)  0, f (1)  0  a  -7
a
Since 0 < sum of roots < 2  0 < - < 2  -8 < a < 0
4
so required ‘a’ will be –7  a < -4 3 .
(D). z1 + z2 + z3 = –a
| z1 + z2 + z3| = |a|  |z1| + |z2| + |z3|
 1 + 1 + 1  |a|  3.

P-2022-CBSE-P1-MATHEMATICS-QEE
35
Problem 33. If f(x) = x + ax + bx + c = 0 has three distinct integral roots and (x + 2x + 2)3 +
3 2 2

a(x2 + 2x + 2)2 + b(x2 + 2x + 2) + c = 0 has no real roots then


List – I List – II
(P) The minimum value of a is equal to (1) 0
(Q) The minimum value of b is equal to (2) 2
(R) The minimum value of c is equal to (3) 3
(S) If the roots of f(x) = k are equal then k is equal to (4) –1
(where a, b, c have taken their minimum values)
Codes:
P Q R S
(A) 4 1 2 3
(B) 3 2 1 4
(C) 1 3 4 2
(D) 2 4 3 1
Solution: B
3 2
x + ax + bx + c = 0 has three distinct integral roots
and (f(x))3 + a(f(x))2 + b(f(x)) + c = 0 has no real roots, where f(x) = x2 + 2x + 2.
Let the roots of x3 + ax2 + bx + c = 0 be x1 > x2 > x3 respectively.
Since f(x) can take all values from [1, )
 x1  0  x2  – 1  x3  – 2  a = – (x1 + x2 + x3)  3
b = x1x2 + x2x3 + x3x1
 b  2 and c = – (x1 x2 x3)  c  0
roots of f(x) = k are equal  3x2 + 2ax + b – k = 0 has equal roots
a2
 a2 = 3(b – k)  k = b – = 2 – 3 = – 1.
3

Problem 34. Match the following:


List – I List – II
(P) If one root of 2x2 – 16x + k = 0 lies in (3, 4) and another in (4, (1) 11
5), then the integral value of k is
(Q) If p, q  {1, 2, 3, 4, 5} and px2 + qx + 1 = 0 has real roots, (2) 8
then the number of possible ordered pair (p, q) is
(R) the least value of cosec2x + 25sec2x is (3) 31
(S) If P is (x, y), F1(6, 0), F2(–6, 0) and 5x2 – 4y2 = 80, then PF1 – (4) 28
PF2 is
Codes:
P Q R S
(A) 1 4 3 1
(B) 4 2 1 3
(C) 2 3 1 4
(D) 3 1 4 2

Solution: D
(P). The roots are real and distinct  k < 32
f(3) f(4) < 0  (k – 30)(k – 32) < 0
f(4) f(5) < 0  (k – 32)(k – 30) < 0
Integral value of k is 31.
2
(Q). q  4p
If p = 1, q can be either 2, 3, 4, 5
If p = 2, q can be either 3, 4, 5
If p = 3, q can be either 4, 5
If p = 4, q can be 5
If p = 5, q can be 5

P-2022-CBSE-P1-MATHEMATICS- QEE
36
 possible ordered pairs (p, q) = 11.
2 2
(R). S = (1 + cot x) + 25(1 + tan x)
= 26 + (cotx – tanx) + 2 = 28 + (cotx – tanx)2
2

least value of S is 28.


x2 y 2
(S). The equation  1
16 20
3
a2 = 16, b2 = 20  e =
2
 F1, F2 are the focii of the hyperbola
PF1 – PF2 = 2a = 8.

Numerical based

Problem 35. The least positive integral value of real  so that the equation (x – a)(x – c)(x – e)
+ (x – b)(x – d) = 0, (a > b > c > d > e) has distinct real roots is
______________

Solution: 1
f  x   x  a  x  c   x  e     x  b   x  d
 f  a     a  b   a  d
 f b   b  a   b  c  b  e   0
 f c    c  b  c  d
 f  d   d  a   d  c   d  e   0
 f  e     e  b   e  d
If   0 f  a   0 , a root lies between b and a.
If  < 0 f  e  < 0, a root lies between e and d.
Always a root lies between d and b  all roots are real and distinct as exactly
two can’t be real. If  = 0 roots are a, c and e.

n
2
Problem 36. The number of real roots of the quadratic equation x  k   0  n  1 is.
k 1

Solution: 0
n
 (x  k)2  0
k 1
number of real root is zero.

Problem 37. The least integral value of k such that  k  2  x 2  8 x  k  4 is positive for all
real values of x is

Solution: 5
 k  2  x 2  8x  k  4   0 x  R
D = 64 – 4(k – 2) (k + 4)  0 and k – 2  0
(k + 56) (k – 4)  0 k2
k  6 or k  4
k4
k=5

P-2022-CBSE-P1-MATHEMATICS-QEE
37
Decimal Type

Problem 38. If the equation x4 + px3 + qx2 + rx + 5 = 0 has four positive real roots, then find
the minimum value of pr.

Solution: 80.00
Let , , ,  be the four positive real roots of the given equation. Then
 +  +  +  = – p.
 +  +  + +  +  = q.
 +  + +  = –r.
 = 5.
Using A.M  G.M
              4
    4 33  3 3    5
4 4
 p  r 
       5  pr  80  minimum value of pr = 80.
 4  4 

Problem 39. Find a, if ax2 – 4x + 9 = 0 has integral roots.

Solution: 0.33
1
Let a = , so that the given equation becomes x2 – 4bx + 9b = 0.
b
This equation has integral roots if b is an integer and 16b2 – 36b is a perfect
square
Let b(4b – 9) = k2  4b2 – 9b – k2 = 0
2
 9 81
 2
 2b    k   (8b – 9)2 – 16k2 = 81
 4 16
 (8b – 9 – 4k) (8b – 9 + 4k) = 81 = 327.
Since b and k are integers, 8b – 9 – 4k = 3 and 8b – 9 + 4k = 27
1
 16b – 18 = 30  b = 3  a = = 0.33.
3
For any other factorization of 81, b will not be an integer.

P-2022-CBSE-P1-MATHEMATICS- QEE
38
CHAPTER PRACTICE PROBLEM

Subjective:

Level – I

1. If ,  are the roots of the equation ax2 + bx + c = 0, then find the equation whose roots
1 1
are , .
a  b a  b

2.  
If 2  i 3 is a root of the equation x 2  px  q  0 where p and q are real,
then find (p, q).

3. If the roots of the equation x 2  a2  8x  6a are real, then find the interval of a.

Level – II


4. In a triangle PQR, R  . If tan(P/2) and tan(Q/2) are the roots of the equation
2
ax 2  bx  c  0 where a  0, then find the relation in a, b and c.

5. If roots of ax 2  bx  c  0 are ,  and roots of Ax 2  Bx  C  0 are  + k,  + k. then

prove that
B 2
 4AC  = (A/a)2.
b 2
 4ac 

Objective:

MCQ Single Correct


2
1. The number of real solutions of the equation x  3 x  2  0. is
(A) 4 (B) 3
(C) 1 (D) 2

2. If sin  and cos  are the roots of the equation ax 2  bx  c  0, then


2 2
(A)  a  c   b2  c 2 (B)  a  c   b2  c 2
2 2
(C)  a  c   b2  c 2 (D)  a  c   b2  c 2

3. 
The values of ‘a’ for which the quadratic equation 3x 2  2  a  1 x  a2  3a  2  0 
possesses roots of opposite sign are
(A) 1< a < 2 (B) a  (2, )
(C) 1 < a < 3 (D) None of these

P-2022-CBSE-P1-MATHEMATICS-QEE
39
2
4. In copying a quadratic equation of the form x  px  q  0 a student wrote the coefficient
of x incorrectly and the roots were found to be 3 and 10,another student wrote the same
question but he wrote the constant term incorrectly and thus he found the roots to be 4
and 7. The roots of the correct equation are.
(A) 5, 6 (B) 4, 6
(C) 4, 5 (D) None of these

5. For a  0, the roots of the equation


x 2  2a x  a  3a2  0.
(A) a  6 1  (B) a  6  1
  
(C) a 1  2 ,a 1  6  (D) a 1  2 

2
6. The sum of the real roots of the equation x  2  x  2  2  0 , is
(A) 2 (B) 6
(C) 4 (D) 8

 x 2 .2
x 3 2 x 3  4
7. The number of negative integral solution of x 2 .2x 1  2  2x l is
(A) 4 (B) 2
(C) 1 (D) 0

8 If c, d are the roots of the equation  x  a  x  b   k  0. , then the roots of the equation
 x  c  x  d  k  0 are
(A) c, d (B) a, c
(C) b, d (D) a, b

9. The values of ‘m’ so that the equations 3x 2  2mx  4  0 and x  x  4m  2  0 may
have a common root, is
1 1
(A)  (B) 
2 2 2
1 1
(C) (D) 
2 2

MCQ Multi Correct

1. Let A, G and H be the arithmetic mean, geometric mean and harmonic mean of two
positive numbers a and b, then the quadratic equation whose roots are A and H may be
(A)  
Ax 2  A 2  G2 x  AG2  0 
(B) Hx 2  H2  G2 x  HG2  0 
(C) Ax 2  A 2
 H  x  AH
2 2
0 (D) all above

2. Let the positive numbers a, b, c be in G.P. and  be a complex cube root of unity then the
equation ax 2  bx  c  0 must have
(A) ratio of roots as 1 :  (B) ratio of roots as b : ac
(C) real roots (D) imaginary roots

P-2022-CBSE-P1-MATHEMATICS- QEE
40
2x  1
3. If S is the set of all real x such that is positive, then S contains
2x  3x 2  x
3

 3  3 1
(A)  ,   (B)   ,  
 2  2 4
 1 1 1 
(C)   ,   (D)  , 3 
 2 4 2 

4. The adjoining figure shows the graph of y  ax 2  bx  c . Then


Y

O X
(x1,0) (x2,0)

(A) a  0 (B) b2  4ac


(C) c  0 (D) a and b are of opposite signs

Numerical Based

1. If x  2  22/3  21/3 , then x 3  6x2  6x 

2. The number of real solutions of the equation | x |2 – 3 | x | 2  0 are

3. The number of real solutions of the equation | x2 + 4x + 3| + 2x + 5 = 0 are

4. If 3x 2  7x  30  2x2  7x  5  x  5 ,then x is equal to

5. The number of roots of the equation | x |2 7 | x | 12  0 is

6. A real root of the equation log4 {log2 ( x  8  x )}  0 is

1
7. If x  7  4 3 , then x  
x

8. The number of solutions for the equation x 2  5 | x |  6  0 is

9. If the roots of the given equation


(2k  1)x 2  (7k  3)x  k  2  0
are reciprocal to each other, then the value of k will be

P-2022-CBSE-P1-MATHEMATICS-QEE
41
ASSIGNMENT PROBLEMS

Subjective:

Level – 0

1. Solve the following equations:


(i) 5x2 + 5x -1 = 0 ,
(ii) pqx2  (p2  q2 )x  pq  0 ,
p q 1 1
(iii)  pq x , x ,
px  1 qx  1 p q
(iv) x 2  2px  p2  q2  r 2  2qr  0 ,
(v) 5 x 1  52 x  53  1 ,
x 1  x 13
(vi)   ,
1 x x 6
(vii) x2  7x  17  x 2  7x  8  9 ,
2

(viii) x 2  2   8x2  6x(x 2  2) ,
x 2 3 x  3 x2  3x  3
(ix)  2 3  
 2 3  4,
(x) x 2  a x  a  3a2  0, a  0.

2. (a) For what values of m, will the following equations have equal roots:
(i)  3m  1 x 2  2  m  1 x  m  0 ,
(ii)  m  1 x 2  2(m  3)x  m  8  0 .
(b) If the roots of the equation p  q  r  x 2  q  r  p  x  r  p  q   0 are equal, show
1 1 2
that
  .
p r q
(c) Determine the values of m and n, m, n > 0, such that both the equations
x 2  mx  n  0 and 2x 2  4x  mn  0 have equal roots.

3. A number exceeds its positive square root by 12. Find the number.

4. Find x, if x = 30  30  30  ...........to 

5. For the quadratic equation ax2  bx  c  0, find the condition that


(i) one root is reciprocal of the other root,
(ii) one root is m times the other root,
(iii) one root is square of the other root,
(iv) one root is nth power of the other root,
(v) the roots are in the ratio m : n.

6. If ,  are the roots of the equation x 2  px  q  0 and  4 , 4 are the roots of the
equation x 2  rx  s  0 , show that x 2  4qx  2q2  r  0 has real and distinct roots.

P-2022-CBSE-P1-MATHEMATICS- QEE
42

7. Find the value of a so that the equations x 2  11x  a  0 and x 2  14x  2a  0 have a
common root.

8. Solve the equation (6 – x)4 + (8 – x)4 = 16.

9. If x2 + 3x + 5 = 0 and ax2 + bx + c = 0 have a common root and a, b, c  N, then find the


minimum value of a + b + c.

10. Find the condition that one root of ax 2  bx  c  0 be the reciprocal of a root
of px 2  qx  r  0 .

11. Prove that the arithmetic mean of the roots of x2 – 2ax + b2 = 0 is equal to the geometric
2 2
mean of the roots of the equation x – 2bx + a = 0, and vice–versa.

12. If the roots of the equation x2 – px + q = 0 differ by unity, then prove that, p2 – 4q = 1

13. If x is real and the expression


x 2
 2x  11  takes all the values which do not lie between
 x  3
a and b, find a and b.

14. If the roots of the equation (b – c)x2 + (c – a)x + (a – b) = 0 be equal, then prove that a, b,
c are in arithmetic progression.

15. If c > 0 and 4a + c < 2b then find the interval in which one root of equation ax2– bx + c = 0
lies.

16. If the sum of roots of quadratic equation ax2+ bx + c = 0 is equal to the sum of the
 b2 bc 
squares of their reciprocals then find   .
 ac a2 

17. Find the quadratic equation whose roots are sec2  and cosec2 .

18. Find the values of ‘a’ for which the quadratic expression x2  ax + 4 is nonnegative for
all real values of x.

19. Let S be the set of values of ‘a’ for which 2 lie between the roots of quadratic equation
2
x + (a + 2) x – (a + 3) = 0. Then find the set S.

20. If one root of equation x2 − 3ax + f(a) = 0, is double of the other then find f(x).
2 2
21. If both roots of the equation x − 2ax + a − 1 = 0 lies between − 3 and 4, then find [a],
where [.] denotes greatest integer function.
2 2
22. Find the nature of the roots of equation (ax + bx + c)(ax – dx – c) = 0, x  0.
2 2
23. Find the sum of the non-real roots of the equation (x + x  2)(x + x  3) = 12.

24. If both the roots of the quadratic equation x2  6ax + 9a2  2a + 2 = 0 exceed 3, then find
set of all possible values of a.

25. Solve the equation 2logxa + logaxa + 3loga2 x a = 0, where a > 0, a  1.

P-2022-CBSE-P1-MATHEMATICS-QEE
43
Level – I

tan x
1. Show that the value of , whenever defined, never lies between 1/3 and 3.
tan3x

2. For what integral values of a, the equation x2  x(1  a) – (a + 2) = 0 has integral roots.
Find the roots

3. For what value of , 1 lies between the roots of quadratic equation 3x23sinx 2cos2=0.

2
4. The coefficient of x in the quadratic equation x + px + q = 0 was taken as 17 in place of
13. Its roots were found to be –2, and –15. Find the roots of the original equation.

2
6. If the ratio of the roots of the quadratic equation x + px + q = 0 be equal to the ratio of the
2 2 2
roots of x + lx + m = 0, then prove that p m = l q.

7. If , are the roots of the equation 3x2 + 2x + 1 = 0, then find the equation whose roots
1  1 
are , .
1  1 

8. If the quadratic equation ax2 + bx + c = 0 has real roots of opposite sign in the interval
c b
(–2, 2), then prove that 1 +   0.
4a 2a

9. Show that no real values of x and y other than 4 and 4 respectively will satisfy the
equation x2 – xy + y2 = 4(x + y – 4).


10. If a . 3tanx + a . 3tanx  2 = 0 has real solutions, x  , 0  x  , then find the set of all
2
possible values of parameter ‘a’.

11. The equations ax2 + bx + a = 0, x3– 2x2+ 2x– 1 = 0 have two roots in common. Then find
the value of a + b.
2 2
12. Find the set of values of p, for which x  px + p  4 < 0 for at least one x < 0.

P-2022-CBSE-P1-MATHEMATICS- QEE
44

Level – II

1. Let x2 – (m – 3)x + m = 0 (m  R) be a quadratic equation.

(i) Find the value of m for which the roots


(a) are real and distinct, (b) are equal,
(c) are not real, (d) are opposite in sign,
(e) are equal in magnitude but opposite in sign,
(f) are positive , (g) are negative,
(h) are such that at least one is positive.

(ii) Find the value of m for which


(a) one root is smaller than 2, the other root is greater than 2
(b) both the roots are greater than 2
(c) both the roots are smaller than 2
(d) exactly one root lies in the interval (1, 2)
(e) both the roots lie in the interval (1, 2)
(f) atleast one root lies in the interval (1, 2)
(g) one root is greater than 2, the other root is smaller than 1
(h) atleast one root is greater than 2.

2. For what real values of a do the roots of the equation x2 – 2x – (a2 – 1) = 0 lie between
the roots of the equation x2 – 2(a +1)x + a (a – 1) = 0 ?

3. If a1, a2, a3, …….an (n  2) are real and (n – 1)a12 – 2na2 < 0 then prove that at least two
roots of the equation xn + a1xn-1 + a2xn-2+ ... + an = 0 are imaginary.

4. Find all real value of a for which the equation x4 + (a – 1)x3 + x2 + (a – 1)x + 1 = 0
possesses at least two distinct positive roots

5. Find out minimum non-negative real values of a, b and c given that the equation
x4 + ax3 + bx2 + cx + 1 = 0 has real roots.

x2  ax  c
6. If x is real and a and b are unequal, then prove that the expression can take
x 2  bx  c
any value when c  0.

7. Show that if p, q, r, s, are real numbers and pr = 2(q + s) then at least one of the
equations x2 + px + q = 0, x2 + rx + s = 0 has real roots.

8. The equation ax2 + bx + c = 0 has real and positive roots. Prove that the roots of the
2 2
equation a x + a(3b – 2c)x + (2b – c)(b – c) + ac = 0 are real and positive.

9. Solve the equation log4 (2x2 + x + 1) – log2 (2x – 1) = 1.

10. If one root of the equation (l–m) x2 + lx + 1 = 0 is double of the other and l is real, find the
greatest value of m.

P-2022-CBSE-P1-MATHEMATICS-QEE
45

Objective:

Level – I

1. Let f(x) = x2 + bx + c, where b, c  R. If f(x) is a factor of both x 4 + 6x2 + 25 and 3x4 + 4x2
+ 28x + 5, then the value of b is
(A) 2 (B) 1
(C) –1 (D) –2

2. Let a, b, c be the sides of a triangle. No two of them are equal and   R. If the roots of
the equation x2 + 2(a + b+ c) x + 3 (ab + bc + ca) = 0 are real, then
4 5
(A)   (B)  
3 3
 1 5   4 5
(C)    ,  (D)    , 
3 3  3 3

3. For the equations x2 + bx + c = 0 and 2x2 + (b + 1)x + c + 1 = 0 select the correct


alternative
(A) both the equations can have integral roots
(B) both the equations can’t have integral roots simultaneously
(C) none of the equations can have integral roots
(D) nothing can be said

4. If x2 + ax + b is an integer for every integer x then


(A) ‘a’ is always an integer but ‘b’ need not be an integer.
(B) ‘b’ is always an integer but ‘a’ need not be an integer.
(C) a + b is always an integer. (D) none of these

5. If a, b, c be the sides of ABC and equations ax2 + bx + c = 0 and 5x2 + 12x + 13 = 0


have a common root, then C is
(A) 60° (B) 90°
(C) 120° (D) 45°

6. If the smaller root of the equation x2 + ax  4 = 0 lies between  1 and 2, then a belongs
to
(A) (0, ) (B) (0, 3)
(C) (, 3) (D) (, 3) (0, )

7. If the equations x2 + ax + b = 0 and x2 + bx + a = 0 have exactly one common root, then


the numerical value of a + b is
(A) 1 (B) –1
(C) 0 (D) none of these

8. The set of values of a for which the inequality x2 + ax + a2 + 6a < 0 is satisfied for all
x  (1, 2) lies in the interval
(A) (1, 2) (B) [1, 2]
(C) [–7, 4] (D) None of these

9. Let S denote the set of all real values of a for which the roots of the quadratic equation
x2  2ax + a2  1 = 0 lies between 5 and 10, then S will be
(A) (1, 2) (B) (2, 9)
(C) (4, 9) (D) (6, 9)

P-2022-CBSE-P1-MATHEMATICS- QEE
46

x2  14x  9
10. If x is real then the greatest value of is
x 2  2x  3
(A) 2 (B) 3
(C) 4 (D) 5

11. If |x2| + |x| –2 = 0, then the value of x is equal to


(A) 2 (B) –2
(C)  1 (D) None of these
2 2
12. If one root of the equation x + ax + 12 = 0 is 4, and the equation x – 2ax + 7b = 0 has
real roots, then b lies in the interval
(A) (0, 7) (B) (, 7]
(C) (–7, 0) (D) None of these

13. The equation 2x2 +3x + 1 = 0 has


(A) Rational roots (B) Irrational roots
(C) Imaginary roots (D) None of these

14. If (m2 –3)x2 + 3mx + 3m + 1 = 0 has roots which are reciprocals of each other, then the
value of m equals
(A) 4 (B) –2
(C) 2 (D) None of these

15. If ax2 + bx + 6 = 0 does not have two distinct real roots, then the least value of 3a + b is
(A) 2 (B) –2
(C) 1 (D) –1

16. Let two numbers have A.M.= 9 and G.M. 4. Then these numbers are the roots of the
quadratic equation
(A) x2 + 18x + 16 = 0 (B) x2  18x + 16 = 0
2
(C) x + 18x  16 = 0 (D) x2  18x  16 = 0

17. If (1  p) is a root of the quadratic equation x2 + px + (1  p) = 0, then its roots are


(A) 0, 1 (B) 1, 1
(C) 0, 1 (D) 1, 2

18. The number of real roots of the equation (x  1)2 + (x  2)2 + (x  3)2 = 0 is
(A) 2 (B) 1
(C) 0 (D) 3

19. If ,  are the roots of the equation ax2 + bx + c = 0 then the value of 3 + 3 is
3abc  b3 a3  b3
(A) (B)
a3 3ab
3 3
3abc  b b  3abc
(C) 3
(D)
a a3

20. The roots  and  of the quadratic equation ax2 + bx + c = 0 are real and of opposite sign.
Then the roots of the equation (x – )2 + (x – )2 = 0 are
(A) positive (B) negative
(C) real and of opposite sign (D) imaginary

P-2022-CBSE-P1-MATHEMATICS-QEE
47
2 4 2
21. If the equation (a  12)x  8x  4 = 0 has no real solution, then
(A)  4  a  4 (B)  2 3 < a < 2 3
(C) a  4 (D) none of these

Level – II

Multi-choice (Single Correct Answers)

1. One of the roots of the quadratic equation (sin2 ) x2  x + cos2  = 0 is given by


(A)  1 (B) 2
(C) 1 (D) none of these

2. The value of ‘p’ for which the sum of the square of the roots of 2x2 – 2(p – 2)x – p –1= 0
is least, is
(A) 1 (B) 3/2
(C) 2 (D) – 1

3. If the equation (a – 5)x2 + 2(a – 10)x + a + 10 = 0 has roots of opposite signs, then
(A) a > 10 (B)  15 < a < 5
(C)  10 < a < 5 (D) none of these

4. If sin, cos are the roots of equation cx2 + bx + a =0, then a, b, c are connected by the
relation
(A) b2 + 2ac – c2 = 0 (B) c2 + 2ac + b2 = 0
2 2
(C) b – 2ac – c = 0 (D) 2ac – b2 – c2 = 0

5. The set of values of ‘a’ for which 1 lies between the roots of x2 – ax – a + 3 = 0 is
(A) (–, –6) (B) (–, +6)
(C) (–, –6)  (2, ) (D) (2, )

6.  > 0 and  < 0 are the roots of the equation x2  px + q = 0, whereas  > 0 and  < 0
are the roots of the equation x2  px + q + k = 0 where k > 0. Then
(A)  > ,  >  (B)  < ,  < 
(C)  > ,  <  (D)  < ,  > 

7. Let a, b, c be three distinct positive real numbers. Then the number of real roots of
ax2 + 2b|x| + c = 0 is
(A) 0 (B) 1
(C) 2 (D) 4

8. Let S denote the set of real values of ‘a’ for which the roots of the equation
x2 – ax – a2 = 0 exceed ‘a’. Then S belongs to
(A) (– , 0) (B) (–2, – 1/2)
(C) (–1/2, 1/4) (D) null set

9. If a and b are distinct real numbers and difference between the roots of x2 + ax + b = 0
and that of x2 + bx + a = 0 is same then the value of a + b is
(A) 2 (B) – 2
(C) – 4 (D) 0

10. If the equation x2  (2 + m)x + (m2  4m + 4) = 0 has coincident roots, then


(A) m = 0, 1 (B) m = 0, 2
(C) m = 2/3 (D) m = 2/3, 6

P-2022-CBSE-P1-MATHEMATICS- QEE
48
2
11. One root of the equation 5x + 13x+ k = 0 is the reciprocal of the other, if
(A) k = 0 (B) k = 5
(C) k = 1/6 (D) k = 6
2
12. If the sum of the roots of the equation ax + bx + c = 0 is equal to the sum of the squares
2 2 2
of their reciprocals, then bc , ca , ab are in
(A) A.P. (B) G.P.
(C) H.P. (D) none of these
2 2
13. If the equation formed by decreasing each root of ax + bx + c = 0 by 1 is 2x + 8x + 2 = 0,
then
(A) a =  b (B) b =  c
(C) c =  a (D) b + a + c
2
14. The equation whose roots are opposite in sign to those of the equation x  3x  4 = 0 is
given by
(A) 4x2  3x + 1 = 0 (B) x2 + 3x  4 = 0
2
(C) x + 3x + 4 = 0 (D) none of these

15. Sum of the roots of the equation x5  5x3 + x + 1 = 0 is given by


(A) 0 (B) 5
(C)  1 (D) none of these

Multi-choice (One or More correct answers)

16. If p and q are odd integers, then the equation x2 + 2px +2 q = 0


(A) has no integral roots (B) has no rational roots
(B) has no irrational roots (D) has no imaginary roots.

x 2  ax  3
17. If takes all real values for possible real values of x, then
x2  x  a
(A) 4a3 + 39  0 (B) 4a3 + 39 < 0
(C) a < 1/4 (D) a  ¼

18. If the quadratic equation ax2 + bx + c = 0 (a > 0) has two roots  and  such that  <  2
and  > 2, then
2
(A) b  4ac > 0 (B) c < 0
(C) a + |b| + c < 0 (D) 4a + 2|b| + c < 0

19. Let a, b and c be three positive real numbers and are in G.P. and   1 be a cube root of
unity, then the equation ax2 + bx + c = 0 has
(A) real roots (B) imaginary roots
(C) imaginary roots with negative real part (D) roots are , 2 where  = b/a

20. The values of a for which x4 – 2ax2 + a2 – a = 0 has all real roots are
(A) – 1 (B) 1
(C) 2 (D) 3

21. If ordered pair (, ) where ,   I satisfy the equation 2x2 – 3xy – 2y2 = 7, then value of
 +  can be
(A) 5 (B) 4
(C) – 4 (D) 3

P-2022-CBSE-P1-MATHEMATICS-QEE
49
1/3 1/3 1/3
22. The solution of x + (2x – 3) = [3(x – 1)] is
(A) 0 (B) 3/2
(C) 1 (D) none of these

23. If f(x) = 0 is a polynomial whose coefficients all  1 and whose roots are all real, then the
degree of f(x) can be equal to
(A) 1 (B) 2
(C) 3 (D) 4

Reason Assertion

Questions (24 – 26) contain STATEMENT-1 (Assertion) and STATEMENT-2 (Reason).


Option (A) if both the statements are TRUE and STATEMENT-2 is the correct explanation of
STATEMENT-1
Option (B) if both the statements are TRUE but STATEMENT-2 is NOT the correct
explanation of STATEMENT- 1
Option (C) if STATEMENT-1 is TRUE and STATEMENT-2 is FALSE.
Option (D) if STATEMENT-1 is FALSE and STATEMENT-2 is TRUE.

24. STATEMENT  I: If the roots of the equation x2 + ax + b = 0, a, b  I, are rational, then


they must be integers
because
STATEMENT  II: If the coefficient of x2 in a quadratic equation with real coefficient is
unity, its roots must be integers

25. STATEMENT I : If a>b>c and a3 + b3 +c3 = 3abc then the equation ax2 + bx + c = 0 has
one positive and one negative real root.
STATEMENT II : If roots are of opposite nature, then product of roots< 0 and
sumof roots  0

COMPREHENSIONS
(I)
Read the following write up carefully and answer the following questions:

Lets consider the equation


ax2  bx  c  0 … (1)
where a, b, c  R and a  0. If equation (1) has roots  and  then
ax2  bx  c  a  x    x   
Also if a1,a2 ,a3 ,a 4 ... are in A.P then a2  a1  a3  a2  a 4  a3 ...
b b b
And if b1,b2 ,b3 ... are in G.P then 2  3  1 ...
b1 b2 b3

2 2
26. Let 1, 2 be the roots of x – x + p = 0 and 3 ,  4 be the roots of x – 4x + q = 0. If
1,  2 , 3 ,  4 are in G.P then the integral values of p & q respectively are
(A)  2,  32 (B)  2, 3
(C)  6, 3 (D)  6,  32

27. If the roots of the equation a(b – c) x2 + b(c – a) x + c(a – b) = 0 be equal, then a, b, c, are
in
(A) A.P. (B) G.P.
(C) H.P. (D) None of these

P-2022-CBSE-P1-MATHEMATICS- QEE
50

(II)
Read the following write up carefully and answer the following questions:

Let f(x) = a0 xn  a1xn 1  ...  an  a0  0 


If f(a) and f(b) are of opposite signs, (a, b  R ) i. e.
f(a) f(b) < 0 then at least one or in general odd number of roots of the equation f(x) = 0 lie
between a and b.
3
28. If 0  p  16 , then the equation x – 12x – p = 0 has one root in
(A) (2, 3) (B) (3, 4]
(C) (4, 5) (D) None of these

  
29. The equation 2sin2x2 – 3sinx + 1 = 0,    ,  has one root lying in the interval :
 4 2
(A) (0, 1) (B) (1, 2)
(C) (2, 3) (D) ( 1,0)

30. If f(x) = ax2 + bx + c, such that c < 0 and a – 2b + 4c > 0, then f(x) has
 1  1 
(A) One root in the interval  0,  (B) One root in the interval   ,0 
 2  2 
(C) both root are positive (D) None of these

(III)

Read the following write up carefully and answer the following questions:
 and  are the roots of the equation ax2 + bx + c = 0 and 4, 4 are the roots of the equation
lx2 + mx + n = 0 (,  are real and distinct). Let f (x) = a2lx2  4aclx + 2c2l + a2m = 0, then

31. Roots of f (x) = 0 are


(A) real and same in sign (B) real and opposite in sign
(C) equal (D) data is insufficient

32. One root of the f (x) = 0 is


a2 b2
(A) 2 (B)
b a2
(C) b/a (D) a/b

33. If 3 + 3 = 0 (b  0), then


(A) a, b, c are in G.P. (B) 2a, b, c are in G.P.
(C) 3a, b, c are in G.P. (D) 4a, b, c are in G.P.

(IV)
Read the following write up carefully and answer the following questions:
n n1
Let a0x + a1x +  + an1x + an = 0 be the nth degree equation with a0, a1,  an integers. If p/q is
a rational root of this equation, then p is a divisor of an and q is a divisor of a0. If a0 = 1, then every
rational root of this equation must be an integer.

34. The roots of the equation x3  9x2 + 23x  15 = 0, if integers, are in


(A) A.P. (B) G.P.
(C) A.G.P. (D) none of these

P-2022-CBSE-P1-MATHEMATICS-QEE
51
3 2
35. At least one integral root of the equation x  13x + 15x + 189 = 0 exceeds another root
by
(A) 1 (B) 2
(C) 3 (D) none of these

Matrix Match

36. Match the following:


List I List II
2
(P) If ax + bx + 6 = 0 does not have two distinct real roots, (1) 4
then the least value of 3a + b is
(Q) The value of ‘p’ for which the sum of the square of the (2) 0
2
roots of 2x – 2(p –2)x – p –1= 0 is least, is
(R) If a, b, c are three distinct positive real numbers, then (3) 3/2
the number of real roots of ax2 + 2b|x| + c = 0 is
(S) If a and b are distinct real numbers and difference (4) 2
between the roots of x2 + ax + b = 0 and that of x2 + bx + a
= 0 is same then the value of a+ b is
Codes :
P Q R S
(A) 3 2 1 4
(B) 4 3 2 1
(C) 3 1 4 2
(D) 4 1 2 3


37. If ,  are the roots of the equation x2  px + r = 0 and , 2 are the roots of the
2
equation x2  qx + r = 0, then
Column  I Column II
(A) , 2 are the roots of the equation (p) 36x2  6(p + q)x + (2p q)(2q  p) = 0
(B) /2,  are the roots of the equation (q) 9x2  6(p + q)x + 4(2p  q) (2q  p) = 0
3 (r) 9x2  3(p + q)x + (2p  q) (2q  p) = 0
(C) , 3 are the roots of the
2
equation
2
  (s) x  (p + q)x + (2p  q) (2q  p) = 0
(D) , are the roots of the equation
4 2

38. Consider a, b, c  R – {0}. A real valued function f is defined as


y z x y z z x y z x x y z x y
f(x, y, z) = 2 3 a b c + 2 3 a b c + 2 3 a b c , where x, y, z  I.
Given that f(1, 0, 0) = 4, f(2, 0, 0) = 6, then
Column – I Column – II
(A) The possible value of [a] (where [.] denotes the greatest (p) 0
integer function) can be
1 (q) 1
(B) The integral values of can be
b
(C) If all possible values of c satisfying given condition is [p, q], (r) 2
then 4q – 3p is equal to
(D) The maximum value of [a]2 + [b]2 – [c] (where [.] denotes (s) 3
the greatest integer function) is
(t) 5

P-2022-CBSE-P1-MATHEMATICS- QEE
52
39. Matching List Type(3 Column & 4 Rows)
Column – 1 Column – 2 Column – 3
(I) y = |x|2 – 2|x| – 3 (i) Df = (–,–1] [3, ) (P) Ranges = [–4, )
2
(II) y = |x – 2x – 3| (ii) Df = R (Q) Range = R
2
(III) y = x – 2x – 3 (iii) Df = [–4, ) (R) Range = [0, )
2
(IV) y = ||x| – 2|x| – 3| (iv) Df = [0, ) (S) Range = (–, –1] [3,
)
(V) |y| = x2 – 2x – 3 (v) Df = (–, –1)  (3, ) (T) Range = (–4, )
(I). Which of the following is correct combination
(A) (V) (i) (Q) (B) (V) (v) (T)
(C) (V) (ii) (Q) (D) (V) (ii) (R)
(II). Which of the following is correct combination
(A) (IV) (ii) (P) (B) (IV) (ii) (R)
(C) (IV) (ii) (Q) (D) (IV) (ii) (T)
(III). Which of the following is not correct combination
(A) (I) (ii) (P) (B) (II) (ii) (R)
(C) (III) (ii) (P) (D) (I) (iii) (R)

Numerical Based
k
40. Find the possible value of (k – 5) for which the equation  (x  m  1) (x + m) = 10k has
m 1
solution  and  + 1 for some .

x3
41. Number of points with integral coordinates on the curve y = is __________
x2  x

Decimal Type

42. Find the value of k such that the equation |x2 – 5 |x| + 6| = k is satisfied by six distinct
values of x.

43. Find the least integral value of a such that ax 2  12x  3x 2  6  a  0  x  R.

P-2022-CBSE-P1-MATHEMATICS-QEE
53
ANSWERS TO CHAPTER PRACTICE PROBLEMS

Subjective:

Level – I

1. acx2  bx + 1 = 0

2. ( 4, 7)

3. a   2,8

Level – II

4. a+b=c

Objective:

MCQ Single Correct

1. A 2. D 3. A 4. A
5. C 6. C 7. D 8 D
9 A

MCQ Multi Correct

1. A, B 2. A, D 3. A, C, D 4. A, D

Numerical Based

1. 2 2. 4 3. 2 4. 6

5. 4 6. 1 7. 4 8. 4

9. 1

P-2022-CBSE-P1-MATHEMATICS- QEE
54
ANSWERS TO ASSIGNMENT PROBLEMS

Subjective:

Level – 0

 5 5 q p pq 2
1. (i) (ii) ,  (iii) ,
10 p q pq pq
4 9
(iv) p + q – r, p – q + r (v) –1, 2 (vi) ,
13 13
3i 7
(vii) –1, 8 (viii) 2  2, 1  i (ix) 1, 2,
2

(x) a,

a 1  17 
2

1 1
2. (a) (i)  , 1 (ii) (c) m = 2, n = 1
2 3

3. 16 4. x =  5, 6

2
5. (i) c = a (ii) mb2  ac 1  m  (iii) ac  a  c   b3  2abc
1 1

 
(iv) an c n 1
 
 ac n n1 b  0 (v) mnb2  ac(m  n)2

7. a = 0, 24 8. x1  8, x2  6
9. 9
10.  cr  ap 2  (aq  br)(bp  cq) . 13. a = 4, b = 12

15. (0, 2) 16. 2

17. sin2 . cos2x2  x + 1 = 0 18. [ 4, 4]

19. (, 5) 20. 2x2

21. −1, 0, 1, 2 22. at least two real roots

23. 1 24. a > 11/9

25. a1/2, a4/3

Level – I

2. a = – 2, 0 roots are 0, 3 and – 1, 2


 5
3. 2n + <  < 2n + and   (4n + 1)/2, n  I
6 6

P-2022-CBSE-P1-MATHEMATICS-QEE
55
2
4. –3, –10 7. x – 2x + 3 = 0
4
11. 0 12.  <p<2
3

Level – II

1. (i) (a) m  (–, 1) (9, ) (b) m  {1, 9} (c) m  (1, 9)


(d) m < 0 (e) m   (f) m  [9, )
(g) m  (0, 1] (h) m  (–, 0) [9, )
(ii) (a) m > 10 (b) m  [9, 10) (c) m  (–, 1]
(d) m > 10 (e) m   (f) m  (10, )
(g) m   (h) m  [10, )

 1 
2.  , 1 4. a  (–, –1/2)
 4 

5. 4, 6, 4

9. x=1 10. m = 9/8

Objective:

Level – I

1. D 2. A 3. B 4. C
5. B 6. C 7. B 8. D
9. D 10. C 11. C 12. B
13. A 14. A 15. B 16. B
17. C 18. C 19. C 20. C
21. B

Level – II

1. C 2. B 3. C 4. C
5. D 6. C 7. A 8. D
9. C 10. D 11. B 12. A
13. B 14. B 15. A 16. A, B
17. B, C 18. A, B, C, D 19. B, C, D 20. B, C, D
21. B, C 22. A, B, C 23. A, B, C 24. C
25. A 26. A 27. C 28. B
29. B 30. B 31. B 32. B
33. C 34. A 35. B 36. B
37. (A)  (q); (B)  (r); (C)  (s); (D)  (p)
38. (A)  (p, q, r); (B)  (q, r, s); (C)  (r); (D)  (t)
39. (I). A (II). B (III). D
40. 6 41. 2 42. 0.25 43. 7.00

P-2022-CBSE-P1-MATHEMATICS- QEE
PINNACLE-CBSE

SETS, RELATIONS & FUNCTIONS


CONTENTS SYLLABUS

 Set 1
CBSE: Sets and their representations finite and
 Operation on sets 3
infinite sets, Equal sets, Subsets of the set of real
 Applications 6 numbers especially intervals with notations,
Power set, Universal set, Venn diagrams,
 Exercise 1 8 Complement of a set. Operations on sets (union,
intersection and difference of two sets).
 Relation 8 Applications of sets. Ordered pairs, Cartesian
product of sets. Number of elements in the
 Function 9
Cartesian product of two finite sets, Relation,
 Exercise 2 9 pictorial diagrams, domain, co-domain and range
of a relation. Real valued function of the real
 Definitions of function, domain variable, domain and range of these functions,
constant, identity, polynomial, rational, modulus,
and range 9
signum and greatest integer function and their
 Exercise 3 11 graphs. Sum, difference, product and quotient of
functions.
 Graphs of some elementary
Functions 12

 Algebra of Functions 13

 Answers to Exercises 14

 Solved Problems 15

 Chapter Practice Problem 22

 Assignment Problems 25

 Answers to Chapter 36
Practice Problems
 Answers to Assignment 37
Problems
[[[ [

Set:
A set is a collection of well defined objects i.e. the objects follow a given rule or rules. If we say
that we have a collection of short students in a class, then this collection is not a set as “short
students” is not well defined. If, however, we say that we have a collection of students whose
height is less then 5 feet, then it represents a set.

Elements of a set:
The members of a set are called its elements. A set is usually denoted by capital letters A, B, C
etc, where as the elements of a set are generally denoted by lower case letters a, b, c, d etc. If an
element x is in set A, we say that x belongs to A and write x  A. If the element x is not in A then
we write x  A.

Examples of sets:
1. The set of vowels in the alphabet of English language.
2. The set of all points on a particular line.
3. The set of all lines in a particular plane.
4. The set of all odd natural numbers.
5. The set of all real numbers.
The elements in a set can be written in any order e.g.
A = {1, 3, 5, 7, 9, …}, B = {11, 9, 5, 7}, C = {…,  2,  1, 0, 1, 2, …}, D = {Amar, Aman, Ajay}.
This is called the roster method of representing a set. A set can also be represented by stating
the properties within braces, which are satisfied by the elements of the set e.g.
A = {x: x = 2n + 1, n  1, n  N}, A = {x: 6  x  12, x  N}.
This method of representing a set is called the set builder method.

Illustrations 1. Rewrite the following sets by the set builder method:


1 1 1 1 
(i) 1, 3, 5, 7, 9 , (ii)  , , , .
 9 25 49 81 

Solution: (i) The general term, here, is 2n + 1. For n = 0, 1, 2, 3, 4, we get the elements of
the set. Hence, we write the given set as
{x: x = 2n + 1, n  N + {0} 0  n  4}.
1
(ii) The general term is . For n  1, 2, 3, 4 we get the elements of the
 2n  12
 1 
 
set. Hence, we write the given set as  x : x  , n  N, n  4  .
  2n  12 

Illustration 2. Write the following sets in the roster form:


(i) {x : x2  5x + 6 = 0, x  N} (ii) {x : 2x  5 < 4, x  Z}

Solution: (i) Solving equation x 2  5x  6  0 , we have x  3, 2 . Since 2, 3 are in N, we


write the set as {2, 3}.
(ii) Here 2x  5  4  2x  9  x  9 / 2 and since x  Z, x  4, 3, 2,...
Hence the required set is ..., 2,  1, 0, 1, 2 ,3 ,4 .

P-2022-CBSE-P1-MATHEMATICS-SRF
2
Some special sets:
(i) Finite and infinite sets:
A set A is finite if it contains only a finite number of elements; we can find the exact number of
elements in the set. Otherwise, the set is said to be an infinite set; e.g. The set 1, 3, 5, 7, 9 is a
finite set, having five elements. The set of letters in the word MISSISSIPI is a finite set containing
four letters M, I, S, P , although there are, in all, eleven letters in the word; only distinct elements
are considered in a set. Other examples are:
(1) The set of all odd natural numbers is an infinite set.
(2) The set of all points on a particular straight line is an infinite set.
(3) The following sets are all infinite sets:
N = set of all natural numbers = 1, 2, 3, ...
Z = set of all integers = ..., 2,  1, 0, 1, 2,...
p 
Q = set of all rational numbers =  :p, q  Z,q  0 
q 
R = set of all real numbers = {x : x is a rational and an irrational number}
C = set of all complex numbers =  x  iy ; x, y R

(ii) Null set:


A set which does not contain any element is called a null set and is denoted by  . A null set is
also called an empty set.

(iii) Singleton set:


A set which contains only one element is called a singleton set.

(iv) Equal sets:


Two set are said to be equal, if every element of one set is in the other set and vice-versa e.g.
A  p, q, r, s , B  r, q, p, s are equal sets, since the order of the elements is immaterial. Also
A  x : 10  x  14, x  N and B  14, 13, 12, 11, 10 are equal sets. In these cases we write
A = B. But A  1, 4, 6 , and B  1, 6, 4, 3 are not equal sets as 3  A , and we write A  B .

(v) Equivalent sets:


Two sets A and B are equivalent if the elements of A can be paired with the elements of B, so
that to each element of A there corresponds exactly one element of B e.g. A  p, q, r ,
B  5, 6, 8 are equivalent sets.
Note: Equal sets are equivalent, but equivalent sets may not be equal.

(vi) Subsets:
If each element of a set A is also an element of a set B, then A is called a subset of B, and we
write A  B , e.g. A  a, k, l, o, u , B  a, b, c,......x, y, z . Here A  B . And if A  2, 4, 6,
B  1, 2, 3, 4, 5, 6, C  4, 6 , then A  B, C  A, C  B . Note that N  Z, N  Q, R  C .

P-2022-CBSE-P1-MATHEMATICS-SRF
3
(vii) Proper subsets:
A set A is called a proper subset of B if and only if each element of A is an element of B and there
is at least one element of B which is not in A i.e. A  B and A  B and we write A  B e.g. if
A  a, b, c, d , B  a, b, c, ... , x, y, z , then A  B , and if
A  2, 4, 6, , B  1, 2, 3, 4, 5, 6 , C  4, 6 , then A  B , C  A, C  B .

Note: The null set  is a subset of every set and every set is a subset of itself.

(viii) Power set:


The power set of a set A is the set of all of its subsets, and is denoted by P  A  e.g. if
A  4, 5, 6 then P  A   , 4 , 5 , 6 , 4, 5 , 5, 6 , 4, 5, 6 .

Note: The null set  and set A are always elements of P  A  .

Theorem: If a finite set has n elements, then the power set of A has 2n elements.

Proof: The number of subsets having no element  1  n C0 .


The number of subsets having exactly one element  n  n C1 .
The number of subset having 2 elements = number of groups of 2 elements out of n
 n C2 .
The number of subset having n element (A itself)  1  n Cn .
Hence, the number of elements in P  A 
 n C0  nC 1  n C2  ....  n Cn  2n .

Operations on sets:
The operations on sets, by which sets can be combined to produce
new sets, can be best illustrated through Venn diagram. A set S is
called a universal set if every set we consider, is a subset of S. In A
B
Venn diagram, the universal set is shown in the form of a rectangle
and the subsets of the universal set are generally shown as circles and AB
ellipses. If A is a proper subset of B i.e. A  B , we say that A is
contained in B.
(i) Union of sets:
The union of two set A and B is defined as the set of all elements AB

which are either in A or in B or in both. The union of two sets is A


written as A  B ; e.g. If A  30, 55, 62, , B  30, 66, 72 , then
B
A  B  30, 55, 62, 66, 72 .. Also if
A  x : x  N, 2  x  6 , B  x : x  N, 4  x  7 , then
A  B  x : x  N 2  x  7 .
This definition can be extended to the union of more than two set A1, A 2 ,..., An . We define, in this
case, the union as
n Ai  A1  A 2  A 3 ........  An x : x  A i for at least one i,1  i  n .
i 1

P-2022-CBSE-P1-MATHEMATICS- SRF
4
Note : A  A  A (i.e. union of sets is idempotent), A  S  S and A    A. Also
A  B  B  A and A   B  C    A  B   C .

Illustration 3. Find the union of the sets A  5, 7, 9 , B  9, 13, 15 and
C  13, 15, 17 . Also show that  A  B   C  A   B  C  .

Solution: We have A  B  5, 7, 9  9, 13, 15  5, 7, 9, 13, 15 and
B  C  9, 13, 15  13,15,17  9, 13, 15, 17
  A  B   C  5, 7, 9, 13, 15  13, 15,17  5, 7, 9, 13, 15, 17 and
A   B  C   5, 7, 9  9, 13, 15, 17  5, 7, 9, 13, 15, 17
  A  B  C .

(ii) Intersection of sets:


AB
The intersection of two sets A and B is defined as the set of
those elements which are in both A and B and is written as A B
A  B  x : x  A and x  B ; e.g. if A  a, b, c, d, e and
B  b, d, e , then A  B  b, d, e and if

A = {x : x  R, 0 < x < 3}, B = {x : x  R, 3 < x < 4}, A  B = .

Note: A  A  A, A  S  A and A     .
The commutative, associative and distributive laws hold for intersection of two sets i.e.
 A B  B  A
  A  B   C  A  B  C 
 A  B  C    A  B    A  C 
 A  B  C    A  B    A  C 
The intersection of n sets A1, A 2 ........An is written as
n
 A i  A1  A 2  A 3 .........  A n  x : x  A i for all i, 1  i  n .
i1

Illustration 4. Find the intersection of the sets


A  1, 2, 3, 4, 5, 6 , B  2, 4, 5 , C  2, 6 and show that
 A  B   C  A  B  C  .

Solution: We have A  B  1, 2, 3, 4, 5, 6  2, 4, 5  2, 4, 5 and


B  C  2, 4, 5  2, 6  2
  A  B   C  2, 4, 5  2, 6  2 and
A   B  C   1, 2,3, 4, 5, 6  2  2 .
Disjoint sets:
Two set A and B are said to be disjoint, if there is no element which is in both A and B, i.e.
A  B   ; e.g. A  a, b, c , B  d, e, f  are disjoint.

P-2022-CBSE-P1-MATHEMATICS-SRF
5
iii) Difference of sets: S A-B

The difference of two set A and B, taken in this order, is defined B


as the set of all those elements of A which are not in B and is A
denoted by A – B i.e. A  B =  x : x  A and x  B .
Similarly set B – A is the set of all those element of B which are
not in A i.e. B  A  x : x  B, x  A ; e.g. if A  a, b, c, d, e
and B  b, c, d, e, f  , then
A  B  a and B  A  f  .
A  B is also denoted by A\B.

iv) Complement of a set: S


Complement of a set A is defined as S – A where S is the
c
universal set and is denoted by A c or A  i.e. A c  S  A or A
A
A c   x : x  S, x  A .

c
Note: A 
c
 A, Sc  , A  A c   , A  A = S.
c

Illustration 5. For the sets S  10, 11, 12, 13, ...., 17, 18 , A  11, 13, 15 , B  12, 14, 16 ,
show that B c  Ac  A  B  A  B c .

Solution: We have A c  S  A  10, 12, 14, 16, 17, 18 , and


Bc  10, 11, 13, 15, 17, 18 so that Bc  A c  11, 13, 15 .
Also A  B  11, 13, 15  Bc  Ac .
Moreover, A  Bc  11, 13, 15  10, 11, 13, 15, 17, 18  11, 13, 15
= A  B  Bc  A c .

Note: The properties of the complement of sets are known as DeMorgan laws, which are
(i) A c  Bc  B  A
c
(ii)  A  B   A c  Bc
c
(iii)  A  B   A c  Bc
c
To prove (iii), we have  A  B    x : x   A  B   x : x  A or x  B

 
 x : x  A c or x  Bc  A c  Bc .

Note: A   B  C    A  B    A  C  ; A  B  C    A  B    A  C  .
A
A
A A – (B  C) A
A-C

A-B
(B  C)
B
B
B
B
C C
C C

P-2022-CBSE-P1-MATHEMATICS- SRF
6
A
(A – B)  (A – C)

B
C

Venn diagram for A   B  C    A  B    A  C  .

(v) Applications:
Let A be a finite set. The number of elements in A is denoted by n  A  . Let A and B be two finite
sets. If A and B are two disjoint sets, then n  A  B   n  A   n B  .
If A and B are not disjoint, then
(i) n  A  B   n  A   n B   n  A  B 
(ii) n  A  B   n  A  B   n B  A   n  A  B 
(iii) n  A   n  A  B   n  A  B 
(iv) n  B   n  B  A   n  A  B 

S A-B A B
B-A
S
A

B
A B

Illustration 6. Set A has 3 elements and set B has 6 elements. Find the maximum and
minimum number of elements in A  B .

Solution: We have n  A   3, n B   6 so that the maximum number of element in


A  B  3 . Also n  A  B   n  A   n B   n  A  B 
 minimum n  A  B   n  A   n B   maximum n  A  B   3  6  3  6 .
The number of elements in A  B is maximum when A and B are disjoint
 n  A  B   n  A   n B   3  6  9 .

Illustration 7. In a certain examination, the candidates can offer papers in English or Hindi or
both the subjects. The number of candidates who appeared in the examination is
1000 of whom 650 appeared in English and 200 both in English and Hindi. Find
the number of candidates who offered paper in
(i) Hindi, (ii) English only, (iii) Hindi only.

Solution: Let A = The set of candidates who offered paper in English


and B = The set of candidates who offered paper in Hindi. Hence
n  A  B   1000, n  A   650, n  A  B   200 and from
n  A  B   n  A   n B   n  A  B  , we have

P-2022-CBSE-P1-MATHEMATICS-SRF
7
1000  650  n B   200  n B   550 .
Also the number of candidates who offered paper in English only
 n  A  B   n candidates  A, and candidates  B
 n  A  B   n  A   n  A  B   650  200  450 .
Similarly, the set of candidates who offered paper in Hindi only is B – A
 n B  A   n B   n  A  B   550  200  350 .

(vi) Cartesian product of sets:


Let a be an arbitrary element of a given set A i.e. a  A and b be an arbitrary element of B i.e.
b  B . Then the pair  a, b  is an ordered pair. Obviously  a, b    b, a  . The cartesian product
of two sets A and B is defined as the set of ordered pairs  a, b  . The cartesian product is
denoted by A  B
 A  B   a, b  ; a  A, b  B .
In general A  B  B  A and if A or B is a null set then A  B   .
Moreover, n  A  B   n  A   n B  .

Note: (i) A  B  C    A  B    A  C 
(ii) A   B  C    A  B    A  C 
(iii) A   B  C    A  B    A  C 
(iv)  A  B   C   A  C   B  C 
(v)  A  B   C   A  C   B  C 
(vi)  A  B   C   A  C   B  C 

Illustration 8. If A  2, 5 , B  3, 4, 7  and C  3, 4, 8 then evaluate


A  B, B  A, A  A and verify that
(i) A   B  C    A  B    A  C  ,
(ii) A   B  C    A  B    A  C  .

Solution: Here A  B  2, 5  3, 4, 7   2, 3  ,  2, 4  ,  2, 7  ,  5, 3  ,  5, 4  ,  5, 7  ,


B  A  3, 4, 7  2, 5   3, 2  ,  3, 5  ,  4, 2  ,  4, 5  ,  7, 2  ,  7, 5  and
A  A  2, 5  2, 5   2, 2  ,  2, 5  ,  5, 2  ,  5, 5  .
Also A  C  2, 5  3, 4, 8   2, 3  ,  2, 4  ,  2, 8  ,  5, 3  ,  5, 4  ,  5, 8  ,
B  C  3, 4, 7  3, 4, 8  7
 A  (B  C) = {2, 5}  {7} = {(2, 7) (5, 7)},
A  B   A  C    2, 3  ,  2, 4  ,  2, 7  ,  5, 3  ,  5, 4  ,  5, 7 
  2, 3  ,  2, 4  ,  2, 8  ,  5, 3  ,  5, 4  ,  5, 8    2, 7  , 5, 7  A  B  C  .
To verify (ii), we write B  C  3, 4, 7, 8
 A   B  C   2, 5  3, 4, 7, 8
  2, 3  ,  2, 4  ,  2, 7  ,  2,8  ,  5, 3  ,  5, 4  ,  5, 7  ,  5, 8  , and

P-2022-CBSE-P1-MATHEMATICS- SRF
8
 A  B    A  C    2, 3  ,  2, 4  ,  2, 7  ,  2, 8  ,  5, 3  ,  5, 4  ,  5, 7  , 5, 8 
 A  B  C  .

Exercise 1.
i) Prove by using venn diagram that ( A  B )  A  B .
ii) If A  a, e, i , o, u , B   b, c, d  , then find  A  B   B .
iii) If S  a, b, c, d , e, . f , g , A  a, b, c, d  , B  c, d , e, f  , then verify that
c
 A  B  Ac  B c .
iv) If A = {1, 2, 3, 4} and B = {2, 3, 5}, prove that A   B  A    .
v) If A  2 , B  5 , C  3, 4, 6 , then verify that A   B  C    A  B    A  C  .
vi) In a conference attended by 550 delegates tea and coffee are served. The number
of delegates who drink tea is 400 and the number of delegates who drink tea only
is 200. Find (a) how many of them drink coffee,
(b) how many drink coffee only.

Relation:
Let A and B be two sets. A relation R from the set A to set B is a subset of the cartesian product
A  B . Further, if  x, y   R , then we say that x is R-related to y and write this relation as x R y.
Hence R  x, y  ; x  A, y  B, x R y .
As an example, consider A  1, 2, 3 and B  1, 8, 27 , so that
A  B  1, 1 , 1, 8  , 1, 27  ,  2, 1 ,  2, 8  ,  2, 27  , 3, 1 ,  3, 8  ,  3, 27  .
Consider now a subset R of A  B , as R  1, 1 ,  2, 8  ,  3, 27  .

We notice that in every ordered pair of R, the second element is the cube of the first element i.e.
the element of the ordered pairs of R have a common relation- ship which is “cube”.
In case we take A  2, 4, 6 , B  1, 5 ,
then A  B   2, 1 ,  2, 5  ,  4, 1 ,  4, 5  ,  6, 1 ,  6, 5  .

Consider now a subset R of A  B as R   2, 1 ,  4, 1 ,  6, 1 ,  6, 5  .

Here the first element in each of the ordered pair is greater than the second element. Hence the
relationship is “greater than”. Obviously, from the definition, x R y and y R x are not the same,
since R   x, y  : x  A, y  B, x R y and R   x, y  : x  B, y  A, x R y are different.

Domain and Range of a relation: Let R be a relation defined from a set A to a set B,
i.e. R  A  B . Then the set of all first elements of the ordered pairs in R is called the domain of
R. The set of all second elements of the ordered pairs in R is called the range of R. That is,
D = domain of R  x :  x, y   R or  x : x  A and  x, y   R ,

R = range of R  y :  x, y   R or  y : y  B and  x, y   R .

Clearly D  A and R*  B .
For example, for R given in 1, 1 ,  2, 8  ,  3, 27  above, domain of R  1, 2, 3 ,

Range of R  1, 8, 27 .

P-2022-CBSE-P1-MATHEMATICS-SRF
9
Function:
Let A and B be two non-empty sets. Let to each element of A, there correspond exactly one
element of B. This correspondence between the elements of A and B is called a function from A
to B. Function is a special case of a relation, since a relation may relate an element of A to more
than one elements in B. A function from A to B is usually denoted by the symbols f, g etc. and we
write f : A  B . We also say that “f is a mapping from A to B”.
The set A is called the domain of the function f and
B is called the co domain of the function f. f
a
b

A B

Let an element a  A correspond to b  B under the function f. Then we say that b is the image
of a under f and a is the pre-image of b. We then write f  a   b . By definition, if f is a function
from A to B, then each element of A has unique image in B. However, every element of B
needs not be an image of some a in A. The subset of B (co domain) which contains all the
images of the elements of A is called the range of the function and is denoted by f  A 
i.e. f  A   f  a  : a  A .

Exercise 2.
i) Let R  1, 3  ,  2, 5  ,  3, 7  ,  4, 9  ,  5, 11  be a relation in the set
A  1, 2, 3, ...... . Find the domain and range of R.

ii) Find the domain and range of the relation R   x , x  : x  4,


2
x N . 
2
3x
iii) Determine domain of the function: f  x   ,
2x
x2  1
(iv) Determine domain of the function: f  x   .
x 1

Definition of Function, Domain, Co-domain and Range:


We note that, function is a rule (or a set of rules) which associates to each possible real number
x, a unique real number y, written as f (x). Function can be easily defined with the help of the
concept of mapping. Let X and Y be any two non-empty sets. “A function from X to Y is a rule or
correspondence that assigns to each element of set X, one and only one element of set Y”. Let
the correspondence be ‘f’. Then mathematically we write f : X Y where y = f(x), x X and y Y.
We say that ‘y’ is the image of ‘x’ under ‘f’(or x is the pre image of y). Accordingly
(i) A mapping f: X  Y is said to be a function if each element of X has it’s image in the set
Y. It is possible that a few elements in Y are present which are not the images of any
element in the set X.
(ii) Every element of X should have one and only one image. That means it is impossible to
have more than one images for a specific element of X. Functions can’t be multi-valued
(A mapping that is multi-valued is called a relation from X to Y)
 The set ‘X’ is called the domain of ‘f’.
 The set ‘Y’ is called the co-domain of ‘f’.

P-2022-CBSE-P1-MATHEMATICS- SRF
10
 Set of images of different elements of the set X is called the range of ‘f’. It is obvious
that range could be a subset of the co-domain as we may have some elements in the
co-domain which are not the images of any element of X (of course, these elements of
the co-domain will not be included in the range). Range is also called domain
of variation.
 If R is the set of real numbers and X and Y are subsets of R, then the function
f (x) is called a realvalued function or a real function.
Domain of a function ‘f’ is normally represented as Domain (f). Range is represented as
Range (f). Note that some times domain of the function is not explicitly defined. In these cases
domain would mean the set of values of ‘x’ for which f (x) assumes real values that is if y = f (x)
then Domain (f) = {x: f (x) is a real number}.
e.g. Let X = {a, b, c}, Y = {x, y, z}. Suppose f(a) = y, f(a) = x, f(b) = y, f(c) = z. Then f is not a
function of X into Y since a  X has more than one f-images in Y.
On the other hand, if we set f(a) = x, f(b) = x and f(c) = x, then f:X  Y is a function since each
element in X has exactly one f-image in Y.
Consider the following examples:
(i) Let X = R, Y = R and y = f(x) = x2.
Then f : X  Y is a function since each element in X has exactly one f - image in Y. The
range of f = {f(x) : x  X} = {x2 : x  R} = [0, ).
(ii) Let X = R+, Y = R+ and y = x. Then f : X  Y is a function. The range of f is R+
(iii) Let X = R, Y= R and y2 = x. Here f(x) =  x i.e. f is not a function of X into Y since each
x > 0 has two f-images in Y, and further, each x<0 has no f-image in Y.
We are primarily interested in functions whose domains and ranges are (sub) sets of real
numbers.
1
e.g. Let the function f be defined by f(x) = .
2x  6
In this case we must have 2x + 6 > 0  x >  3. Therefore, the domain of f is (–3, ), the range of
1
f = (0, ). Thus we have the function f : (–3, )  (0, ) defined by f(x) = .
2x  6
x
Let the function f be defined by f(x) = . The function makes sense for all values of x
 x  1 x  2 
except x = 1 and x = 2. Therefore, the domain of f is R – {1, 2}.

Illustration 9. Find the domain and range of the function f  x   x 5 .

Solution: The function f  x   x  5 is defined


for x  5  the domain is  5,   .
Also, for any x  a  5, f  a   a  5  0
 the range of the function is  0,   .

Illustration 10. Find the domain and the range of the function y = f (x), where f (x) is given by
2
(i) x  2x  3, (ii) x 2  2x  3 , (iii) sin x,
(iv) tan x, (v) log10 (x).

Solution: (i) Here y = (x  3) (x + 1). The function is defined for all real values of x
2
 its domain is R. Also x  2x  3  y = 0 for real x

P-2022-CBSE-P1-MATHEMATICS-SRF
11
 4 + 4 (3 + y)  0   4  y < .
Hence the range of the given function is [ 4, ).
(ii) Here y = (x  3)(x  1)  (x  3) (x + 1)  0 so that x  3 or x   1.
Hence the domain is R  ( 1, 3) or ( ,  1]  [3, ).
Since f(x) is nonnegative in the domain, the range of f(x) is the interval [0, ).
(iii) The function f(x) = sin x, (x in radians) is defined for all real values of x
 domain of f(x) is R. Also  1  sin x  1, for all x,
so that the range of f (x) is [ 1, 1].
sin x
(iv) The function f(x) = tan x = is not defined when
cos x

cos x = 0, or x = (2n + 1) , n = 0,  1,  2, …
2
  
Hence domain of tan x is R  (2n  1) , n  0,  1,  2,.... , and its range is R.
 2 
(v) The function f (x) = log10 x is defined for all x > 0. Hence its domain is (0, ) and
range is R.
Illustration 11. Find the domain of definition of the following functions:
 x  1  x  2 
(i) f(x) = log 1  2x  3  , (ii) f(x) =
2
 x  3  x  4 
Solution: (i) For f(x) to be defined log1/2(2x – 3)  0
 2x – 3  1  x  2. . . . (1)
3
Also 2x – 3 > 0  x > . . . . (2)
2
Combining (1) and (2) we get the required values of x. Hence the domain of
3 
definition of f(x) is the set  , 2
 2 
 x  1 x  2 
(ii) For f(x) to be defined  0 and x  3, 4.
 x  3  x  4 
By wavy - curve method the domain of definition of f(x) is the set
x  (– , – 2]  [1, 3)  (4, ).

Exercise 3.
i) Find the domains of the functions:
x 1 1
(a) , (b) , (c) x 1  3  x ,
x 2 2x  3
x2 1
(d) , (e)
1  x2 | x | x
ii) Find the domain of the function f(x) = loge(x – 2).
1
(iii) Prove exist when x < 0.
x x
(iv) Prove that the domain of f(x) = ( 9  x 2 )  ( x 2  4 ) is [3, 2]  [2, 3].

P-2022-CBSE-P1-MATHEMATICS- SRF
12
Graphs of Some Elementary Functions
A convenient and useful method for studying a function is to study it through its graph. To draw
the graph of a function f : X  Y, we choose a system of coordinate axes in the plane such that
to each x  X, there corresponds the ordered pair (x, f(x)) which determines a point in the plane.
The set of all points {(x, f(x)) : x  X} is the graph of f. We consider some examples of functions
and their graphs.

FUNCTION DOMAIN, RANGE GRAPH


AND DEFINITION
1. The constant function A function f : A  B, A, B  R, y
 The graph of y = f (x) is a is said to be a constant
y=c
line parallel to the xaxis. function if there exists a real
 If c > 0, the line is above number c such that f (x) = c for x
O
the xaxis. all x  A.
 If c = 0, the line coincides Domain : A
with the xaxis Range : {c}
 If c < 0, the line is below
the xaxis.
2. The identity function The function from R  R that
 The graph of y = f (x) = x is associates to each x  R the y
y=x
a straight line with slope 1. same x, is called the identity
 The line passes through function. More precisely
the origin f (x) = x x
O
Domain : R
Range : R
3. The absolute value (or The function f : R  R+ + {0} y
modulus) function. defined by
 The graph of y= f (x) = |x| is f (x) = |x| = x if x  0
y = -x y= x
symmetric about the y  x if x < 0 x
O
axis. is called the absolute value
 It passes through the origin function
and remains above the x Domain : R
axis. Range : [0, ).
y
5. The Greatest Integer The function f : R  Z defined
Function by f(x) = [x], x  R 4
3
 [x] denotes the greatest is called the greatest integer 2
integer less than or equal function 1
to x. In particular x  1 < [x] Domain : R -4 -3 -2 -1 O 1 2 3 4
x
-1
 x. It is also called a step Range : Z.
-2
function. -3
 The graph of y = [x] -4

consists of infinitely many


broken pieces, in the first
and the third quadrant,
each piece parallel to the
xaxis.
6. The fractional part function The function f : R  [0,
If x = [x] + {x}, 0  {x} < 1, 1) defined by f(x) = {x}
then {x} is the fractional part is called the fractional
of x. part function.
 The graph of y = {x} Domain : R

P-2022-CBSE-P1-MATHEMATICS-SRF
13
consists of infinitely many Range: [0, 1) y

broken pieces, each piece y=1

parallel to the line y = x,


-2 -1 0 1 2 3 x
varying from y = 0 to y = 1.

7. Polynomial f(x) = a0xn + a1xn - 1 + .... +an - 1x + an


Function where a0, a1,....,an are real numbers, a0  0.
Domain : R
Range : Depends on the polynomial representing the
function.
8. Rational p  x
Function f(x) = , where p(x) and q(x) are polynomials in x.
q x
Domain is R – {x : q(x) = 0}, Range depends on the
function.
9. Signum Function The function F : R  R
defined by y
1 x  0 y=1

f(x) = 0 x 0 
 1 x0
  O
is called the signum function.
Domain: R, Range: {1, 0, 1} y = 1 

Algebra of Functions:
Given functions f : D  R and g : D  R, we define functions f + g, f - g, gf and f/g as follows:
 f + g : D  R is a function defined by (f + g)(x) = f(x) + g(x) for all x  D
 f – g : D  R is a function defined by (f – g) (x) = f(x) – g(x) for all x  D
 fg : D  R is a function defined by (f g) (x) = f(x) g(x) for all x  D
f  x
 f/g:CR is a function defined by (f/g) (x) = , g  x   0 ,where C={xD:g(x)  0}. i.e.
g  x
x  D  {x : g (x) = 0}.
 If k is any scalar, then kf is a function from DR defined by (kg)(x) = kf(x), xD.

Illustration 12. Let f : [–1, 1]  R and g : R  R be two functions defined by


f(x) = 1  x 2 and g  x   x 3  1 . Find the functions f + g, f – g, fg and f/g.

3 3
Solution: (f + g) (x) = f(x) + g(x) = 1  x2 + x + 1, (f  g) (x)= f(x)  g(x) = 1  x2 x  1,
f f x 1  x2
(fg) (x) = f(x) g(x) = (x3 + 1) 1  x2 and    x    3 , x  -1 .
 g gx x 1
The domain of each of f + g. f - g and fg is [-1, 1] and that of f/g is
{x  [1, 1] : g(x)  0} = (1, 1].

Note: The sum f + g, the difference f  g, the product fg and the quotient f/g are defined only
when f and g are real functions having the same domain. In case f and g, have different domains,
these operations are defined for those values of x which are common to the domains of both f
and g i.e. x  to the intersection of the domains of f and g.

P-2022-CBSE-P1-MATHEMATICS- SRF
14
ANSWERS TO EXERCISES

Exercise 1.
(ii)  (vi) (a) 350 (b) 150

Exercise 2.
(i) Domain = {1, 2, 3, 4, 5}, range = {3, 5, 7, 9, 11}

(ii) Domain = {1, 2, 3, 4}, range = {1, 4, 9, 16}

(iii) domain = R – {2}


(iv) domain = R – (1)

Exercise 3.
 3 
(i) (a) R  {2} (b)   ,  
 2 
(c) [ 1, 3] (d) R
(e) (, 0)

(ii) x  (2, )

P-2022-CBSE-P1-MATHEMATICS-SRF
15
SOLVED PROBLEMS

Subjective:

Level – 0

Problem 1. If A is the null set, find the number of elements in the power set P  P( A)  .

Solution: Here A    P(A)  P()   so that P P( )   ,  .

Problem 2. Find the subsets that can be formed from the set A  4, 5, 6 and the number
of subsets.

Solution: The subsets that can be formed from the given set
are , 4 ,5 , 6 , 4, 5 , 4, 6 , 5, 6 , 4, 5, 6 . These are 8 subsets.

Problem 3. Of the 20 teachers of mathematic and physics, 12 teach mathematic and 4 teach
both the subjects. Find the number of teachers teaching physic.

Solution: Here n M  P   20, n M  12 and n M  P   4 . Hence


n  P   n M  P   n M  n M  P 
= 20 – 12 + 4 = 12.

Problem 4. X, Y are two sets and X has 40 elements, X  Y has 60 elements and X  Y
has 10 elements. Find the number of elements in Y is

Solution: n  X  Y   10 and n  X   40 . x = 40, x  y = 10

Also n  X  Y   40  10  30 .
 n  Y   60  30  30

Level – I

x2  x  1
Problem 5. Find the domain of the following function f(x) =
x2  4x  3

x2  x  1
Solution: Here f(x) = and f(x) is defined for all x  R other than numbers,
x2  4x  3
where
x2 + 4x + 3 = 0  x = -3, -1.
Hence the domain of f(x) = R - {-3, -1}.

Problem 6. Let f(x) = 2cos2x + 3 sin2x + 1. Find its range.

   
Solution: f(x) = 2cos2x + 3 sin2x + 1 = 2sin  2x   + 2 or 2cos  2x   + 2.
 6  3
So range is from [0, 4].

P-2022-CBSE-P1-MATHEMATICS- SRF
16
Problem 7. Prove by venn diagram that  A  B    A  B  is equal to  A  B    B  A  .

Solution: Here A  B is AB

A
B

AB
and A  B is

A
B

so that  A  B    A  B  is

A
B

which is same as  A  B    B  A  .

Problem 8. From 50 students taking examinations in mathematics, physics and chemistry, 37


passed mathematics, 24 physics and 43 chemistry. At most 19 passed
mathematics and physics, at most 29 mathematics and chemistry and at most 20
physics and chemistry. Find the largest possible number that could have passed
all three exams.

Solution: n(M  P  C) = 50
n(M) = 37, n(P) = 24, n(C) = 43
n(M  P)  19, n(M  C)  29, n(P  C)  20
n(M  P  C) = n(P) + n(C) + n(M) – n(P  C) – n(C  M) – n(M  P) + n(P  C
 M)
 50  37 + 24 + 43 – 19 – 29 – 20 + n(P  C  M).
 n(P  C  M)  14.

Problem 9. Find the domain of function f(x) = x 4  9 x .

Solution: Given f(x) = x 4  9 x


Since f(x) is defined when |x| − 4  0 and 9 − |x|  0
 4  |x|  9  –9  x  – 4 and 4  x  9
 Required domain of f(x) = [–9, –4]  [4, 9].

x2  2x  1
Problem 10. Find the range of the function g(x) = .
x

x2  2x  1
Solution: Let =y
x
2
 x + (2 − y) x + 1 = 0
Since x  R
(2 − y)2 − 4  0
 y2 − 4y  0
i.e. y  R − (0, 4) = range

P-2022-CBSE-P1-MATHEMATICS-SRF
17
Objective:

Level – 0

True / False

Problem 1. In a town of 10,000 families it was found that 40% families buy newspaper A,
20% families buy newspaper B and 10% families buy newspaper C, 5% families
buy A and B, 3% buy B and C, 4% buy A and C. If 2% families buy all the three
newspapers, then the number of families which buy A only is 3300.

Solution: True
We have, N = 10,000, n(A) = 40% of 10,000 = 4000,
n(B) = 2000, n(C) = 1000, n(A  B) = 500, n(B  C) = 300,
n(C  A) = 400, n(A  B  C) = 200.
 n ( A  B  C ) = n(A  (B  C)) = n(A)  n(A  (B  C))
= n(A)  n((A  B)  (A  C))
= n(A)  {n(A  B) + n(A  C)  n(A  B  C)}
= 4000  (500 + 400  200) = 3300.

Problem 2. A has 3 elements and B has 6 elements such that A  B. The number of
elements in A  B is 6.

Solution: False
A  B  A  B = A  n(A  B) = n(A) = 3.

Fill in the Blanks

Problem 3. The number of proper subsets of the set {1, 2, 3} is _________.

Solution: Number of proper subset = 23 – 1 = 7.

Problem 4. Let A and B have 3 and 6 elements respectively. The minimum number of
elements in A  B is ______

Solution: 6
A  B will contain minimum number of elements if A  B and in this case
n(A  B) = n(B) = 6.

Problem 5. 20 teacher of a school either teach mathematics or physics. 12 of them teach


mathematics while 4 teach both the subjects, then the number of teachers
teaching only physics is ______

Solution: 8
n(M  P) = 20, n(M) = 12, n(M  P) = 4
n(M  P) = n(M) + n(P) – n(M  P)
 n(P) = 12
 n(P) only = n(P) – n(M  P) = 8.

P-2022-CBSE-P1-MATHEMATICS- SRF
18
Level – I

MCQ Single Correct

Problem 6. Which of the following is an empty set?


(A) The set of prime numbers which are even.
2(2 x  3 ) 2
(B) The solution set of the equation   3  0, x  R.
x 1 x 1
(C) (A × B)  (B × A), where A and B are disjoint.
2
(D) The set of reals which satisfy x + ix + i  1 = 0.

Solution: If A and B are disjoint, then (x, y)  A × B  (y, x)  B × A and x is different


from y. Thus it is not possible (x, y) belongs to both simultaneously A × B and
B × A.

Problem 7. If X and Y are two sets, then X  (Y  X) equals


(A) X (B) Y
(C)  (D) none of these

Solution: X  (Y  X) = X  (Y   X) = X  (X  Y)


= (X  X)  Y =   Y = .

Problem 8. Let F1 be the set of all parallelograms, F2 the set of rectangles, F3 the set of
rhombuses, F4 the set of squares and F5 the set of trapeziums in a plane, then F1
is equal to
(A) F2  F3 (B) F2  F3  F4  F1
(C) F3  F4 (D) none of these

Solution:  Every rectangle, square and rhombus is a parallelogram


 F1 = F2  F3  F4  F1.

Problem 9. A and B are two sets having 3 and 4 elements respectively and having 2
elements in common. The number of relations which can be defined from A to B
is
(A) 25 (B) 210 – 1
12
(C) 2 – 1 (D) none of these

Solution: n(A  B) = 12
Hence the number of subsets of A  B is 212.

Problem 10. If A, B, C are any three sets, then A × (B  C) is equal to


(A) (A × B)  (A × C) (B) (A × B)  (A × C)
(C) A × B  A × C (D) none of these

Solution: A × (B  C) = (A × B)  (A × C), distributive law.

Problem 11. A survey shows that 63% of the Americans like cheese where as 76% like
apples. If x% of the Americans like both cheese and apples then
(A) x = 39 (B) x = 63
(C) 39  x  63 (D) none of these

Solution: n(C) = 63, n(A) = 76, n(A  C)  100


 n(A  C) = n(A) + n(C) – n(A  C)
 n(A  C) = 76 + 63 – n(A  C)  100  139 – n(A  C)

P-2022-CBSE-P1-MATHEMATICS-SRF
19
 n(A  C)  39 …(1)
But A  B  A and A  B  B
 n(A  B)  n(A) and A  B  B
 n(A  B)  63 and n(A  B)  n(B)
 n(A  B)  63 …(2)
from (1) and (2)
39  n(A  B)  63.

Problem 12. Number of relations that can be defined on the set A = {a, b, c, d, e} is
(A) 24 (B) 16
4 25
(C) 4 (D) 2
25
Solution: Since n(A) = 5, therefore, n(A × A) = 25. Hence number of subsets of A × A = 2 .
25
So, number of relations on A = 2 .

Problem 13. If f = {(1, 4), (2, 5), (3, 6)} and g = {(4, 8), (5, 7), (6, 9)}, then gof is
(A) { } (B) {(1, 8), (2, 7), (3, 9)}
(C) {(1, 7), (2, 8), (3, 9)} (D) none of these

Solution: (gof ) (1) = g(f (1)) = g(4) = 8,


(gof ) (2) = g(f (2)) = g(5) = 7
and (gof ) (3) = g(f (3)) = g(6) = 9.

Problem 14. A  (B  C) is equal to


(A) (A  B)  (A  C) (B) (A  B)  (A  C)
(C) (A  B)  A (D) none of these

Solution: Using De-Morgan’s law, we have


A  (B  C) = A  (B  C)
= A  (B  C) = (A  B)  (A  C)
= (A  B)  (A  C).

Problem 15. If Na = {an : n  N} and Nb  Nc = Nd, where a, b, c  N and b, c are relatively


prime, then
(A) d = b + c (B) d = b  c
b
(C) d = bc (D) d 
c
Solution: Nb  Nc = set of all natural numbers which are multiples of both b and c.
 Nd = set of all natural numbers which are multiples of LCM of b and c.
 d = bc ( b, c are relatively prime, therefore, LCM of b and c is bc).

2x  1
Problem 16. If S is the set of all real x such that  0 , then S contains which of the
x3  2x2  x
following intervals :
 3 1   3 1 
(A)  ,  (B)  , 
 2 2  2 4
 1 1  1 
(C)  ,  (D)  , 3 
 4 2 2 

P-2022-CBSE-P1-MATHEMATICS- SRF
20
2x  1 2x  1
Solution: 3 2
0  0
x  2x  x x(x  1)2
2x  1
  0, x  1, 0
x
 2x  1 > 0 and x > 0 or, 2x  1 < 0 and x < 0
1
 x or, x < 0
2
1 
Hence S = ( , 0)   ,    { 1}
2 
1 
Thus  , 3   S.
2 

Problem 17. Let f : [0, 1]  [0, 1] and g : [0, 1]  [0, 1] be two functions defined by
1 x
f(x) = and g(x) = 4x(1  x), then (fog) (x) =
1 x
1  4x  4x2 4(1  x)
(A) 2
(B)
1  4x  4x 1 x
8x(1  x)
(C) (D) none of these
(1  x)2

Solution: (fog) (x) = f (g(x)) = f (4x(1  x))


1  4x(1  x) 1  4x  4x2
 
1  4x(1  x) 1  4x  4x2
Note that Dfog = {x  Dg : g(x)  Df }
= {x  [0, 1] : 4x (1  x)  [0, 1]} = [0, 1] ( 0  4x(1  x)  1 iff 0  x  1).

Problem 18. A survey shows that 63% of the Americans like cheese whereas 76% like apples.
If x % of the Americans like both cheese and apples, then
(A) x = 39 (B) x = 63
(C) 39  x  63 (D) none of these

Solution: Let A denotes the set of Americans who like cheese and B denote who like
apples. Let the population of Americans is 100. Then,
n(A) = 63 and n(B) = 76.
Now, n(A  B) = n(A) + n(B)  n(A  B)
 n(A  B) = 139  n(A  B).
But n(A  B)  100, so that n(A  B)  39.
Further, A  B  A and A  B  B imply that n (A  B)  n(A) and n(A B)  n(B)
that is
n(A  B)  63 and n(A  B)  76]
Hence, 39  n(A  B)  63  39  x  63.

1
Problem 19. If f(x)  , then f(f(f(x))) is an (x  0, 1)
1 x
(A) a quadratic function (B) an identity function
(C) an exponential function (D) not a function

P-2022-CBSE-P1-MATHEMATICS-SRF
21
1 x 1
Solution: fof(x)   , so
1 x
1
1 x
1
fofof(x)  x.
x 1
1
x

Problem 20. Let A, B, C be subsets of the universal set U. If n(U ) = 692, n(B ) = 230, n(C ) =
370, n(B  C) = 20, n(A  B   C ) = 10. Then n(A  B   C ) is equal to
(A) 172 (B) 170
(C) 370 (D) 140

Solution: n(A  B   C ) = n{(B   C )  A}


= n{(B   C )}  n{(B   C )  A}
= n(B  C)  n(A  B   C )
= n()  n(B  C )  n(A  B   C )
= n()  n(B )  n(C ) + n(B  C )  n(A  B   C )
= 692  230  370 + 90  10 = 172.

P-2022-CBSE-P1-MATHEMATICS- SRF
22
CHAPTER PRACTICE PROBLEM

Subjective:

Level – I

1. Find the number of relations that can be defined on the set A = {a, b, c, d}.

2. Find domain of the function f(x) = 4  x  x 1.

3
3. Find range of the function f(x) = 2
.
2x  4x  3

Level – II

1
4. Find domain of the function f(x) = 9  x2   x  2
.
log10

5. If ex + ef(x) = e; then find the range of the function f(x).

Objective:

MCQ Single Correct

1. Let A = {1, 2, 3}, then the relation R = {1, 1), (1, 3)} on A is
(A) domain R is {1, 2, 3} (B) domain R is {1, 2}
(C) Range of R is {1, 3} (D) none of these

2. Let A = {1, 2, 3}, then the domain of the relation R = {(1, 1), (2, 3), (2, 1)} defined on A is
(A) {1, 2} (B) {1, 3}
(C) {1, 2, 3} (D) none of these

3. Let h(x) = e3x, g(x) = lnx, x > 0. Then h(g(x)) is


3
(A) 3x (B) x
(C) ln 3x (D) none of these

4. f(x) = x + 1, x  0
2x – 3, x < 0, then
(A) f(1) = 3 (B) f(–2) = –7
(C) f(0) = –3 (D) none of these

1
5. Domain of the function f (x) = contains the points
log10 (x  2)
(A) 2, 3, 4 (B) 3, 4, 5
(C) all natural numbers (D) 4, 5, 6

6. If f(x) + 2f(1  x) = x2 + 2  xR, then f(x) is given as


2
 x  2 2
(A) (B) x – 2
3
(C) 1 (D) None of these

P-2022-CBSE-P1-MATHEMATICS-SRF
23
7. A  B = A iff
(A) A  B (B) B  A
(C) A = B (D) A  B = 

8. A  B = A if
(A) A  B (B) B  A
(C) A = B (D) A  B = 

9. A  B is equal to
(A) B  A (B) A  B
(C) A  B (D) A  (A  B)

10. If A is finite, then the number of distinct subsets of A is


n(A)
(A) 2n(A) (B) 2
2
(C) (n(A)) (D) none of these

MCQ Multi Correct

1. If P, Q and R are subsets of a set A, then R × (Pc  Qc)c =


(A) (R × P)  (R × Q) (B) (R  Q)  (R  P)
(C) (R  P)  (R  Q) (D) None of these

2. Let X  {1,2,3,4,5} and Y  {1,3,5,7,9} . Which of the following is/are relations from X to Y
(A) R1  {(x,y) | y  2  x, x  X, y  Y} (B) R2  {(1,1),(2,1),(3,3),(4,3),(5,5)}
(C) R3  {(1,1),(1,3)(3,5),(3,7),(5,7)} (D) R 4  {(1,3),(2,5),(2,4),(7,9)}

3. Let A = {1, 2, 3, 4} and R be a relation in A given by R = {(1, 1), (2, 2), (3, 3), (4, 3), (1, 2),
(2, 1), (3, 1), (1, 3)}, then
(A) domain R is {1, 2, 3} (B) range R is {1, 2, 3, 4}
(C) domain R is {1, 2, 3, 4} (D) range R is {1, 2, 3}

x2
4. Let f(x) = , then
2 | x |
(A) domain of f(x) is R (B) domain of f(x) is R – {–2, 2}
(C) f(–1) = 1/3 (D) f(1) = 1

5. Let f(x) = 1 – x; x  [–3, 3] and g(x) = 3 + x, x  [–4, 2], then


(A) (f + g)x = 4; x  [–4, 2] (B) (g – f)x = 2(1 + x); x  [–3, 2]
(C) (f/g)x = (1 – x) / (3 + x); x  (–3, 2] (D) none of these.

Numerical Based

x
1. If f(x)  sinlog x , then the value of f(xy)  f    2f(x).coslog y is equal to _____
y
1 x 
2. If f(x)  cos(logx) , then the value of f(x).f(4)   f    f(4x) is ______
2 4 
3x 3  1
3. Numerical value of the expression for x  3 is _____
2x 2  2

P-2022-CBSE-P1-MATHEMATICS- SRF
24
4. If f be the greatest integer function and g be the modulus function, then
 5  5
(gof )     (fog)     __
 3  3
2x  1
5. If f(x)  , then (fof )(2) is equal to _______
3x  2

6. If A  {2,4,5}, B  {7, 8,9}, then n(A  B) is equal to

7. If n(A)  3 , n(B)  6 and A  B . Then the number of elements in A  B is equal to

8. If n(A)  4 , n(B)  3 , n(A  B  C)  24 , then n(C) 

9. Sets A and B have 3 and 6 elements respectively. What can be the minimum number of
elements in A  B

10. Let A = {1, 2, 3, 4, 5}; B = {2, 3, 6, 7}. Then the number of elements in (A × B)  (B × A)
is

P-2022-CBSE-P1-MATHEMATICS-SRF
25
ASSIGNMENT PROBLEMS

Subjective:

Level – 0

1. Let R = {(1, 2), (2, 3), (3, 4), (4, 1), (1, 4)} be a relation in the set A = {1, 2, 3, 4}. Find the
domain and range of R.

2. Find the domain and range of the relation R =  x, 


x  : 4  x  4, x  Z .

Directions (36) Find the domain and range of the following functions:

3. f (x) = log2  x2  1 .

x 2  2x  3
4. f  x  .
x 2  2x  2

3
5. f  x   1  cos x  .

6. f  x   1 1 x

7. If n (S) = 70,
n (AC  B) = 30
n (A) = 30
then find (i) n (A  B) (ii) n (AC  BC).

8. Find f + g, f  g, fg and f/g if


(a) f (x) = sin x, g (x) = x (b) f (x) = cos x, g (x) = ex.

9. If n (S) = 700, n (A) = 200, n (B) = 300 and n (A  B) = 100, where S is a universal set,
then find n (AC  BC).

10. Let A = {2, 3, 4, 5}, B = {3, 5, 7}, C = {2, 5, 8}. Verify that
(i) A  (B  C) = (A  B)  (A  C)
(ii) A  (B  C) = (A  B)  (A  C).

11. If A = {1, 2, 3} and B = {3, 8}, then find (A  B)  (A  B).

12. If A = {2, 4} and B = {3, 4, 5}, then find (A  B)  (A  B).

13. A and B are two sets having 3 and 5 elements respectively and having 2 elements in
common, then find the number of elements in A  B.

14. Let A = {x: x  R, |x| < 1}, B = {x: x  R, |x – 1|  1} and A  B = R – D, then find set D.

15. Let A = {x: x  R, x  2} and B = {x: x  R, x < 4}, then find set A  B.

16. For any natural number a, we define aN = {ax : x  N}. If b, c, d  N such that
bN  cN = dN, then prove that d is the lcm of both b and c.

P-2022-CBSE-P1-MATHEMATICS- SRF
26
17. If A = {1, 2, 3}; B = {2, 4}, then prove that A = A  (A  B)

18. Write symmetric difference of A = {1, 2, 3} , B = {2, 4}

19. If A = {1, 2, 3, 4, 5}; B = {2, 3, 5, 7}, prove that (A – B)  (B – A) = (A  B) – (A  B)

20. Let U = {1, 2, 3, 4, 5, 6, 7, 8, 9, 10), A = {1, 2, 5), B = (6, 7), then find A  B.

21. If A = {1, 2, 3}; U ={1, 2, 3, 4, 5…. 10}, then prove that A  A = U.

 x 2 ; x  0,3
22. The relation f defined by f(x) =  , prove that f is function.
3x; x  3,10

23. If f(x) = 3x  5, find x when f(x) = 4.

24. Find domain and range of f(x) = 9  x2 .

 1 x  3x  x3
25. If f(x) = log   and g(x)  , then prove fog(x) = 3f(x).
 1 x  1  3x 2

Level – I

1. Prove by venn diagram that A  B = A  BC.

2. Find the range of


(i) f (x) = |x2  3x + 2|
(ii) f (x) = |x2  2x + 3|

3. The Cartesian product A  A has 9 elements among which are found (–1, 0) & (0, 1). Find
set A

4. Participation in sports is compulsory in a school and A class has 80 student out of which
60 students play football and 40 students play basketball. Find how many students
(a) play football only,
(b) play basketball only,
(c) Play both football and basketball.

5. Let A ={1, 2, 3, 4, …… 14}. Define a relation R from A to A by R = {(x, y); 3x –y = 0,


where x, y  A}. Find domain and range

6. Prove that (A  B  C)  (A  B  C)  C = B  C.

7. If A and B are two sets then prove that (A  B)  (A  B) = A.

8. If R is a relation from a finite set A having m elements to a finite set B having n elements,
then find the number of relations from A to B.

9. Let A = {1, 2, 3, 4} and R be a relation in A given by R = {(1, 1), (2, 2), (3, 3), (1, 2), (2, 1),
(3, 1), (1, 3)}. Then find range of the relation R.

10. If f(x) = x2 + 1 defined on R  R, find value of x for which f(x) = 5.

P-2022-CBSE-P1-MATHEMATICS-SRF
27
Level – II

1. A college awarded 38 medals in football, 15 in basket ball and 20 in cricket. If these


medals went to a total of 58 men & only three men got medals in all the three sports, how
many received medals in exactly two of the three sports?

2. In a certain town 25% families own a phone and 15% own a car, 65% families own
neither a phone nor a car, 2000 families own both a car and a phone. How many families
live in the town?

3. Prove by venn diagram that (A  B)  (A  B) = A.

4. If X = {8n  7n  1, n  N} and Y = {49 (n  1), n  N). Prove that X is a subset of Y.

5. If A = {1, 2, 3, 4}. Prove that f defined by f = {(x, y), x + y = 5} is function from A to A.

 x 2 ; x  0,2
6. The relation g is defined by g(x) =  . Prove that g is not a function.
3x; x   2,10

7. Let f = {(1, 1), (2, 3), (0, –1), (–1, –3)} be a function from Z to Z defined by f(x) = ax + b for
some integers a, b. Find a & b.

8. Let A = {9, 10, 11, 12, 13} and let f : A  N be defined by f(n) = then highest prime factor
of n. Find the range of f.

9. Out of 800 boys in a school, 224 played cricket, 240 played hockey and 336 played
basketball. Of the total, 64 played both basketball and hockey; 80 played cricket and
basketball and 40 played cricket and hockey ; 24 played all the three games. Find the
number of boys who did not play any game.

10. If A = {x : x2  5x + 6 = 0), B = {2, 4}, C = {4, 5}, then find A  (B  C).

P-2022-CBSE-P1-MATHEMATICS- SRF
28

Objective:

Level – I

1. Let f (x) = |x  2|, then


(A) f(1) = –1 (B) f(2) = 0
(C) f(x) = –3; for some x (D) none of these
3
2. The range of x when domain is [ 1, 1], is
(A) ( 1, 1) (B) [ 1, 1]
(C) [0, 1] (D) [ 1, 0]

3. The range of y = e|x| is


(A) (0, ) (B) [1, )
(C) ( , ) (D) none of these

4. If f (x) = ex, then what should be (fofof) (x)


ex x
(A) ee (B) ee
x
ee
(C) ee (D) none of these

  
5. The range of f (x) = sin3 x in domain   ,  is
 2 2
(A) [ 1, 1] (B) [ 1, 0]
(C) [0, 1] (D) none of these

1
6. The range and domain of y = are
1  x2
(A) R, R (B) (0, 1], R
(C) R+, R (D) none of these
7. The range of y = sin2x + 2 sinx + 5 contains
(A) 3 integers (B) 4 integers
(C) 5 integers (D) none of these

8. If f : R  R, f (x) = x2 and g (x) = 2x + 1, then


(A) fog = gof (B) fog  gof
(C) fof = gog (D) none of these

9. Range of the function f (x) = ln x2  2x  3 is


(A) [0, ) (B) ( , )
1 
(C) [ln 2, ) (D)  ln 2,  
 2 

10. Range of the function f (x) = 2 + |x2  1| is


(A) [3, ) (B) [2, )
(C) [0, ) (D) (2, )

11. Domain and range of the function f (x) = cos  1  x 2  are


(A) [ 1, 1], [ 1, 1] (B) [ 1, 1], [0, cos 1]
(C) [0, 1], [0, cos 1] (D) [ 1, 1], [cos 1, 1]

P-2022-CBSE-P1-MATHEMATICS-SRF
29
2
x 2
12. Range of the function f (x) = is
2x  3
(A) ( , 1]  [2, ) (B) [1, 2]
(C) ( , 0)  [2, ) (D) (0, 2]

13. Domain and range of the relation R = {(x, y) : x + y = 2, x > 0, y > 0} are
(A) (0, 3), (0, 3) (B) [0, 3), [0, 3)
(C) (0, 2], (0, 2] (D) (0, 2), (0, 2)

14. If n (A  B) = 20, n (A  B  C) = 5, then n (A  (B  C)) is


(A) 15 (B) 10
(C) 5 (D) 20
2
15. The domain and range of the function y = e x are
(A) R, (0, 1] (B) R, [0, 1]
(C) R, (0, 1) (D) none of these

16. Domain of f (x) = log10 1  x3  is


(A) [ 1, ) (B) ( ,  1)
(C) ( 1, 1) (D) ( 1, )

x 1
17. If f(x)  , then f(2x) is
x 1
f(x)  1 3f(x)  1
(A) (B)
f(x)  3 f(x)  3
f(x)  3 f(x)  3
(C) (D)
f(x)  1 3f(x)  1

18. If f(x) = (a  xn)1/n, where a > 0 and n  N, then fof(x) equals


(A) a (B) x
(C) xn (D) an

19. If f(x) = (x + 1)2  1, then f(x) = x give x as


(A) {1, 1} (B) {0, 1}
(C) {0, 1} (D) {0, 1, 1}

2
20. The value of the function f(x)  3 sin  x 2 is defined when x 
16
    3 
(A)   ,  (B) 0 , 
 4 4  2
(C) (3, 3) (D) none of these

P-2022-CBSE-P1-MATHEMATICS- SRF
30
Level – II

Multi-choice (Single Correct Answers)


1. Which of the following is not true ?
(A) 0  {0, {0}} (B) {0}  {0, {0}}
(C) {0}  {0, {0}} (D) 0  {0, {0}}

2. Two finite sets A and B have m and n elements. Number of subsets of A is 56 more than
that of B. The value of m and n are
(A) 6, 3 (B) 7, 6
(C) 7, 3 (D) none of these
2 2
3. Let P = {x; x  N and x satisfies x + 2x – 3 = 0} and Q = {x; x  I and x satisfies 2x – 3x
+ 1 = 0}, then
(A) P = Q (B) P  Q = 
(C) P  Q = R (D) none of these

4. Let P = {x; x  N & x  10}, Q = {x; x is a prime digit}, then


(A) P  Q =  (B) P = Q
(C) P  Q = Q (D) none of these

5. If Q is the set of rational numbers and P is the set of irrational numbers, then
(A) P  Q =  (B) P  Q
(C) Q  P (D) P  Q = 

6. If A and B are finite sets with m and n elements respectively, then the number of relations
from A to B is
(A) mn (B) 2mn
m+n
(C) 2 (D) none of these

2
 x ; x  0,3
7. The relation f defined by f(x) =  , then f(1)  f(5) is equal to
3x; x  (3,10]
(A) 10 (B) 5
(C) 15 (D) 20

8. The range of the relation R = {(–1, 1), (–1, 2), (2, 3), (1, 5)}
(A) {–1, 1, 2} (B) {–1, 1}
(C) {1, 2, 3} (D) {1, 2, 3, 5}
9. Suppose that A1, A2, A3, …, A30 are thirty sets, each containing 6 elements and B1, B2, B3,
30 n
…, Bn are n sets, each containing 3 elements. If  Ai  S  Bi and each element of S
i 1 i 1
belongs to exactly 10 Ai’s and to exactly 9 Bi’s, then n =
(A) 45 (B) 54
(C) 18 (D) none of these
2n+2
10. If X = {3  8n  9 : n  N} and Y = {64n : n  N}, then
(A) Y  X (B) X  Y
(C) X = Y (D) none of these
11. If X = {23n  1 : n  N} and Y = {7n : n  N}, then
(A) X = Y (B) X  Y
(C) Y  X (D) none of these

P-2022-CBSE-P1-MATHEMATICS-SRF
31
2n
12. If P = {3  1 : n  N} and Q = {8n : n  N}, then
(A) P  Q (B) P = Q
(C) Q  P (D) none of these

13. If P = {n3 + (n + 1)3 + (n + 2)3 : n  N} and Q = {8n : n  N}, then


(A) P = Q (B) P  Q
(C) Q  P (D) none of these

14. Power set of the set A = {, {}} is


(A) {, {}, {{}}} (B) {, {}, {{}}}, A}
(C) {, {}, A} (D) none of these

15. The set A = {x  R : 2f(x) = f(2x)}, where f(x) = x2 + 3x, is


(A) {0, 1} (B) {0}
(C) {1, 1} (D) none of these

Multi-choice (One or More Correct Answers)

16. Which one of the following is a function


(A) y = |x| (B) y2 = 4ax
(C) x4 = y2 (D) 3y = (log x)2

17. A  (B  C) equals to
(A) A  B  A  C (B) A  B  A  B  C
(C) B  C  A  C (D) B  C  A  B  C

18. If n ((C  A)  (C  B)) = 30, n (C) = 40, n (A  C) = 5, n (B  C) = 6, then


(A) n (C  B) = 34 (B) n (C  A) = 35
(C) n (A  B  C) = 1 (D) none of these

19. If A = {x : x = n n  N}, B = {x : x = 2n n  N}, C = {x : x = 4n n  N}, then


(A) A  B = {2, 4, 6, …} (B) A  B = {1, 2, 3, …}
(C) A  B = (B  C) = B (D) B  C = B

20. f (x) = x |x| should be


(A) x2, x > 0 (B)  x2, x < 0
(C) 0, x = 0 (D) none of these

21. f (x) = cos (log x), then


1 
(A) f (x) is defined in x  (0, e] (B) f (x) is defined in x   , e 
e 
1 x  1 x 
(C) f  x  f  y    f    f  xy    0 (D) f  x  f  y    f    f  xy    1
2 y  2 y 

22. Let R = {x; x  I & 2 < |x| < 5} and R = {x; x  N & x < 3}, then
(A) R = {3, 4} (B) R = {1, 2, 3, 4…. 8}
(C) R = {–4, –3, 3, 4} (D) R = {0, 1, 2, 3 ….. 8}

x ex
23. Let f(x) = and g(x) = , then
1  x2 1 | x |
(A) domain f(x) is R (B) domain g(x) is R – {1}
(C) domain f(x) is R – {1} (D) domain g(x) is R – {–1, 1}

P-2022-CBSE-P1-MATHEMATICS- SRF
32
2x  1
24. If S is the set of all real numbers x for which  0 , and P is the subset of S,
2x  3x 2  x
3

then P can be
 3 1  1 
(A)   ,   (B)   , 0 
 2 4  2 
1 
(C)  , 3  (D) (0, )
2 

 x  1; x  0
25. Let f  x    2 , then
 x ; x  0
(A) f(–2) = 4 (B) f(3) – f(–2) = 0
(C) f(f(–2)) = 5 (D) none of these.


26. Let f(x)  sin and D = {x : f(x) > 0}, then D contains
x
 1 1  1 1
(A)  ,  (B)  , 
3 2 5 4
 1  1
(C)  1,   (D)  ,  
 2   2 

27. If R = {(x, x + 3); x  {0, 1, 2, 3, 4, 5}}


(A) domain of R = {0, 1, 2, 3, 4, 5} (B) range of R doesn’t contain 2
(C) range R = {3, 4, 5} (D) range R = {3, 4, 5, 6, 7, 8}

28. Let A = {1, 2, 3, 5} & B = {4, 6, 9} relation R from A to B is defined by R = {(x, y); the
difference between x & y is odd; x  A, y  B}, then
(A) domain of R is {1, 2, 5} (B) domain of R is {1, 2, 3, 5}
(C) range of R is {4, 6, 9} (D) domain of R is {4, 6}

Reason Assertion

29. Statement  I: X = {8n + 1 : n  N}, then X  Y = Y


Y = {(2n + 1)2 : n  N}
because
Statement  II: X is a subset of Y
(A) Statement -1 is True, Statement -2 is True; Statement-2 is a correct explanation for
Statement-1
(B) Statement -1 is True, Statement -2 is True; Statement-2 is NOT a correct explanation
for Statement-1
(C) Statement -1 is True, Statement -2 is False
(D) Statement -1 is False, Statement -2 is True

P-2022-CBSE-P1-MATHEMATICS-SRF
33
COMPREHENSIONS
(I)
Read the following write up carefully and answer the following questions:

All possible real values of x for which a function y = f (x) is defined, is called Domain of f (x).
e.g. y = logx e is defined for all possible values of x except x = 1.
 Domain = {x : x  (0, )  {1}}.

30. The domain of the function f (x) = log0.3  x  1 is


(A) [1, ) (B) (1, )
(C) (1, 2] (D) none of these

1
31. Domain of f (x) = is given by
x  2  x2  2x
(A) (–1, 2) (B) (–1, )
(C) (2, ) (D) (–, –1)

32. The domain of the function f (x) = 3  2x  21 x is


(A) (0, 1) (B) [0, 1)
(C) [0, 1] (D) (0, 1]

(II)

Read the following write up carefully and answer the following questions:

x ; x   2, 1   
Let f(x) = , g  x   2 sin x; x    , 
x  1; x  (1,2]  2 2

33. Then f(g(/6)) is equal to


(A) –2 (B) –1
(C) 0 (D) 1

34. g(f(–/2)) is equal to


(A) –1 (B) 0
(C) 1 (D) 2

(III)

Read the following write up carefully and answer the following questions:

For finding range we sometimes use the quadratic equation for example f(x) = x 2 + 2x + 2 will
have range [1, ) as f(x) = x2 + 2x + 2 = (x + 1)2 + 1  1.
2
35. Range of sin x + 2sinx + 2 is
(A) [1, 5] (B) [1, )
(C) [1, 1] (D) (0, )

36. Range of x2 + 4x + 6 is


(A) [4, ) (B) (, 4]
(C) (0, 5) (D) (0, )

P-2022-CBSE-P1-MATHEMATICS- SRF
34
Matrix Match

37. Match the following:


Column  I Column  II
3 +
(A) f(x) = x (p) domain is R
(B) f(x) = |x| (q) range is R
(C) y = loge x (r) domain is R
(D) f(x) = cos x (s) range is [–1, 1]

38. Let A = {2, 3, 4, 6, 12}. Let R be relation on A defined by {(a, b) : a, b  A, b is exactly


divisible by a and a  b}. Let X & Y represent the domain and range of relation R.
List  I ListII
(I) X (P)  {2, 3, 4, 6, 12}
(II) Y (Q) = {2, 3, 4, 6, 12}
(III) XY (R) = {2, 3, 4, 6}
(IV) X–Y (S)  {3, 4, 6, 12}
(T) = {2, 3}
(U) = {4, 6, 12}

Which of the following is the only correct combination?


(A) (IV)  P, R, T (B) (III)  Q
(C) (IV)  P, T (D) (II)  P, Q, U

39. Matching List Type(3 Column & 4 Rows)


Column – 1 Column – 2 Column – 3
(I) A = {prime no  11} (i) Df = { 3,  2, 1, (P) Rf = {0, 1, 2, 3, 4}
B = {x: x  N & x  [1, 5]} 0}
(II) (ii) AB = {1, 2, 3, 4, (Q) A  B = {2, 3, 5}
A B
1 3 4 8 5, 7, 11}
2 5 7
A
6
(III) 2 (iii) AB = {1, 2, 3, 4, (R) A  B = {3}
R = {(x, y): y   x  1 , 5}
x  Z & –3  x  3
(IV) A = {x : x  Z, x2 < 10} (iv) AB = {–3, –2, – (S) A  B = {–2,–1,1, 2}
B = {–2,–1, 1, 2} 1, 0, 1, 2, 3}

(I). Which of the following is correct combination


(A) (I) (i) (P) (B) (II) (i) (P)
(C) (III) (i) (P) (D) (IV) (i) (P)

(II). Which of the following is correct combination


(A) (IV) (i) (P) (B) (IV) (ii) (Q)
(C) (IV) (iii) (R) (D) (IV) (iv) (S)

(III). Which of the following is not correct combination


(A) (I) (ii) (Q) (B) (II) (iii) (R)
(C) (III) (iv) (S) (D) (IV) (iv) (S)

P-2022-CBSE-P1-MATHEMATICS-SRF
35
Numerical Based

40. A market research group conducted a survey of 1000 consumers and reported that 720
consumers like product A and 450 consumers like product B. Then minimum number of
consumers who liked both product is 100a + 10b + c, then a + b + c is _______

41. Two finite sets have m and 3 elements respectively. The total number of subsets of first
set is 56 more than the total number of subsets of the second set. The value of m is
__________

42. Sets A and B have 3 and 6 elements each. The maximum possible number of elements
in A  B is __________

P-2022-CBSE-P1-MATHEMATICS- SRF
36
ANSWERS TO CHAPTER PRACTICE PROBLEM

Subjective:

Level – I
16
1. 2
2. [–1, 4]
3. (0, 3]

Level – II

4. (–2, 3] – {–1}
5. (−, 1)

Objective:

MCQ Single Correct

1. C 2. A 3. B 4. B

5. D 6. A 7. D 8. B

9. D 10. B

MCQ Multi Correct

1. A, B 2. A, B 3. C, D 4. B, D

5. B, C

Numerical Based

1. 0 2. 0 3. 4 4. 1

5. 2 6. 9 7. 6 8. 2

9. 6 10. 4

P-2022-CBSE-P1-MATHEMATICS-SRF
37
ANSWERS TO ASSIGNMENT PROBLEMS

Subjective:

Level – 0

1. Domain {1, 2, 3, 4}
Range {1, 2, 3, 4}

2. Range : {0, 1, 2 , 3 , 2}
Domain: {0, 1, 2, 3, 4}

3. Domain: R
Range [0, )

4. Domain : R
Range (1, 2]

5. Domain : R
Range : [0, 8]

6. Domain: [-1, )
Range: [1, )

7. n (A  B) = 60
n (AC  BC) = 10

sin x
8. (a) sin x + x, sin x  x, x sin x, ,x0
x
(b) cos x + ex, cos x  ex, ex cos x, ex cos x

9. 300

11. {(1, 3), (2, 3), (3, 3), (8, 3)} 12. {(4, 2), (4, 3), (4, 4), (4, 5)}

13. 15 14. D = {x: 1  x < 2}

15. {x: x  R, 2  x < 4} 18. {1, 3, 4}

20. A 23. 3

24 domain : [–3, 3]; range : [0, 3]

Level – I

2. (i) Range : [0, ) (ii) Range : [2, )


3. A = {–1, 0, 1} 4. 40, 20, 20
5. domain = {1, 2, 3, 4}, Range = {3, 6, 9, 12} 8. 2mn
9. {1, 2, 3} 10. x=2

P-2022-CBSE-P1-MATHEMATICS- SRF
38
Level – II

1. 9 2. 40,000
7. a = 2, b = – 1 8. {3, 5, 11, 13}
9. 160 10. {(2, 4), (3, 4)}

Objective:

Level – I

1. B 2. B 3. B 4. A
5. A 6. B 7. C 8. B
9. D 10. B 11. D 12. A
13. D 14. A 15. A 16. D
17. B 18. B 19. B 20. A

Level – II

1. D 2. A 3. A 4. C
5. A 6. B 7. C 8. D
9. C 10. B 11. B 12. A
13. B 14. B 15. B 16. A, D
17. A, B 18. A, B, C 19. A, B, C, D 20. A, B, C
21. A, B, C 22. B, C 23. A, D 24. B, C
25. A, B, C 26. A, B, C 27. A, B, D 28. B, C
29. B, D 30. C 31. A 32. C
33. C 34. D 35. A 36. B
37. (A)  (q, r) (B)  (r) (C)  (p, q) (D)  (r, s)
38. C
39. (I). C (II). D (III). C
40. 8 41. 6 42. 9

P-2022-CBSE-P1-MATHEMATICS-SRF
PINNACLE-CBSE

TRIGONOMETRIC RATIOS & IDENTITIES


CONTENTS SYLLABUS
 Angle and its Measurement 1
 Definitions of Trigonometric Ratios or CBSE: Degree measure and radian measure of positive
circular functions 3 and negative angle, relation between degree and
 Trigonometric Identities 5 radian, Definition of trigonometric functions with the
 Exercise 1 6 help of a unit circle, Periodic functions, concept of
 Trigonometric Ratios of Standard periodicity of Trigonometric functions, Values of
angles 7 trigonometric functions of x for x = 0, /6, /4, /3, 
 Allied Angles 9 /2, , 3 /2, 2. Trigonometric functions of sum and
 Exercise 2 12 difference of numbers.
 Trigonometric Ratios of compound
angles 12 Trigonometric functions of multiples and submultiples
 Trigonometric Ratios of multiple of numbers.
angles 15 Conditional identities for the angles of a triangle,
 Exercise 3 17 Graph of the following trigonometric functions:
 Trigonometric Ratios of sub-multiple
of an angle 17 y = sinx, y = cosx, y = tanx, y = asinx, y = acosx, y =
asinbx, y = acosbx.
 Exercise 4 19
 Identities 20
 Exercise 5 22
 Graph of Trigonometric functions
22
 Miscellaneous Exercise 23 JEE: Trigonometric functions, their periodicity and
 Answers to Exercises 25 graphs, addition and subtraction formulae, formulae
 Solved Problems 26 involving multiple and sub-multiple angles
 Chapter Practice Problem 46
 Assignment Problems 49
 Answers to Chapter 61
Practice Problems
 Answers to Assignment
Problems 62
Angle and Its Measurement

Angle:

Let a revolving line, starting from OX, revolve about O in a plane in the direction of the arrow (in
anticlockwise direction) and occupy the position OP. Then it is said to trace an angle XOP. OP is
called the final (or terminal) position of OX which is the initial position. The point O is called the
vertex. More precisely an angle may be defined as a measure of the rotation of a half ray about
its origin. An angle XOP is positive if it is traced by a ray revolving in the anticlockwise direction
and negative if it is traced by a ray revolving in the clockwise direction.

P Initial Position
X
O

Terminal Position Negative Angle
Positive Angle
 Terminal Position
X
O Initial Position P

Measurement of an Angle:

There are three systems of measurement of an angle:


i) Sexagesimal System or English System
ii) Centesimal system or French System
iii) Circular System

Sexagesimal System (degree measure):


In this system an angle is measured in degrees, minutes and seconds. One complete revolution
0 0 0
is 360 where one degree is written as 1 . Further 1 is divided into 60 equal parts where one part
is one minute (written as 1). Again one minute is divided into 60 equal parts and one part is one
0
second (written as 1). Thus 1 = 60 minutes (or 60) and 1 = 60 seconds (or 60). An angle of
0
90 is also called a right angle.

Centesimal System (grade measure):

In this system an angle is measured in grades, minutes and seconds.


Here 1 right angle = 100 grades, written as 100g,
1 grade = 100 minutes, written as 100,
and 1 minute = 100 seconds, written as 100.

P-2022-CBSE-P1-MATHEMATICS-TRI
2
Circular System (radian measure):
In this system an angle is measured in radians. A radian is an angle subtended at the centre of a
circle by an arc whose length is equal to its radius.
Let AB be an arc of a circle of radius r such that length of the
B
arc AB = r. Then AOB = 1 radian (written as 1C).
Since the whole circle subtends an angle of 3600 (4 right
r
angles) at the centre and the angles at the centre of a circle O 
are in the ratio of subtending arcs, r
AOB arc AB A
 so that
4 right angles circumference
arc AB  4 right angles
AOB =
2r
r 2
 1 radian =  4 right angles =  right angle   radians = 2 right angles = 1800 = 200g
2r 
180 180  7 6300
 1 radian = degree =  = 5701622 nearly.
 22 11

Number of Radians in an Angle Subtended by an Arc of a Circle at the Centre:

Let AOP =  radian be the angle subtended by an arc AP P

(length of AP = l unit) of a circle at the centre O. An arc AB = r B


is cut off from AP. Then, AOB = 1 radian and C
r
AOP arc AP l
  O 1C
AOB arc AB r
r
l l A
 AOP =  1 radians = radians.
r r
arc AP
Hence number of radians in AOP = .
radius

Illustration 1. Express in radians the fourth angle of a quadrilateral which has three angles
0 0 0
60 , 55 and 100 .

Solution: The sum of three angles = 600 + 550 + 1000 = 2150.


0 0 0
The fourth angle = 360 – 215 = 145 .

Since 10 = radians,
180
 29
1450 =  145 = radians.
180 36

22
Illustration 2. The difference between two acute angles of a right angled triangle is
35
radians. Find these angles in degrees.

Solution: Let x and y be two acute angles. Then:

P-2022-CBSE-P1-MATHEMATICS-TRI
3
22 22 180
x–y= radian =  degrees
35 35 
180 0
or x – y = = 36 . ……(1)
5
0
Also x + y = 90 ……(2)
0 0
 x = 63 and y = 27 .
g
 4x  2 x
Illustration 3. The angles of a triangle are 3x0,   and radians. Find all the angles.
 3  75

0 g 0
g 0 g 9  4x   36x 
Solution: Since 100 = 90  1 =    
10  3   30 
1800
and C = 1800  1C =

C 0
 2x   24x  36x 24x
      3x +  = 180
 75   5  30 5
 9x = 1800  x = 200
0 0 0
 60 , 24 , 96 are angles of the triangle.

Illustration 4. A circular wire of radius 3 cm is cut and bent so as to lie along a circle of radius
48 cm. Find the angle subtended by the wire at the centre of the circle.

Solution: Length of the circular wire = 2r = 6 cm.


arc
Angle subtended by the arc at the centre =
radius
6  1800
  radians = = 22.50.
48 8 8

Definitions of Trigonometric Ratios or Circular Functions:


Let a revolving line starting from OX trace an angle ‘’ in any of the four quadrants. Let M be the
foot of the perpendicular from P upon XOX. Regarding OM and MP as directed lengths (taking
OP positive), the ratios of OM, MP and OP with one another are called circular functions or
trigonometric ratios of angle .

Y Y
P P
r
r 
y

X X X
X O x M M O

Y Y

P-2022-CBSE-P1-MATHEMATICS-TRI
4
Y Y

 

M O M
X X X
O X

P P
Y Y

Let OM = x, MP = y and OP = r > 0. The circular functions are defined as:


PM y OM x
(i).  = sin  (ii).  = cos 
OP r OP r
PM y OM x
(iii).  = tan , x  0 (iv).  = cot , y  0
OM x PM y
OP r OP r
(v).  = cosec , y  0 (vi).  = sec , x  0
PM y OM x

Trigonometric ratios (or functions) may also be defined with respect to a triangle.
In a right angled triangle ABC, CAB = A and BCA = 90° = /2. AC is the base, BC the altitude
and AB is the hypotenuse. We refer to the base as the adjacent side and to the altitude as the
opposite side. With reference to angle A, the six trigonometric ratios are:
BC opposite side B
 is called the sine of A, and written as sinA.
AB hypotenuse
AC adjacent side
 is called the cosine of A, and written as cosA.
AB hypotenuse
BC opposite side
 is called the tangnet of A, and written as tanA. A C
AC adjacent side
sinA
Obviously, tan A= . The reciprocals of sine, cosine and tangent are called the cosecant,
cosA
secant and cotangent of A respectively. We write these as cosecA, secA, cotA respectively. Since
the hypotenuse is the greatest side in a right angle triangle, sinA and cosA can never be greater
than unity and cosecA and secA can never be less than unity. Hence sin A 1,
cos A 1, cosec A 1, sec A1, while tan A and cot A may take any numerical value.

Notes:
 All the six trigonometric functions have got a very important property in common that is
of periodicity.
 Remember that the trigonometrical ratios are real numbers and remain same as long as
angle A is real.

Signs of Trigonometric Ratios:


The following table describes the signs of various trigonometric ratios:
Ist quadrant IInd quadrant IIIrd quadrant IVth quadrant
All positive sin , cosec  positive tan , cot  positive sec , cos  positive

P-2022-CBSE-P1-MATHEMATICS-TRI
5
Trigonometrical Identities:
An identity is a relation which is true for all values of the independent variable. There are three
fundamental identities involving trigonometrical ratios:
i) sin2  + cos2  = 1 ii) 1 + tan2  = sec2 
2 2
iii) 1 + cot  = cosec 

Proof: (i) In triangle OPM; Y


2 2 2 P
OP = OM + PM ……(1)
2
Dividing both sides of (1) by OP , we get
OM2 PM2
1=   1 = cos2  + sin2 . 
X
OP2 OP2 X O M
(ii) Dividing both sides of (1) by
OP2 MP2
OM2 we get:  1 
OM2 OM2 Y
2 2
 sec  = 1 + tan .
(iii) Dividing both sides of (1) by MP2 we get
OP2 OM2 2 2
  1  cosec  = cot  + 1.
MP2 MP2

cos A sin A
Illustration 5. Prove that  = sin A + cos A.
1  tan A 1  cot A

cos A sin A cos A sin A


Solution: LHS =   
1  tan A 1  cot A sin A cos A
1 1
cos A sin A
cos2 A sin2 A cos2 A  sin2 A
=  = = sin A + cos A = RHS.
cos A  sin A sin A  cos A cos A  sin A

1  cos 
Illustration 6. Prove that = cosec  + cot .
1  cos 

1  cos  1  cos  1  cos  1  cos 


Solution: LHS = =   = cosec  + cot  = RHS.
1  cos  1  cos  1  cos  sin 

Illustration 7. Eliminate ‘’ from the equations x = a cos  + b sin , y = a sin  – b cos .
Solution: Squaring and adding the given equations we get:
2 2 2 2 2 2 2 2 2 2
x +y = a cos +b sin +2ab cos  sin +a sin  + b cos  – 2ab cos  sin 
 x2+y2 = a2(cos2  + sin2 ) + b2(sin2  + cos2 )  x2+y2 = a2 + b2.
n n
Illustration 8. If un = cos  + sin , then prove that 2u6 - 3u4 + 1 = 0.

Solution: Given un = cosn  + sinn   u6 = cos6  + sin6 


2 2 4 4 2 2
= (cos  + sin )(cos  + sin  – cos  sin )
= (cos  + sin ) – 3cos  sin  = 1 – 3cos2  sin2 .
2 2 2 2 2

4 4 2 2
Also u4 = cos  + sin  = 1 – 2cos  sin 
2 2 2 2
 2u6 – 3u4 + 1 = 2 – 6cos  sin  – 3 + 6cos  sin  + 1 = 0.

P-2022-CBSE-P1-MATHEMATICS-TRI
6
Illustration 9. Express tan in terms of cos.

Solution: By definition y
OA x B
cos     x where OB is taken as
OB 1 1
unity and OA = x. 1 x 2
Hence in AOB, OA = x, OB = 1, O 
x A x
AB  1  x 2 . Fig. 3
By definition,
AB 1  x2 1  cos2 
tan    = .
OA x cos 

21 5
Illustration 10. If sin  = and ‘’ lies in the second quadrant, show that sec  + tan  = – .
29 2

Solution: Since  lies in the second quadrant, cos  is negative and tan  is also negative.
2 2
2 2  21  400  20 
Now cos  = 1  sin  = 1      
 29  (29)2  29 
20 sin  21
 cos  =  and hence tan  =  .
29 cos  20
29 21 50 5
Hence sec  + tan  =     .
20 20 20 2

Illustration 11. Prove that (sin + cosec)2 + (cos + sec)2  9.

Solution: The given expression on the LHS is


sin2 + cosec2 + 2 + cos2 + sec2 + 2
= 4 + (sin2 + cos2) + (1 + tan2) + (1 + cot2)
= 7 + tan2 + cot2 = 7 + (tan - cot)2 + 2  9.

Exercise 1.
1 1 1 1
i) Prove that    .
secA  tanA cosA cosA secA  tanA
2 2 3 2 3/2
ii) If tan  = 1 – e then show that sec + tan  cosec = (2 – e ) .
4xy
iii) For what real values of x and y is the equation sec2 = possible?
2
 x  y 
iv) Prove the following trigonometric identities:
1  sin  2
(a) sec (1 – sin )(sec  + tan ) = 1 (b) = (sec  – tan )
1  sin 
sin x 1  cos x tan A  sec A  1
(c)  = 2 cosec x (d) =secA+tanA
1  cos x sin x tan A  sec A  1
(e) (cosec  – sec )(cot  – tan ) = (cosec  + sec )(sec  cosec  – 2).
p p sin   q cos  p 2  q 2
v) If tan  = show that  .
q p sin   q cos  p 2  q 2

P-2022-CBSE-P1-MATHEMATICS-TRI
7
Trigonometric Ratios of Standard Angles

Trigonometric Ratios of 450:


In the adjacent figure OM = PM = y and POM = 450. Here P
y 1
sin450 =  , cosec 450 = 2 ,
y 2 2 y 2 45 0
y
0 y 1 0
cos45 =  , sec 45 = 2 , 90 0
y 2 2 4 50
y O y M X
tan450 = 0
 1 , cot 45 = 1.
y
0
Trigonometric Ratios of 30 :

Let the revolving line trace POM = 300. Let PM = y. Let P


be the image of P in OX so that OPP is an equilateral P

triangle. Then OP = PP = 2y 2y


y
2 2
 OM = 4y  y  3y , so that
300 3y
0 y 1 0 X
sin 30 =  , cosec 30 = 2, O 300 M
2y 2
3y 3 2 y
cos 300 =  , sec 300 = ,
2y 2 3
P
y 1
tan 300 =  , cot 300 = 3 .
3y 3

Trigonometric Ratios of 600:


Let a revolving line, starting from OX, trace XOP = 600. Y P
PM is perpendicular from P on OX, P is a point on OX such
that OPP is equilateral. Let OP = 2y  OP = 2y 2y 300 300
2 2
 OM = y  PM = 4y  y  3y .
3y
3y 3 2
Hence sin 600 = , cosec 600 =
0
 , 60
2y 2 3 X
O y M P
0 y 1 0
cos 60 =  , sec 60 = 2,
2y 2
3y 1
tan 600 = = 3 , cot 600 = .
y 3

Trigonometric Ratios of 00:


Y
Let a revolving line, starting from OX, trace XOP = 00. So
that P lies on OX. Here PM = 0. We have
PM 0 OP OM P
sin 00 =  = 0, cosec 00 =  (not defined) X
OM OM PM 0 O M
OM
cos 00 = 0
 1 , sec 0 = 1,
OM
PM 0 OM
tan 00 =  = 0, cot 00 = (not defined).
OM OM 0

P-2022-CBSE-P1-MATHEMATICS-TRI
8
Trigonometric Ratios of 900:
0 Y
Let the revolving line OX, trace POX = 90 . From P, perpendicular
PM is drawn to OX so that M coincides with O. Hence P
PM y OP y
sin 900 =  = 1, cosec900 =  1
OP y PM y 900
X
OM 0 y O M
cos 900 =  = 0, sec 900 = (not defined),
OP y 0
PM y 0
tan 900 =  (not defined), cot 900 = = 0.
OM 0 y

Illustration 12. If sec  + tan  = 4, find sin , cos . Also find the quadrant in which ‘’ lies.

Solution: Given sec  + tan  = 4. …(1)


Also sec2  – tan2  = 1.
1
Hence sec   tan  = . …(2)
4
17
Solving (1) and (2), we get sec  =
8
8
 cos  =
17
15 15 8 15
and tan  =  sin  =  = .
8 8 17 17
Since both sin  and cos  are positive,  lies in the 1st quadrant.

Illustration 13. If 3 sin  + 5 cos  = 4, find the value of 5 sin  – 3 cos .

Solution: Let x = 5 sin  – 3 cos 


 x2 = (5 sin  – 3 cos )2
= 25sin2  + 9cos2  – 30 sin  cos 
= 25  25 cos2  + 9  9 sin2   30 sin  cos 
= 34 – (3 sin  + 5 cos )2 = 34 – 16 = 18.
Hence, 5 sin  – 3 cos  =  3 2 .

Illustration 14. If A, B, A – B and A + B are positive acute angles, find the values of A and B from
1 1
the equations sin (A – B) = , cos (A + B) = .
2 2

1
Solution: The equations are sin (A – B) = = sin 300  A – B = 300. …(1)
2
1 0 0
And cos (A + B) = = cos 60  A + B = 60 . ...(2)
2
From (1) and (2), A = 450 and B = 150.

P-2022-CBSE-P1-MATHEMATICS-TRI
9
Allied Angles
Trigonometrical Ratios of Angle (–) in Terms of :
From the figure, we find that
y Y
sin (–) = = – sin ,
r P
x
cos (–) = = cos , r
r y
y x
tan (–) = = – tan , 
x X
O – M
x
cot (–) = = – cot , r –y
y
r
cosec (–) = = – cosec , P
y
r
sec (–) = = sec .
x
Trigonometrical Ratios of Angle (900 – ) in Terms of :

From the figure, we get


x Y
sin (900 – ) = = cos ,
r P
y 90–
cos (900 – ) = = sin , r
r y
x
tan (900 – ) = = cot ,  x
X
y O M
y
cot (900 – ) = = tan ,
x
r
cosec (900 – ) = = sec ,
x
0 r
sec (90 – ) = = cosec .
y

Trigonometrical Ratios of (900+ ) in Terms of :


0
Let a revolving line OP trace angle POX = 90 + , where Y
P
POM = . Also let PM = y, OM = x, OP = r.
P
In similar triangles OPM and OPM, OM = –y, PM = x. 
 r
x x y
Hence sin (900 + ) = = cos ,
r –y 
y x M O x M X
cos (900 + ) = = –sin , tan (900 + ) = = – cot ,
r y
y
cot (900 + ) = = – tan ,
x
r
sec (900 + ) = = – cosec ,
y
r
cosec (900 + ) = = sec .
x

P-2022-CBSE-P1-MATHEMATICS-TRI
10
Trigonometrical Ratios of (1800 – ) in Terms of :

If the revolving line OP traces angle POM =  – , then


Y
in similar triangles OPM and OPM, PM = PM = y,
P P
OM = –OM = – x. Hence r
y r  –
y y
sin (180 – ) = = sin , 
r 
X
M –x O x M
x
cos (180 – ) = = – cos ,
r
y
tan (180 – ) = = – tan ,
x
x
cot (180 – ) = = – cot ,
y
r
sec (180 – ) = = – sec ,
x
r
cosec (180 – ) = = cosec .
y

Trigonometrical Ratios of (1800 + ) in Terms of :


Here
sin (180 + ) = sin (180 – (–)) = sin (–) = – sin ,
cos (180 + ) = cos (180 – (–)) = – cos (– ) = – cos ,
tan (180 + ) = tan (180 – (–)) = – tan (– ) = tan ,
cosec (180 + ) = cosec (180 – (–)) = cosec (– ) = – cosec ,
cot (180 + ) = cot (180 – (–)) = – cot (–) = cot ,
sec (180 + ) = sec (180 – (–)) = – sec (–) = – sec .
Trigonometric Ratios of (3600 – ) in Terms :

Let XOP = 360 – . Then POM = , Y


OP = OP = r, PM = – PM = y so that P
y 360 – 
sin (360 – ) = = – sin , r
r y
x x
cos (360 – ) = = cos ,  X
r O M
y
tan (360 – ) = = – tan , –y
x r
x
cot (360 – ) = = – cot , P
y
r
sec (360 – ) = = sec ,
x
r
cosec (360 – ) = = – cosec 
y
and sin (360 + ) = sin , cos (360 + ) = cos ,
tan (360 + ) = tan ,
cot (360 + ) = cot ,
sec (360 + ) = sec ,
cosec (360 + ) = cosec .

P-2022-CBSE-P1-MATHEMATICS-TRI
11
Table – I

 equals sin cos tan cot sec cosec


– – sin cos –tan – cot sec –cosec 
90° –  cos sin cot tan cosec sec
90° +  cos – sin –cot – tan –cosec sec
180°–  sin – cos – tan – cot – sec cosec
180°+  – sin – cos tan cot – sec –cosec
360°–  – sin cos – tan – cot sec –cosec
360°+  sin cos tan cot sec cosec

Note:
 Angle  and 90° –  are complementary angles,  and 180° –  are supplementary
angles
 sin(n + (– 1)n) = sin, n  
 cos(2n ± ) = cos, n  
 tan(n + ) = tan, n  
i.e. sine of general angle of the form n + (–1)n will have same sign as that of sine of angle  and
so on. The same is true for the respective reciprocal functions.

Illustration 15. Prove that sin (–6900) cos (–3000) + cos (–7500) sin (2400) = 1.

Solution: We have sin (–6900) = – sin 6900 = – sin (3600 + 3300) = – sin 3300
1
= – sin (3600 – 300) = sin 300 = ,
2
1
cos (–3000) = cos 3000 = cos (3600 – 600) = cos 600 = ,
2
3
cos (–7500) = cos 7500 = cos (2  360 + 30)0 = cos 300 = ,
2
3
sin (–2400) = – sin 2400 = – sin (180 + 60)0 = sin 600 = .
2
1 1 3 3
Hence sin (–6900) cos (–3000) + cos (–7500) sin (–2400) =  +  = 1.
2 2 2 2

 3 5 7
Illustration 16. Prove that cos2 + cos2 + cos2 + cos2 = 2.
8 8 8 8

5   
Solution: We know that cos = cos    = – sin
8  2 8  8
7   3  3
and cos = cos    = – sin
8 2 8  8
2  2 3  2 5  2 7  2  2 3 2 2 3
 cos +cos +cos +cos =cos +cos +sin +sin =2.
8 8 8 8 8 8 8 8

Illustration 17. Find x from the equation cosec (900 + ) + x cos  cot (900 + ) = sin (900 + ).

Solution: The given equation cosec (900 + ) + x cos  cot (900 + ) = sin (900 + )

P-2022-CBSE-P1-MATHEMATICS-TRI
12
 sec  + x cos  (– tan ) = cos   sec  – x sin  = cos 
1  cos2  sin2 
 x sin  = sec  – cos   x =  = tan .
cos  sin  cos  sin 

1 1
Illustration 18. Find the general value of  satisfying both sin   and tan   .
2 3

Solution: Let us first find out  lying between 0 and 360°.


1
Since sin    ,  = 210° or 330°
2
1
and tan     = 30° or 210°.
3
7
Hence  = 210° or , satisfying both the equations
6
 7 
 The general value of    2n  , n   .
 6 

Exercise 2.
i) Find the value of tan1 tan2 tan3 …. tan89.
1
ii) Prove that sin25 + sin210 +…. + sin290 = 9 .
2
iii) If cos  – sin  = 2 sin , show that cos  + sin  = 2 cos .
iv) Prove that tan700 = 2 tan500 + tan200.

Trigonometric Ratios of Compound Angles:

Compound Angle:

An angle made up of algebraic sum of two or more angles is called compound angle. If A, B and
C are given angles, then A + B, A – B, A + B + C, A – B + C, A + B – C etc. are compound
angles.

The Addition Formulae:


(i). sin (A + B) = sin A cos B + cos A sin B
(ii). cos (A + B) = cos A cos B – sin A sin B
tan A  tanB
(iii). tan (A + B) =
1  tan A tanB
Proof:
Let the revolving line, starting from OX, describe first XOY = A and then proceed further so as
to describe YOZ = B in its position OZ. Then XOZ = A + B.
Also P and Q are points on OY and OZ such that QPO = 900 and PL and QM are
perpendiculars to OX. Draw PH perpendicular to QM so that PQH = A = OPH.

P-2022-CBSE-P1-MATHEMATICS-TRI
13
In OQM, Z
Q A
QM QH  PL QH PL
sin(A + B) =  = 
OQ OQ OQ OQ A
QH QP PL OP Y
=    = cos A sin B + sin A cos B
QP OQ OP OQ H P
A
OM OL  LM OL HP
and cos(A + B) =   
OQ OQ OQ OQ
OL OP HP PQ B
=   
OP OQ PQ OQ A
X
= cosA cosB – sinA sinB . O M L
QH PL PQ PL
 
QM QH  PL OL OL
Moreover, tan(A + B) =    OP OL .
OM OL  ML PH PH PL
1 1 
OL PL OL
tan A  tanB  QH PH PQ 
=  since   in similar trianlges HPQ and OPL  .
1  tan A tanB  OL PL OP 
sin  A  B  sin A cosB  cos A sinB tan A  tanB
Alternatively: tan(A + B) =   .
cos  A  B  cos A cosB  sin A sinB 1  tan A tanB

Subtraction Formulae:

(i). sin(A – B) = sin A cos B – cos A sin B


(ii). cos(A – B) = cos A cos B + sin A sin B
tan A  tanB
(iii). tan(A – B) =
1  tan A tanB
Proof:
Let the revolving line starting from the position OX describe first XOY = A and then revolving in
the opposite direction describe YOZ = B in its position OZ. Then XOZ = A – B.
Let Q be a point on OZ. From Q draw QM  OX and QP  OY. From P draw PH  QM and
PL  OX.
Y
Now PQH = POX = A,
P H
QPO = 900 and from, QOM
QM HM  HQ PL  HQ 900 A
sin (A – B) =   Z
OQ OQ OQ
PL OP HQ PQ Q
=    = sin A cos B – cos A sin B
OP OQ PQ OQ
OM OL  LM OL  PH B
and cos (A – B) =   A
OQ OQ OQ
A–B
OL OP PH PQ O
X
=    L M
OP OQ PQ OQ

= cos A cos B + sin A sin B.

P-2022-CBSE-P1-MATHEMATICS-TRI
14
PL QH PL QH
 
QM HM  QH PL  QH OL OL
Moreover, tan (A – B) =     OL OL .
OM OL  LM OL  PH PH PH PL
1 1 
OL PL OL
From similar triangles QPH and POL
QH PH PQ
 
OL PL OP
PL PQ

tan A  tanB
 tan (A – B) = OL OP = .
PQ PL 1  tan A tanB
1 
OP OL
Subtraction formulae can also be derived from the addition formulae by writing
sin (A  B) = sin (A + ( B)) = sin A cos ( B) + cos A sin ( B) = sin A cos B  cos A sin B
and so on.

Some Important Deductions:


(i). sin (A + B) sin (A – B) = sin2 A – sin2 B
(ii). cos (A + B) cos (A – B) = cos2 A – sin2 B
cot A cot B  1
(iii). cot (A + B) =
cot B  cot A
cot A cot B  1
(iv). cot (A – B) =
cot B  cot A

Illustration 19. Find the values of (a) cos 150 (b) sin 1050.

Solution: We know that cos 150 = cos (45 – 30)0 = cos 450 cos 300 + sin 450 sin 300
1 3 1 1 3 1
=     .
2 2 2 2 2 2
3 1
Also sin 1050 = sin (45 + 60)0 = sin 450 cos 600 + cos 450 sin 600 = .
2 2

Illustration 20. Prove that cos 180 – sin 180 = 2 sin 270.

0 0 0 0 0 0
Solution: RHS = 2 sin 27 = 2 sin (45 – 18) = 2 (sin 45 cos 18 – cos 45 sin 18 )
 cos180  sin180  0 0
= 2  = cos 18 – sin 18 = LHS.
 2
 

0 0 0
Illustration 21. Prove that tan 70 = 2 tan 50 + tan 20 .

tan 500  tan 200


Solution: We have tan 700 = tan (500 + 200) =
1  tan 500 tan 200
 tan 70 (1 – tan 50 tan 20 ) = tan 500 + tan 200
0 0 0

 tan 700 = tan 700 tan 500 tan 200 + tan 500 + tan 200
0 0 0 0 0 0 0 0
 tan 70 = cot 20 tan 50 tan 20 + tan 50 + tan 20 = 2 tan 50 + tan 20 .

P-2022-CBSE-P1-MATHEMATICS-TRI
15
Transformation Formulae:
(a) Transformation of products into sums or differences:
(i). 2 sin A cos B = sin (A + B) + sin (A – B)
(ii). 2 cos A sin B = sin (A + B) – sin (A – B)
(iii). 2 cos A cos B = cos (A + B) + cos (A – B)
(iv). 2 sin A sin B = cos (A – B) – cos (A + B)
The above formulae can be easily derived by taking the sum and the difference of the addition
and subtraction formulae.
(b) Transformation of sums or differences into products:
(i). sin (A + B) + sin (A – B) = 2 sin A cos B
CD C D
or, sin C + sin D = 2 sin cos
2 2
(ii). sin (A + B) – sin (A – B) = 2 cos A sin B
CD C D
or, sin C – sin D = 2 cos sin
2 2
(iii). cos (A + B) + cos (A – B) = 2 cos A cos B
CD C D
or, cos C + cos D = 2 cos cos
2 2
(iv). cos (A – B) – cos (A + B) = 2 sin A sin B
CD DC
or, cos C – cos D = 2 sin sin
2 2
CD DC
Here A – B = C and A + B = D  A = and B = .
2 2
sin(A  B)
(v). tan A + tan B = .
cos A cosB

Trigonometric Ratios of Multiple Angles:


(i). sin 2A = 2 sin A cos A
(ii). cos 2A = cos2 A – sin2 A = 2 cos2 A – 1 = 1 – 2 sin2 A
2 tan A
(iii). tan 2A =
1  tan2 A
3 0 0
(iv). sin 3A = 3 sin A – 4 sin A = 4 sin (60  A) sin A sin (60 + A)
(v). cos 3A = 4 cos3 A – 3 cos A = 4 cos (600  A) cos A cos (600 + A)
3 tan A  tan3 A
(vi). tan 3A = 2
= tan (600  A) tan A tan (600 + A).
1  3 tan A
Proof:
(i). We know that sin (A + B) = sin A cos B + cos A sin B.
Write A = B, so that sin 2A = sin A cos A + cos A sin A = 2 sin A cos A.
(ii). Since cos (A + B) = cos A cos B – sin A sin B,
cos (2A) = cos A cos A – sin A sin A = cos2 A – sin2 A.
(iii). We write A = B in
tan A  tanB
tan (A + B) = and get
1  tan A tanB
tan A  tan A 2 tan A
tan 2A =  .
1  tan A tan A 1  tan2 A

P-2022-CBSE-P1-MATHEMATICS-TRI
16
(iv). Here sin 3A = sin (A + 2A) = sin A cos 2A + cos A sin 2A
2 2 3
= sin A (1 – 2 sin A) + 2 sin A (1 – sin A) = 3 sin A – 4 sin A
= sin A [ 3  2 sin A] [ 3 + 2 sin A]
 3  3 
= 4 sin A   sin A    sin A 
 2   2 
= 4 sin A (sin2 600  sin2 A)
= 4 sin (600  A) sin A sin (600 + A).
(v). Here cos 3A = cos (A + 2A) = cos A cos 2A – sin A sin 2A
2 2 3
= cos A (2 cos A – 1) – 2 cos A (1 – cos A) = 4 cos A – 3 cos A
 3  3 2 2 0
= 4 cos A  cos A    cos A   = 4 cos A (cos A  sin 60 )
 2 2
  
0 0
= 4 cos (60  A) cos A cos (60 + A).
tan A  tan 2A
(vi). Also tan 3A =
1  tan A tan 2A
2 tan A
tan A  3 3
= 1  tan2 A  tan A  tan A  2 tan A = 3 tan A  tan A .
2 tan A 2 2
1  tan A  2 tan A 2
1  3 tan A
1  tan A
1  tan2 A

Illustration 22. If A + B = 45, show that (1+ tanA) (1+ tanB) = 2.

tan A  tanB
Solution: Since tan(A + B) =  1,
1  tan A tanB
tanA + tanB + tanA tanB + 1= 1 + 1
or tanA(1+ tanB) + (1+ tanB) = 2 or (1+ tanA) (1+ tanB) = 2.

Illustration 23. Find the values of (i) sin 18°, (ii) tan 15°.

Solution: (i) Let  = 18°. Then 2 = 36° = 90° – 54° = 90° – 3.
Now sin2 = 2sin cos and
sin(90° – 3) = cos3 = 4cos3 – 3cos.
Hence we have 2sin cos = cos (4cos2 – 3) = cos (1 – 4sin2)
2
or 2 sin = 1 – 4sin  (as cos  0)
2 2  4  16 1  5
 4sin  + 2sin – 1 = 0  sin =  .
2.4 4
5 1 5 1
Since sin > 0, we have sin = i.e. sin18° = .
4 4
(ii) Let  = 15°  2 = 30°.
2 tan  1
Since tan 2  2
 tan 30  ,
1  tan  3
2 2 3  12  4 2 3  4
tan  + 23 tan – 1 = 0  tan =    3 2.
2 2
For tan > 0, we ignore negative value
and get tan = 2 – 3 i.e. tan15° = 2 – 3.

P-2022-CBSE-P1-MATHEMATICS-TRI
17
Alternative solution:
2

tan 150 = tan (600 – 450) =


3 1

 3 1  
42 3
 2 3.
1 3 3 1 2
sin   sin 
Illustration 24. If ,  and  are in A.P., show that cot = .
cos   cos 


Solution: Since ,  and  are in A.P., 2 =  +   cot = cot
2
  
cos 2 cos sin
= 2  2 2 = sin   sin  .
   cos   cos 
sin 2 sin sin
2 2 2

Illustration 25. Show that sin 12°. sin48°.sin54° = 1/8.


1 1 1
Solution: L.H.S. = cos 36  cos 60  sin 54  cos 36 sin 54  sin 54 
2  2  2 

1 1
=  2 cos 36 sin 54  sin 54   sin 90  sin18  sin 54
4 4
1 1
= 1   sin 54  sin18    1  2 sin18 cos 36 
4 4
1  2 sin18  1  sin 36 cos 36 
= 1  cos18 cos 36   1  
4  cos18  4  cos18 
1  2 sin 36 cos 36  1  sin 72  1  1 1
= 1    1 

 1    R.H.S.
4  2 cos18  4  2 sin 72  4  2  8
Alternative Method
Let  = 12°.
1
L.H.S. = sin12 sin 48 sin 72 sin 54
sin 72

1 sin 3 12  sin 54 sin 36 sin 54 cos 36 1
 
  
 
=  R.H.S.
4 sin 72 8 sin 36 cos 36 8 cos 36 8

Exercise 3.
   
i) Show that cot   x  cot   x  = 1.
4  4 
ii) Given 3 tan tan = 1, show that 2cos( + ) = cos(  – ).
2
iii) Prove that 2sin  +4cos( + ) sin. sin + cos2(  + ) is independent of .
iv) If tan = b/a, then find the value of acos2 + bsin2.

Trigonometric Ratios of Submultiple of an Angle:


A A
(i). sin A = 2 sin cos
2 2
2 A 2 A 2 A 2 A
(ii). cos A = cos – sin = 2 cos – 1 = 1 – 2 sin
2 2 2 2

P-2022-CBSE-P1-MATHEMATICS-TRI
18
A
2 tan
(iii). tan A = 2
2 A
1  tan
2
A A
(iv). sin A = 3 sin – 4 sin3
3 3
A A
(v). cos A = 4 cos3 – 3 cos
3 3
A A
3 tan  tan3
(vi). tan A = 3 3
2 A
1  3 tan
3
A A
 sin  cos  1  sin A.
2 2
  A 3
A A  if 2n    2n 
or sin + cos =  1  sin A  4 2 4
2 2   otherwise
A A
 sin  cos  1  sin A
2 2
  A 5
A A  if 2n    2n 
or sin – cos =  1  sin A  4 2 4
2 2   otherwise
A  tan2 A  1  1
 tan =
2 tan A
A
The ambiguities of signs are removed by locating the quadrant in which lies.
2
A A
sin + cos is + ve
2 2
A A
sin – cos is + ve
2 2

A A A A
sin 2 + cos 2 is – ve sin 2 + cos 2 is + ve
/4
A A A A
sin 2 –- cos 2 is + ve sin 2 – cos 2 is – ve

A A
sin 2 + cos 2 is – ve
A A
sin 2 –- cos 2 is – ve

 | a cosA + b sinA |  a2  b2
Also cosA  sinA =    
2 sin  A   2 cos A  .
 4   4 
Notes:
A
 Any formula that gives the value of sin in terms of sinA shall also give the value of
2
n
n   -1 A
sine of .
2

P-2022-CBSE-P1-MATHEMATICS-TRI
19
 Any formula that gives the value of cos(A/2) in terms of cosA shall also give the value of
2n  A
cos of .
2
 Any formula that gives the value of tan(A/2) in terms of tanA shall also give the value of
n  A
tan of .
2

sin A  sin 2A  sin 4 A  sin 5 A


Illustration 26. Prove that = tan 3A.
cos A  cos 2A  cos 4A  cos 5 A

sin A  sin 2A  sin 4A  sin 5A (sin 5A  sin A)  (sin 2A  sin 4A)


Solution: LHS = =
cos A  cos 2A  cos 4A  cos 5A (cos 5A  cos A)  (cos 2A  cos 4A)
2 sin 3A cos 2A  2 sin 3A cos A
= = tan 3A.
2 cos 3A cos 2A  2 cos 3A cos A
Illustration 27. Prove that cos 2 cos 2 + cos2 ( + ) – cos2 ( – ) = cos (2 + 2).

Solution: LHS = cos 2 cos 2 + cos2 ( + ) – cos2 ( – )


= cos 2 cos 2 + 1 – sin2 ( + ) – 1 + sin2 ( – )
= cos 2 cos 2 + sin2 ( – ) –sin2 ( + )
= cos 2 cos 2 – sin ( +  +  – ) sin ( +  –  + )
= cos 2 cos 2 – sin 2 sin 2 = cos (2 + 2).
0 0
1 1
Illustration 28. Find the values of sin67 and cos67 .
2 2
0 0
1 1
Solution: We have sin67 + cos67 = 1  sin1350
2 2
1 1
= 1 = 42 2 … (1)
2 2
0 0 0
1 1 1
and sin67 – cos67 = 1  sin1350 (because 45  67  135)
2 2 2
1
= 42 2 … (2)
2
From (1) and (2), we get
0
1 1
sin67 =  4  2 2  4  2 2  and
2 4  
0
1 1
cos67 =  4 2 2  4 2 2.
2 4  

Exercise 4.
i) Find the value of cos22(1/2)º.
 1e  cos   e
ii) If tan  tan , prove that cos = .
2 1e 2 1  e cos 
   
iii) Prove that (cos – cos)2 + (sin – sin)2 = 4sin2  .
 2 
 2   4 
iv) Prove that sin + sin     + sin     = 0.
 3   3 

P-2022-CBSE-P1-MATHEMATICS-TRI
20
Identities
A trigonometric equation is an identity if it is true for all values of the angles involved. A given
identity may be established by reducing either side equal to the other one, or reducing each side
to the same expression, or any convenient modification of these.

(a) For any angles A, B, C:


sin (A + B +C) = sinA cosB cosC + cosA sinB cosC + cosA cosB sinC – sinA sinB sinC
cos (A + B +C) = cosA cosB cosC- cosA sinB sinC - sinA cosB sinC – sinA sinB cosC
tanA+tanB+tanC-tanA tanB tanC
tan (A+B+C)= ;
1-tanA tanB-tan BtanC-tanA tanC
cotAcotBcotC-cotA-cotB-cotC
cot (A+B+C)=
cotA cotB  cot BcotC  cotA cotC-1

(b) If A, B, C are the angles of a triangle ( or A + B + C =  ), then


 sinA cosB cosC + cosA sinB cosC + cosA cosB sinC = sinA sinB sinC
 cosA sinB sinC + sinA cosB sinC + sinA sinB cosC = 1 + cosA cosB cosC
 tanA + tanB + tanC = tanA tanB tanC
 cotB cotC + cotC cotA + cotA cotB = 1
B C C A A B
 tan tan +tan tan +tan tan =1
2 2 2 2 2 2
A B C A B C
 cot  cot  cot  cot cot cot
2 2 2 2 2 2
 sin2A + sin2B + sin2C = 4sinA sinB sinC
 cos2A + cos2B + cos2C = -1-4cosA cosB cosC
 cos2A + cos2B + cos2C = 1 - 2cosA cosB cosC
A B C
 sin A  sinB  sin C  4 cos cos cos
2 2 2
A B C
 cos A  cosB  cos C  1  4 sin sin sin
2 2 2
Illustration 29. If A, B and C are the angles of a triangle, show that
A B C
tan2  tan 2  tan2 1.
2 2 2

A B C 
Solution: We have    , so that
2 2 2 2
A B
tan  tan
 A B  C 2 2  1
tan     tan    
 2 2 2 2 A B C
1  tan tan tan
2 2 2
A B B C C A
 tan tan  tan tan  tan tan  1
2 2 2 2 2 2
2 A B C 1  A A B
 tan  tan2  tan2  1 =  2 tan2
   2 tan 2 tan 2 
2 2 2 2 2
2 2 2
1  A B  B C  C A 
=  tan  tan    tan  tan    tan  tan    0 .
2  2 2  2 2  2 2  

P-2022-CBSE-P1-MATHEMATICS-TRI
21
sin 2A  sin 2B  sin 2C
Illustration 30. If A + B + C = , prove that = 8cos A/2 cosB/2 cosC/2.
cos A  cos B  cos C  1

2 sin  A  B  cos  A  B   2 sin C cos C


Solution: L.H.S. =
A B A B C
2 cos cos  2 sin2
2 2 2
2 sin    C  cos  A  B   2 sin C cos     A  B  
=
  C A B C  A B
2 cos    cos  2 sin sin   
2 2 2 2 2 2 
C C
sin C cos  A  B   sin C cos  A  B  2 sin 2 cos 2  cos  A  B   cos  A  B  
= 
C A B C A B C A B A B
sin cos  sin cos sin  cos  cos 
2 2 2 2 2 2 2 
C 2 sin A sinB A B C
= 2cos .  8 cos cos cos  R.H.S.
2 A B 2 2 2
2 sin sin
2 2
Alternative Method:
Using directly the corresponding formulae, we get
4 sin A sinB sin C A B C
L.H.S. = = 8 cos cos cos .
A B C 2 2 2
4 sin sin sin
2 2 2

Illustration 31. Find the maximum and minimum values of a cos + b sin.

Solution: Let a = r cos, b = r sin so that a2 + b2 = r2


 r = a2  b2 .
Also a cos + b sin = r(cos cos + sin sin) = r cos(  ).
Now the maximum value of cos(  ) is 1 and minimum value is 1.
Hence  r  r cos(  )  r
  a2  b2  a cos + b sin  a2  b2 .
Hence the maximum value = a2  b2 and minimum value =  a2  b2 .

Illustration 32. Simplify the product cosA cos2Acos22A …. cos2n–1A.

Solution: We write cosAcos2A….cos2n-1A


1
=  (sinAcosA)cos2A…. cos2n–1A
2 sin A
1 n–1
= 2 (2 sin 2A  cos 2A) …. cos2 A
2 sin A
1 n–1
= 2 (sin 4A  cos 4A) .... cos2 A
2 sin A
sin 2n A
= (continuing (n  1) times).
2n sin A

P-2022-CBSE-P1-MATHEMATICS-TRI
22
Exercise 5:
2 4 8 16 1
i) Prove that cos .cos cos .cos = .
15 15 15 15 16
 
ii) Prove that 5cos  + 3cos     + 3 lies between –4 and 10.
 3
iii) Find minimum and maximum value of y = 7cos + 24sin.
 3 5 7 3
iv) Prove sin 4  sin 4  sin 4  sin 4  .
16 16 16 16 2

Graphs of Trigonometric Functions:


We have seen that all trigonometric function are periodic. Since sin (2 + x) = sin x, cos (2 + x) =
cos x, and tan ( + x) = tan x, the period of sine and cosine functions is 2 where as the period of
2 
tangent function is . Moreover, period of sin ax or cos ax is and that of tan ax is .
a a

Illustration 33. Draw the graph of y = sin x.

Solution: Since sin x is periodic with period 2, it is sufficient to sketch its graph only for
0  x  2. It can be easily extended by repeating it over the intervals of length
2. We have the following table:
x 0    2 5  7 4 3 5 11 2
6 3 2 3 6 6 3 2 3 6
sin x 0 1 3 1 3 1 0 1 3 1 3 1 0
  
2 2 2 2 2 2 2 2

-/2 3/2
-
x
-2 -3/2 0 /2  2

Similarly we may draw the graph of y = cos x.

Illustration 34. Draw the graph of y = tan x.

Solution : tan x is periodic with period . As x increases from 0 to /2, tan x keeps
increasing from 0 to . As x crosses the value /2, tan x becomes negative and
increases from - to 0 with x increasing from /2 to . We have the following
table:
   2 3 5 
x 0
6 4 3 3 4 6
1 1
tan x 0 1 3  3 -1  0
3 3

P-2022-CBSE-P1-MATHEMATICS-TRI
23
y

- -/2 /2  3/2


O 2 x

Note:
  2  
For y = b sin ax, y = b cos ax, a  0 we find that sin a  x     sin  ax  2   sin ax
  a 
  2  
and cos a  x     cos  ax  2   cos ax . The period of both of these
  a 
2
function is . The amplitude of both the curves y = b sin ax and y = b cos ax is |b|. The
a
graph is magnified if |b|  1and is contracted if |b|  1.

Miscellaneous Exercise:

4 3 sin A  cos A
i) If sin A = , find the value of .
5 4 cos ecA  3 tan A
n tan A
ii) If tan B = , prove that tan (A  B) = (1  n) tan A.
1  (1  n ) tan 2 A
   
iii) Prove that sec   A  sec   A   2 sec 2 A .
 4   4 
1 1 xy
iv) If cos x + cos y = , sin x + sin y = , find the values of tan   and tan (x + y).
8 4  2 
v) If sinx + siny = 3 (cosy – cosx) show that sin3x + sin3y = 0
cos  A  B  cos  C  D 
vi) If   0 then prove that tanA.tanB. tanC tanD = – 1
cos  A  B  cos C  D 
sin     1m    
vii) If  , prove that tan     tan     = m.
cos     1 m  4   4 
3
viii) Prove that sin 200 sin 400 sin 800 = .
8
 9 3 5
ix) The value of 2 cos  cos  cos  cos is equal to
13 13 13 13
(A) 2 (B) 0
(C) 1 (D) 3

P-2022-CBSE-P1-MATHEMATICS-TRI
24

x) If sin = nsin( + 2) then tan( + ) is equal to


1n 1n
(A) tan  (B) tan 
2n 1n
1n
(C) tan (D) tan 
1n


xi) The value of 2  2  2  2 cos 8 where 0 <  < is equal to
8
(A) 2cos (B) cos
(C) 2sin (D) 2cos

 5  3
xii) If tan  and tan  , then the value of cos( + ) is
2 2 2 4
364 627
(A)  (B) 
725 725
240
(C)  (D) none of these
339

1  tan2 15 
xiii) The value of is
1  tan 2 15 
(A) 1 (B) 3
3
(C) (D) 2
2

xiv) If cos + sin = 2 , then sin  cos is equal to


(A) 0 (B) 1
(C) 1 (D)  2

 3 5 7 9 11 13
xv) The value of sin  sin  sin  sin  sin  sin  sin is
14 14 14 14 14 14 14
1 1
(A) (B)
16 64
1
(C) (D) none of these
128

P-2022-CBSE-P1-MATHEMATICS-TRI
25
ANSWERS TO EXERCISES

Exercise 1.

iii) x=y

Exercise 2.
i) 1

Exercise 3.
iv) a

Exercise 4.
1
i) 2 2
2

Exercise 5.
iii) minimum = 25
maximum = 25

Miscellaneous Exercise:

63 3 24
i) iv) ,
20 4 7
ix) B x) D
xi) A xii) B
xiii) C xiv) A
xv) B

P-2022-CBSE-P1-MATHEMATICS-TRI
26
SOLVED PROBLEMS

Subjective:

Level  0

   3  5  7  1
Problem 1. Prove that 1+cos 1+cos 1+cos 1+cos  .
 8  8  8  8  8

5   
Solution: Writing cos  cos      sin
8  2 8  8
3    7   
cos  cos     sin , cos  cos       cos
8 2 8 8 8  8 8
        
 L.H.S. =  1  cos   1  cos   1  sin   1  sin 
 8  8  8  8
2
     1  1 1 1
=  1  cos2   1  sin2   sin2 cos2   sin  = .   R.H.S.
 8  8 8 8 4 4 4 2 8

A  B m 1 BA
Problem 2. If cosA = m cosB, then prove that cot  tan .
2 m 1 2

Solution: We have cosA = m cosB


cos A m cos A  cos B m  1
   
cos B 1 cos A  cos B m  1
A B BA
2 cos cos
 2 2  m 1
A B BA m 1
2 sin sin
2 2
A  B  m  1 BA
 cot   tan .
2  m  1 2


Problem 3. Let 0 < A, B < satisfying the equation 3sin2A + 2sin2B = 1 and
2

3sin2A – 2sin2B = 0. Prove that A  2B  .
2

Solution: From the second equation, we have


3
sin 2B  sin 2A .…(1)
2
and from the first
3sin2A = 1– 2sin2B = cos2B. .…(2)
Now cos (A + 2B) = cosA  cos2B – sinA  sin2B
2 3
= 3cosA  sin A –  sinA  sin2A
2
= 3cosA  sin A – 3sin2A  cosA = 0
2

P-2022-CBSE-P1-MATHEMATICS-TRI
27
 3
 A + 2B = or .
2 2
  
Given that 0 < A < and 0 < B <  0 < A + 2B <  + .
2 2 2

Hence A + 2B = .
2

 3 5 7 9 11 13 1
Problem 4. Pr ove that sin .sin .sin .sin .sin .sin sin  .
14 14 14 14 14 14 14 64

Solution: We know that


  6 3  4 5  2 9  2
  ,   ,   ,  
14 2 14 14 2 14 14 2 14 14 2 14
11  4 13  6
  ,   .
14 2 14 14 2 14
2 4 6
 L.H.S  cos2 .cos2 cos2
14 14 14
1   2 3
 4 sin2 cos2 cos2 cos2
 7 7 7 7
4. sin2
7
4 2 2 3 1 4 3
 . sin2 . cos2 cos2  .4 sin2 cos2
 7 7 7  7 7
16. sin2 64. sin2
7 7
1  4  2 3
 4 sin2     cos 7
  7 
64. sin2
7
1 3 3 1 6
 4 sin2 cos2  . sin2
 7 7  7
64. sin2 64.sin2
7 7
1  1
 . sin2  .
 7 64
64. sin2
7

Level  I

Problem 5. Prove that tanA + 2tan2A + 4tan4A + 8cot8A = cotA.

Solution: L.H.S. = tanA + 2tan2A + 4tan4A + 8


1  tan 2
4A 
2 tan 4A

 tan A  2 tan 2A  4 cot 4A  tan A  2 tan 2A  4


1  tan 2
2A 
2 tan 2A
2
1  tan A
 tan A  2 cot 2A  tan A 
tan A
= cot A = R.H.S.

P-2022-CBSE-P1-MATHEMATICS-TRI
28
Problem 6. If a cos2 + b sin2 = c has  and  as its solutions, then prove that
2b c a
tan + tan = , tantan =  .
c  a  c a

Solution: We have a cos2 + b sin2 = c


 a(cos2 - sin2) + 2bsincos = c
 a(1 - tan2) +2b tan = c sec2 = c(1+ tan2)
 tan2(c + a) -2btan + c - a = 0. … (1)
This has tan and tan as its roots.
2b c a
 tan + tan = , tantan =  .
 c  a  c a

Problem 7. For all  in [0, /2], show that cos(sin ) > sin (cos ).

 1 1 
Solution: We have cos  sin   2 cos   sin  
 2 2 
     
 2 cos cos   sin sin    2 cos    
 4 4   4 
 
 cos + sin  2   cos < - sin
2 2
 
 sin(cos) < sin   sin    sin(cos) < cos(sin)
 2 
 cos(sin) > sin(cos).

Alternative:
For 0  x  /2, x  sin x. ... (1)
Replace x by cos 
 cos  sin(cos).
Take cosine of both sides of (1) and write  for x
 cos  cos(sin). Hence sin(cos) < cos(sin).

Problem 8. Find the smallest positive number p for which the equation
cos (p sin x) = sin (p cos x) has a solution x  [0, 2].

Solution: We have cos (p sin x) = sin (p cos x)


 
 sin   p sin x   sin  p cos x 
 2 

  p sin x  p cos x
2

  p cos x  p sin x
2
  1 1 
  cos x  sin x  2  cos x  sin x 
2p  2 2 
  
  cos  x    1
2 2p  4

P-2022-CBSE-P1-MATHEMATICS-TRI
29
 
 p . Hence the smallest value of p =
2 2 2 2
 7
at x  , .
4 4

Problem 9. Let cosA + cosB + cosC = 3/2 in a triangle ABC. Show that the triangle is
equilateral.

Solution: In a triangle A + B + C = 
A B A B 3
 cosA + cosB + cosC = 2 cos cos  cosC 
2 2 2
  C  A  B C 3
 2 cos   cos  1  2sin2 
 2 2  2 2 2
C C A -B
 4 sin2 – 4sin cos +1 = 0. …..(1)
2 2 2
Now sin(C/2) is real
A B
 16 cos2  16  0
2
A B A B 2 A B
 cos2 – 1  0  cos2  1 . But cos 1
2 2 2
A -B
 cos2 =1
2
 A = B. …(2)
Similarly it can be shown that B = C, C = A. Hence the triangle is equilateral.


Problem 10. Prove that 1 + cot  cot for 0 <  < . Find  when equality sign holds.
2

 
2 tan cot 2
1
Solution: Since tan = 2
 cot = 2 .
2  
1  tan 2 cot
2 2

cot 2  1
 2 
Now 1 + cot  – cot = 1   cot
2  2
2 cot
2
2
    θ 
2 cot  cot 2  1  2 cot 2   cot  1
2 2 2 =  2 
=  0 for 0 <  < 
 θ
2 cot 2cot
2 2

 1 + cot  cot .
2
 
Equality holds when cot  1  0   = .
2 2
Alternative Solution:

P-2022-CBSE-P1-MATHEMATICS-TRI
30
 c os  cos  / 2 cos  2 cos2  / 2
1 + cot   cot= 1   1 
2 sin  sin  / 2 sin  2 sin  / 2 cos  / 2
cos  1  cos 
=1+  = 1  cosec   0.
sin  sin 

Level  II

Problem 11. If a 2  b2  2ab cos   1 , c 2  d 2  2cd cos   1 , sin  0 and


(ac  bd )  ad  bc  cos   0 . Prove that a 2  c 2  cos ec 2 .

Solution: The given relations imply


2
 b  acos   a2  a2 cos2   1
2
  b  acos    1  a2 sin2  ….(1)
2 2 2
 d  c cos    1  c sin  … (2)
Multiplying (1) and (2), we get
2 2
 b  acos    d  c cos    1  a2 sin2 1  c 2 sin2   … (3)
since  b  acos   d  c cos    bd   ad  bc  cos   ac cos2 
= bd  ac   ad  bc  cos   ac sin2   ac sin2 

  
Hence 1  a2 sin2  1  c 2 sin2   a2 c 2 sin4 
 a2  c 2  cosec 2 

2
Problem 12. If  = , prove that tan  tan 2 + tan 2 tan 4 + tan 4 tan  = -7.
7

2
Solution: Given  =  cos ( + 2+ 4) = 1
7
 cos ( + 2) cos 4 -sin ( + 2) sin 4 = 1
 cos  cos 2 cos 4 -sin  sin 2 cos 4 -sin  cos 2 sin 4 -cos  sin 2
sin 4 = 1
 1 –tan  tan 2 -tan  tan 4 -tan 2 tan 4 = sec  sec 2 sec 4 ….(1)
Again; 7 = 2  sin 8 = sin (2 + )
 8 sin  cos  cos 2 cos 4 = sin 
 sec  sec 2 sec 4 = 8
Putting in (1); we get
tan  tan 2 + tan  tan 4 + tan 2 tan 4 = 7

Problem 13. If A, B, C are the angles of a triangle, then find the minimum value of
B C CA AB
cos cos cos
2  2  2 .
B C CA AB
cos cos cos
2 2 2

BC BC BC


cos 2sin cos
2  2 2 sinB  sinC sinB sinC
Solution: =  
BC BC BC sin A sin A sin A
cos 2cos sin
2 2 2

P-2022-CBSE-P1-MATHEMATICS-TRI
31
 sin A sinB 
Hence total sum is   sinB  sin A 
sin A sinB
Now  2 (AM  GM)
sinB sin A
 given expression  6 and = 6 when A = B = C.

6 6
Problem 14. Consider the expression 3 sin x
 3 cos x
. Find the minimum value.

6 6
Solution: 3sin x and 3cos x are positive numbers .
And A.M.  G. M.
6 6
3sin x
 3cos x
6
x  cos6 x
 3sin .... (1)
2
3
1 sin2 2x
= 2 3 4

1
6 6 1/8
 3sin x
 3cos x
 2  3 4 = 2.3

Alternate:
6 6
Clearly the equality (1) holds for 3sin x
= 3cos x

6
 1  1
 sin6 x = cos6 x =    .
 2 8

tan       tan 
Problem 15. If  , prove that sin2 + sin2 + sin2 = 0.
tan       tan 

Solution: The given equation may be written as


sin        cos        sin  cos 
=
sin        cos        sin  cos 
sin 2  sin  2  2  sin  cos 
 
sin2  sin  2  2  sin  cos 
Using compodendo dividendo, we get
sin 2 sin  cos   sin  cos  sin     
 = .
sin  2  2  sin  cos   sin  cos  sin     
sin 2 sin     
 
2 sin      cos      sin     
 sin2 + 2 sin( + ) cos ( - ) = 0
 sin2 + sin2 + sin2 = 0 .

P-2022-CBSE-P1-MATHEMATICS-TRI
32

Objective:

Level  0

True / False

Problem 1. If in ABC sin2A + sin2B + sin2C = 2, then the triangle is always right angled.
Above statement is true or false.

Solution: True
sin2A + sin2B + sin2C = 2
2 2 2
sin A + sin B  1 + sin C  1 = 0
sin2A  cos2B  cos2C = 0
2cosAcosBcosC = 0
 either A = 90° or B = 90° or C = 90°.

Problem 2. If the sides of a right angled triangle are in G.P then the cosines of the acute
 5 1 5 1
angles of the triangle are , .
2 2

Solution: False
a, b, c are in G.P  b2 = ac A
and c2 = a2 + b2
 c2 = a2 + ac
c
2
 a  a  b
      1 0
 c  c 
a 1  5 B
 =cos B = a C
c 2
5 1
 cos B = (B is acute)
2
5 1
cos A = sin B = 1  cos2 B =
2

3
Problem 3. If  < 2 < , then 2  2  2 cos 4 is – 2sin
2

Solution: False
2  2(1  cos 4)  2  2 | cos 2 |
= 2(1  cos 2)
 3
= 2 | sin | = 2sin as  .
2 4

Problem 4. The number of integral values of a for which the equation cos2x + a sinx = 2a – 7
possesses solutions is 5

Solution: True
The given equation may be written as

P-2022-CBSE-P1-MATHEMATICS-TRI
33
2
2 sin x – a sinx + 2a – 8 = 0,
a  a  8  a4
so that sinx =  sinx = .
4 2
a4
This is possible only if 1   1 or 2  a  6.
2

Problem 5. The minimum value of cos(cosx) is 1

Solution: False
cosx varies from –1 to 1 for all real x.
Thus cos(cosx) varies from cos1 to cos0.
 minimum value of cos(cosx) is cos1.

Fill in the Blanks

Problem 6. If sinx = cos2x, then cos2x (1 + cos2x) is _______

Solution: 1
cos2x (1 + cos2x) = cos2x + cos4x = cos2x + sin2x = 1.

x x
Problem 7. The maximum value of 4sin2 x + 3cos2x + sin  cos is ____
2 2

Solution: 4 2
Maximum value of 4sin2x + 3cos2x i.e. sin2x + 3 is 4 and that of
x x
sin + cos = 2 , both attained at x = /2.
2 2
Hence the given function has maximum value 4  2 .

Problem 8. If  and  are the solutions of sin2x + asinx + b = 0 as well as that of


cos2x + ccosx + d = 0, then sin( + ) is equal to _____

2ac
Solution:
a  c2
2

According to the given condition,


sin + sin = –a and cos  + cos = – c .
     
2 sin cos  a and 2 cos cos  c
2 2 2 2
 a
 tan 
2 c

2 tan
2 2ac
 sin(  )   2 2
.
2    a  c
1  tan
2

P-2022-CBSE-P1-MATHEMATICS-TRI
34
Level  I

MCQ Single Correct

Problem 9. If in a triangle ABC, C =90°, then the maximum value of sinA sinB is
1
(A) (B) 1
2
(C) 2 (D) None of these

Solution: A
1 1
sinA sinB =  2 sin A sinB =  cos(A  B)  cos(A  B)
2 2
1 1 1
= cos(A  B)  cos 90 = cos(A  B) 
2 2 2
1
 Maximum value of sinA sinB = .
2

Problem 10. Maximum value of the expression 2sinx + 4cosx + 3 is


(A) 2 5 + 3 (B) 2 5 - 3
(C) 5 + 3 (D) none of these

Solution: A
Maximum value of 2sinx + 4cosx = 2 5 . Hence the maximum value of
2sinx + 4cosx +3 is 2 5  3 .

Problem 11. If sin = 3sin( + 2), then the value of tan ( + ) + 2tan is
(A) 3 (B) 2
(C) 1 (D) 0
Solution: D
Given that sin  = 3sin ( + 2)  sin ( +   ) = 3sin ( +  + )
 sin ( +)cos –cos( + ) sin =3sin ( + ) cos + 3cos ( + ) sin
 –2sin ( + ) cos = 4cos ( + ) sin
 sin(   ) 2 sin 
   tan(+) + 2tan = 0.
cos(   ) cos 

Problem 12. If sin sin  cos cos + 1 = 0, then the value of cot tan is
(A) –1 (B) 0
(C) 1 (D) none of these
Solution: A
Given sin sin – cos cos + 1 = 0
 cos( + ) = 1  sin( + ) = 0
 sin cos + cos sin = 0  cot tan = -1.
Problem 13. If cos 28 + sin28 = k3, then cos17 is equal to
k3 k3
(A) (B) 
2 2
k3
(C)  (D) none of these
2

P-2022-CBSE-P1-MATHEMATICS-TRI
35
Solution: A
cos17 = cos (45 – 28) = cos45 cos28 + sin45 sin28
cos 280  sin 280 k 3
=  .
2 2

Level  II

MCQ Single Correct

Problem 14. If sin, sin and cos are in G.P, then roots of the equation x2 + 2x cot + 1 = 0
are always
(A) equal (B) real
(C) imaginary (D) greater than 1

Solution: B
sin, sin, cos are in G.P.
 sin2 = sin cos
 cos2 = 1 – sin2  0
Now, the discriminant of the given equation is
4cot2 – 4 = 4 cos2  cosec2  0
 Roots are always real.

 2 ( n  1)
Problem 15. If S  cos 2  cos 2    cos 2 , then S equals
n n n
n 1
(A) ( n  1) (B) ( n  1)
2 2
1 n
(C) ( n  2 ) (D)
2 2

Solution: C
 2 
S  cos2  cos2    cos2 (n  1)
n n n
1 2 4 6 
1  cos  1  cos  1  cos    1  cos 2(n  1) 
2 n n n n
n1
1 2k  1 1
= n  1 
2   cos
k 1
 = n  1  1   n  2  .
n  2 2

Problem 16. If in a triangle ABC, sin2A + sin2B + sin2C = 2, then the triangle is always
(A) isosceles triangle (B) right angled
(C) acute angled (D) obtuse angled

Solution: B
sin2A + sin2 B + sin2C = 2
2 2 2
 sin A – cos B + sin C = 1
or sin2C – (cos2B – sin2A) = 1
or - cos(B + A) cos(B – A) = cos2C
2
 cosC cos(B – A) = cos C
 cosC = 0 i.e. C = /2 or B – A =  C  B = /2 or A = /2.

P-2022-CBSE-P1-MATHEMATICS-TRI
36
Problem 17. Let k = 1, then 2 sin2k + 4 sin4k + 6 sin6k + … + 180 sin180k is equal to
(A) 90cosk (B) 90tan89
(C) 90 tank (D) 90cot89

Solution: B
Let y = 2 sin2 + 4 sin4 + … + 178 sin178
 y = 178 sin178 + 176 sin176 + … + 2 sin2
 2y = 180(sin2 + sin4 + … + sin178)
sin89
 y = 90. sin  90  = 90tan89.
sin1

Problem 18. If 0  A, B, C   and A + B + C =  than the minimum value of sin3A + sin3B +


sin3C is
3 3
(A) – 2 (B)
2
(C) 0 (D) none of these
Solution: A
Let y = sin3A + sin3B + sin3C for sin3A to be non positive
2
we have 2 < 3A < 3  <A<
3
since A + B + C =   all of sin3A, sin3B, sin3C can’t be negative.

Let us take sin3A = – 1  A =  sin3A = – 1, sin3B = – 1 and sin3C = 0 is
2
possible
 the minimum value is – 2.
Hence (A) is the correct answer.

Problem 19. If , , ,  are the smallest positive angles in ascending order of magnitude
which have their sines equal to the positive quantity k, then the value of 4 sin
   
 3 sin  2 sin  sin is equal to
2 2 2 2
(A) 2 1  k (B) 2 1  k
(C) 2 k (D) none of these

Solution: B
Given  <  <  <  also sin  = sin  = sin  = sin  = k
and , , ,  are smallest positive angles
  =   ,  = 2 + ,  = 3  
as sin  = sin  and  > 
sin  = sin  and  > 
sin  = sin  and  > .
Putting these values in the given expansion, we have given expression
  
= 2  sin  cos   2 1  sin   2 1  k .
 2 2

Problem 20. If 3sin + 5cos = 5, then the value of 5sin – 3cos is equal to
(A) 5 (B) 3
(C) 4 (D) none of these

Solution: B

P-2022-CBSE-P1-MATHEMATICS-TRI
37
2
3sin = 5(1 – cos) = 5  2sin /2
 tan/2 = 3/5
 2 
2 tan  1  tan 2 
2 
5sin – 3cos = 5  3
2  2 
1  tan 1  tan
2 2
3 3  1  9 
2 
5   25 
= 5  3
9 9
1 1
25 25

n
Problem 21. Let n be an odd integer. If sin n = b r sinr  , for every value of , then
r 0

(A) b0 = 0, b1= n (B) b0 = 1, b1= 3


(C) b0 = –1, b1= – n (D) none of these

Solution: A
Writing  = 0 on both sides, we get b0 = 0.
n
sinn r 1
Now,
sin 
  b sin
r 1
r .

Making   0, we get n = b1

MCQ Multi Correct

Problem 22. If in a ABC sinC  cosC  sin  2B  C   cos  2B  C   2 2 , then ABC is


(A) equilateral (B) isosceles
(C) right angled (D) obtuse angled

Solution: B, C
We have
sinC  cosC  sin  2B  C   cos  2B  C   2 2
 sin  2B  C   sinC   cos C  cos  2B  C    2 2
 sin 180  A  cosB  sin 180  A  sinB  2
 sin A  cosB  sinB   2
 1 1 
 sin A  cosB  sinB   1
 2 2 
 
 sin A sin   B   1, it is possible only when
4 
  
sin A  1& sin   B   1
4 
 A  90 & B  45 then C  45

P-2022-CBSE-P1-MATHEMATICS-TRI
38
Problem 23. If sin22.50   then
2 2 2 2
(A)   (B)  
2 2
1
(C)  2  (D) 4 2  2  2
2

Solution: C, D
2 2
sin22.50   
2
2  2 1
 2  , 2 
4 2

Problem 24. For a positive integer n, let


   
  2 
 
fn      tan    . 1  sec  1  sec  2   1  sec  4   ......... 1  sec 2n   . Then
 
     
(A) f2    1 (B) f3    1
 16   32 
     
(C) f4    1 (D) f5   1
 64   128 

Solution: A, B, C, D
 sin   / 2  2cos 2   / 2 
tan   . 1  sec    .
2 cos   / 2  cos 
2 sin   / 2  cos   / 2 
sin 
=  tan 
cos  cos 
Repeating this process again and again, we get
 
fn     tan 2n  (1)
       
This  (A) f2    tan  22   tan    1
 16   16  4
     
 
(B) f3    tan  23.   tan    1 etc.
 32   32  4

Assertion / Reasoning
(A) Statement–1 is True, Statement–2 is true; Statement–2 is a correct explanation for
Statement–1
(B) Statement–1 is True, Statement -2 is true; Statement–2 is NOT a correct
explanation for Statement–1
(C) Statement –1 is True, Statement –2 is False
(D) Statement –1 is False, Statement –2 is True


Problem 25. Statement – 1: If A  0, B  0 and A  B  , then the maximum value of
3
tan A.tanB is 3.

P-2022-CBSE-P1-MATHEMATICS-TRI
39
tan A  tanB
Statement – 2: tan  A  B   , None of the angles A, B and (A+B)
1  tan A.tanB

should be an odd multiple of .
2

Solution: D
Using maximisation principle for symmetrical functions the value of tan A.tanB
will be maximum when A  B .
 1
So; For A  B  , the maximum value of tan A.tanB will be .
6 3

   
Problem 26. Statement – 1: The value of 5cos   3cos      3  lies in [ 4 ,10]
  3  
Statement – 2: The value of  a cos x  b sin x  C  lies in [C  a2  b2 ,
C  a2  b2 ]

Solution: A
 
5 cos   3 cos      3
 3 
3 3 3
 5 cos   cos   sin   3
2 2
13 3 3
 cos   sin   3
2 2
The value lies between
2 2 2 2
 13   3 3   13   3 3 
3       and 3       
 2   2   2   2 
i.e 4 and 10
Both statements I and II are true and statement II is a correct explanation of
statement I.

Comprehension – I

If A + B +C=1800, then
(i) sin2A  sin2B  sin2C = 4 sin A sin B sin C
A B C
(ii) cos2A  cos2B  cos2C  4cos cos cos
2 2 2
A B C
(iii) cos2A  cos2B  cos2C  4cos cos cos
2 2 2
A B C
(iv) cos A  cosB  cosC = 1  4 sin sin sin
2 2 2
(v) tan A  tanB  tanC  tan A tanB tanC
(vi) cotBcot C  cot Ccot A  cot A cot B  1
A B C A B C
(vii) cot  cot cot  cot cot cot
2 2 2 2 2 2

P-2022-CBSE-P1-MATHEMATICS-TRI
40
Problem 27. If tan A + tan B + tanC= tan A. tan B. tan C, then
(A) A,B C, must be angles of a triangle
(B) The sum f any two of A, B ,C of equal to the third.
(C) A + B + C must be an integral multiple of 
(D) None of these.

Solution: A
tan A  tanB  tanC  tan A tanB tanC
tan(A  B  C) 
1  tan A tanB  tanB tanC  tanC tan A
 A  B  C  n

Problem 28. In a triangle ABC, whose angles are acute and +ve such that A+B+C =  and
A B C
cot cot cot  K,then
2 2 2
(A) K  3 (B) K  3 3
(C) K  3 3 (D) none of these.

Solution: B
 A B C  S  S3
tan      1
 2 2 2  1  S2
   S  S2
 tan    1  S2  1
 2  1  S2
  
 tan 2   
 
A B B C C A
 tan tan  tan tan  tan tan  1
2 2 2 2 2 2
C A B A B C
 cot  cot  cot  cot cot cot  K
2 2 2 2 2 2
A B C
 cot  cot  cot  k
2 2 2
But we know that A  G
A B C
cot  cot  cot 1/3
 2 2 2   cot A cot B cot C 
 
3  2 2 2
1/3 2/3 2
K  3K  K  3  K  27
k3 3

Problem 29. If A, B, C are acute positive angles such that A + B + C =  and cot A cot B cot
C = K, then
1 1
(A) K  (B) K 
3 3 3 3
1 1
(C) K  (D) K 
9 3

Solution: A
We Know that if A+ B +C= 
tan A  tanB  tanC  tan A tanB tanC...(1)
Since A.M.  G.M,

P-2022-CBSE-P1-MATHEMATICS-TRI
41
tan A  tanB  tanC
  (tan A tanB tanC)1/3
3
 tan A  tanB  tanC  3(tan A tanB tanC)1/3
 (tan A tanB tanC)  3(tan A tanB tanC)1/3
 (tan A tanB tanC)3  3(tan A tanB tanC)1/3 (by(1))
3
  tan A tanBC   27(tan A tanB tanC)
 (tan A tanB tanC)2  27
 tan A tanB tanC  3 3
1 1
 cot A cotB cot C  K 
3 3 3 3

Comprehension – II

x, y, z are respectively the sines and p, q, r are respectively cosines of the angles , ,  which
2
are in A.P. with common difference .
3

Problem 30. x + y + z is equal to


(A) 0 (B) 1
(C) p + q + r (D) none of these
Solution: A, C
2  4 
x + y + z = sin + sin   

  sin    
 3   3 
4  2 
= sin + sin   

  sin    
 3   3 
 2   2   2 
= 2 sin     cos    sin    
 3   3   3 
 2  1   2 
= – 2 sin        sin     =0
 3  2   3 
similarly, p + q + r = 0.
Problem 31. yz + zx + xy is equal to
(A) p + q + r (B) x + y + z
3 3
(C) – (D) –
4 8

Solution: C
 2   2   4   4 
yz + zx + xy = sin  sin      sin     sin      sub     sin 
 3   3   3   3 
1  2   2   2   6   4   4  
= cos    cos  2    cos    cos  2    cos    cos  2  
2   3   3   3   3   3   3  
1 3    3
=     2cos 2 cos    cos 2     .
2 2  3  4

P-2022-CBSE-P1-MATHEMATICS-TRI
42
Matrix Match Type

Problem 32. Match the following:


List  I ListII
(P) If sin   cos    cos   sin   then the value of (1) 0
 
cos     is
 4
(Q) 3 (2) 1
If tan   cos    cot   sin   ,0    then
4
 
sin2     is
 4
(R)  3  (3) 1
The value of  sin6   cos6   sin2 2  is
 4  2 2
(S) 0 0
The value of cos52  cos68  cos172  is 0 (4) 1
8
Codes :
P Q R S
(A) 4 3 1 2
(B) 3 4 1 2
(C) 3 4 2 1
(D) 4 3 2 1

Solution: C
 
(P) sin   cos    cos   sin    sin    sin  
2 
 1  1 1  1
 cos     sin   cos   sin    2 cos   sin   
2 2  2 2  2
   1
cos     
 4 2 2
 
(Q) tan   cos    cot   sin    tan    sin  
 2 

  cos   sin   
2
1
cos   sin  
2
   1   1
sin       sin2     
 4 2 2  4 8
(R) sin2   cos2   1
3

 sin2   cos2   1
 sin   cos   3 sin2 .cos2 . 1  1
6 6

3 2
 sin6   cos6  
sin 2  1
4
(S) cos520  cos 680  cos1720
 2cos1120 cos600  cos 680  cos1120  cos680  2cos900.cos 220  0

P-2022-CBSE-P1-MATHEMATICS-TRI
43
Problem 33. If A + B + C = , then match the following (equality must hold)
Column – I Column – II
(A) cos A + cos B + cos C (p)  3/2
A B C (q)  – 3
(B) sin sin sin
2 2 2
1 3
(C)  cos A cosB  cosBcosC  cosCcos A  (r) 
2 8
(D) cos A cos B cos C (s)  – 1

Solution: (A)  p, q; (B)  s; (C)  p; (D)  s


(A). cosA + cosB + cosC becomes – 3 for A = B = –  and C = 3
so cosA + cosB + cosC  – 3
and using y = cosA + cosB + cosC
C A B C
y = 2sin cos  1  2 sin2
2 2 2
3
we get, y  .
2
A B C
(B). Also, cosA + cosB + cosC = 1 + 4 sin sin sin
2 2 2
A B C 1
 – 1  sin sin sin  .
2 2 2 8
1 3
(C).  cos A cosB  cosBcosC  cosCcos A   equality holds when all of
2 2
cosA, cosB, cosC are – 1.
(D). cosA cosB cosC  – 1 (when all of cosA, cosB, cosC are – 1).

Numerical Based

12   3  3 
Problem 34. If sin   0     and cos        then
13  2 5  2 
65 sin       60 equals;

Solution: 4
sin       sin .cos   cos  sin 
12 3 5 4 56
    
13 5 13 5 65
 65 sin       56

 12 3
Problem 35. If  and  lie between 0 and such that cos       and sin      
2 13 5
then the value of  65 sin 2  56  is

Solution: 0
sin2  sin            
5 4 12 3 56
    
13 5 13 5 65
 65 sin 2  56  0

P-2022-CBSE-P1-MATHEMATICS-TRI
44
+
Problem 36. If A + B = 90° where A, B  R , let maximum value of sinA + sinB is equal to t,
then t2 is ______

Solution: 2
sin A  sinB  A B 
 sin    sin .
2  2  4

2 cos y  1 x y 2
Problem 37. If cosx = where x, y (0, ) let tan  cot is equal to t, then t is
2  cos y 2 2
equal to _______

Solution: 3
 1  tan2 (y / 2) 
2  2  1
1  tan2 (x / 2)  1  tan y / 2 

1  tan2 (x / 2) 1  tan2 y / 2
2
1  tan2 y / 2
y x
3 tan2    tan2  
2  2
x y
tan cot  3 .
2 2

Problem 38. If sinx + siny  cos cosx  x  R, then siny + cos is equal to ___________

Solution: 1
sinx + siny  cos  cosx  x  R.

Let x = 
2
 sin y  1  (sin y = 1)
 1 + sin x  cos  cos x
 cos  cos x  sin x  1
 cos2   1  1
 cos  = 0.

Decimal Type

Problem 39. Determine the smallest positive value of x (in degree) for which
tan(x + 100) = tan(x + 50) tanx tan(x − 50).

Solution: 30.00
tan(x + 100) = tan(x + 50) tanx tan(x − 50).

tan x  1000 

 tan x  500 tan x 

tan x  500

=
  
sin x  1000 cos x  500   sin  x  50  sin x
0

cos  x  100  sin  x  50  cos  x  50  cos x


0 0 0

sin  2x  50   sin150
0
cos50  cos  2x  50 
0 0 0


sin  2x  50   sin150
0
cos  2x  50   cos50
0 0 0

P-2022-CBSE-P1-MATHEMATICS-TRI
45
Use componendo and dividendo

We get

sin 2x  500

  cos500
sin150 0

cos 2x  500 
cos50 + 2 sin(2x + 50 ) cos(2x + 500) = 0
0 0

cos500 + sin(4x + 1000) = 0


cos500 + cos(4x + 100) = 0
0 0
cos(2x + 30 ) cos(2x − 20 ) = 0
so x = 30, 55
Hence the smallest value of x = 30.

2 1
Problem 40. If a = and cosa cos2a cos3a … cos999a = , then log2k = ___________
1999 k

Solution: 999.00
1
= cosa cos2a cos3a … cos999a
k
let p = sina sin2a … sin999a
2999 p
 = (2sina cosa)(2sin2a cos2a)…(2cos999a sin999a)
k
2999 p
= sin2a sin4a … sin1998a
k
= (sin2a sin4a…sin998a)(–sin(2 – 1000a))(–sin(2 – 1002a)) … (–sin(2 –
1998a))
 (sin2a sin4a … sin1998a)(sin999a sin997a … sina)
=p
 2999 = k.

P-2022-CBSE-P1-MATHEMATICS-TRI
46
CHAPTER PRACTICE PROBLEM

Subjective:

Level  I

1. Find the maximum and minimum value of sin2 x + 4cos 2x, for x  R.

 3 5 7 3
2. Prove that sin4  sin4  sin4  sin4  .
16 16 16 16 2
9
r
3. Find the value of  cos2 18 .
r 1

Level  II

1  cos  4
4. Prove that 0   for all .
2  sin  3

5. tan 20 tan 40 tan 60 tan 80  

Objective:

MCQ Single Correct

 x  y 2
1. sin2 = , where x, y  R, gives real  if and only if
4xy
(A) x + y = 0 (B) x = y
(C) |x| = |y|  0 (D) none of these

A B A B
2. Let a = cosA + cosB – cos(A + B) and b = 4 sin . sin . cos . Then a – b is equal
2 2 2
to
(A) 1 (B) 0
(C) – 1 (D) none of these

3. If 3 sin + 4 cos = 5, then 4 sin – 3cos is equal to


(A) 0 (B) 5
(C) 1 (D) none of these

4. If in ABC C = 900, then the maximum value of sin A sin B is


(A) 1/2 (B) 1
(C) 2 (D) 3/4

 
5. If  lies in fourth quadrant, then 4cos4   sin2 2  4 cos2    is equal to
 2 2
(A) 1 (B) 2
(C) –2 (D) 0

P-2022-CBSE-P1-MATHEMATICS-TRI
47
6. If ( +  +  + ) =  then cos  cos  -sin  sin  =
(A) 4 (B) 2
(C) 0 (D) none of these
 
7. If x + y = 2 then minimum value of sec x + sec y is, x, y   0, 
 2
(A) 2 cos  (B) cos 2
(C) 2 sec  (D) none of these

tan700  tan200
8. =
4 tan500
(A) 1 (B) 1/2
(C) –1 (D) –1/2

9. In a triangle ABC maximum value of sin A + sin B + sin C is


3 3 2 3
(A) (B)
2 2
3
(C) 3 3 (D)
2
2 3
10. If 1 + sin  + sin  + sin  + …..to  = 4 + 2 3 , 0 <  < ,   /2 then
 
(A)  = (B)  =
6 3
   2
(C)  = or (D)  = or
6 3 3 3

MCQ Multi Correct

1. If A lies in the second quadrant and 3 tan A  4  0, the value of


2 cot A  5 cos A  sin A is equal to
53 7
(A) (B)
10 10
7 23
(C) (D)
10 10

2. tan 5x tan 3x tan 2x 


sin5x  sin3x  sin2x
(A) tan 5x  tan 3x  tan 2x (B)
cos 5x  cos 3x  cos 2x
(C) 0 (D) None of these

n n
 cos A  cosB   sin A  sinB 
3.     (n even or odd) =
 sin A  sinB   cos A  cosB 
A B A B
(A) 2 tann (B) 2cot n
2 2
(C) 0 (D) None of these

P-2022-CBSE-P1-MATHEMATICS-TRI
48
4. If sin   1/ 5 and sin   3 / 5 ,then    lies in the interval
(A) [0,  / 4] (B) [  / 2,3 / 4]
(C) [3 / 4, ] (D) [ ,5 / 4]

Numerical Based

1. sin 50  sin 70   sin10  


2. If A  B  , then (1  tan A)(1  tanB) 
4

1 3
3.  =
sin10 cos10

4. tan100  tan125  tan100 tan125  

5. The value of cos52  cos 68   cos172 is

 9 3 5
6. The expression 2cos .cos  cos  cos is equal to
13 13 13 13

7. tan 9  tan 27   tan 63   tan81 

cos12  sin12 sin147


8.  
cos12  sin12 cos147

P-2022-CBSE-P1-MATHEMATICS-TRI
49
ASSIGNMENT PROBLEMS

Subjective:

Level  0

cot   cos ec


1. Find the values of T, where T  and  lies in the second quadrant
sec   tan 
3
with sin   .
5

tanA + secA - 1 1+ sinA cot   cos ec  1


2. Prove that (i) = (ii)  cos ec  cot 
tanA - secA + 1 cosA cot   cos ec  1

17
3. Prove that – 4 cos2x + 3sinx  for all real x.
8

2b
4. If tan x  (a  c), y  a cos2 x  2b sin x cos x  c sin2 x ,
ac
z  a sin2 x  2b sin x cos x  c cos2 x , prove that y – z = a – c.

3 5
5. If cos  = - and sin    , and  lies in the third quadrant and  lies in the fourth
5 13
quadrant, find tan( + ), cot( – ).

6. Show that 3 cosec20  sec 20  4 .

  5   9   13  1
7. Prove that  1  sin   1  sin   1  sin   1  sin  .
 8  8  8  8  8

0 0
log10 tan30 log10 tan 890
8. Find the value of elog10 tan1 log10 tan 2 .

1
9. Find the minimum value of the expression .
    
cos2   x   sin2   x 
4  4 

10. If sinx + siny + sinz = 3; x , y, z  [0, 2], then show that


2 2 2
(a) x + y + z – xy – yz – zx = 0
3
(b) x 3 + y 3 – z3 = .
8

 3 5 7 3
11. Prove that sin4  sin4  sin4  sin4  .
16 16 16 16 2

3
12. Prove that sin 20 sin 40 sin 80 = .
8

P-2022-CBSE-P1-MATHEMATICS-TRI
50
m  n
13. If m tan (  30°) = n tan ( + 120°), show that cos2 = .
2 m  n

1
14. Prove that 2 sinA cos3A  2sin3A cosA = sin 4A.
2

sin   sin 2
15. Prove that  tan  .
1  cos   cos 2

16. Prove that 1+cos56° + cos58°  cos66° = 4cos28° cos29°sin33°.

17. Prove that 2cosx  cos3x  cos5x = 16cos3x sin2x.

 3 5 7
18. Pr ove that sin4  sin4  sin4  sin4  3/ 2.
8 8 8 8

P-2022-CBSE-P1-MATHEMATICS-TRI
51
Level  I

1. If ,  are the solutions of acos + bsin = c, show that


2bc c 2  a2
(i) sin   sin   , sin  sin   ,
a2  b2 a2  b2
 b
(ii) tan 
2 a

 3 5 7
2. Find the value of cos cos cos cos .
8 8 8 8

3. If  +  = /2 and  +  = , then prove that tan  = tan + 2tan.

7   x
4. If sinx + cosx = where x  0,  , then find the value of tan .
2  4 2

5. If A + B + C =  and A, B, C are acute angles, then prove that


tanA + tanB + tanC  3 3 .

cos6x  6 cos 4x  15cos2x  10


6. Prove that  2cos x .
cos5x  5cos3x  10cos x

tan        tan 
7. If  , prove that sin2 + sin2 + sin2 = 0.
tan        tan 

8. Find the least value of 2sinx + 2cosx.

9. Prove that the expression cos2 + cos2( + )  2 cos  cos  cos ( + ) is independent
of .

10. Without using tables, show that


(i) tan9°  tan27°  tan63° + tan81° = 4
(ii) tan20° tan40° tan60° tan80° = 3
tan 69  tan 66
(iii)  1
1  tan 69 tan 66

11. If  +  = , show that cos2 + cos2 + cos2 = 1 + 2coscoscos.


12. If cos ( + ) = 4/5, sin (  ) = 5/13 and ,  lie between 0 and , find tan 2.
4

P-2022-CBSE-P1-MATHEMATICS-TRI
52
Level  II

1. The angles of a quadrilateral are in G.P. with common ratio r > 1. If the largest angle is 8
times the smallest angle, find the angles of the quadrilateral in degrees.

1 cos 3x
2. Show that  cos ec2x  cos ec4x   ,
2 sin x sin 2x sin 4x
cos 3x cos 5x cos 7x
and hence prove that    ...  n terms
sin 2x sin 4x sin 4x sin 6x sin 6x sin 8x
1
 cos ec2x  cos ec(2n  2)x  .
2 sin x

3. If sin(z + y - x), sin(x + z - y), sin(y + x - z) be in A.P., prove that tanx, tany, tanz are also
in A.P.

4. If sin + sin = a, cos + cos = b, find the value of


(i) sin( + ), (ii) cos2 + cos2. (where |b| > |a|)

1 7
5. If sinx cosy = and 3 tanx = 4 tany then show that sin(x + y) = .
4 16

3
6. If sin( cos) = cos( sin), then show that sin2 =  .
4

acos+b  ab 
7. If cos= ,prove that tan  tan .
a+bcos 2 ab 2

3
8. If A + B + C = , prove that cosA + cosB + cosC  and hence deduce that
2
A B C 1
sin sin sin  .
2 2 2 8

tan 3A sin 3A 2k 1
9. If  k , prove that  and hence deduce that either k > 3 or k < .
tan A sin A k 1 3

10. If a sin2 + b cos2 = m, b sin2 + a cos2 = n and a tan  = b tan, prove that
1 1 1 1
   .
n m a b

11. If cosec   sin = m, sec  cos = n, eliminate .

P-2022-CBSE-P1-MATHEMATICS-TRI
53

Objective:

Level  I

1. If tan  = -4/3, then sin is


4 4 4 4
(A) – but not (B) – or
5 5 5 5
4 4
(C) but not – (D) none of these
5 5

2. If tanx. tany = a and x + y = /6, then tanx and tany satisfy the equation
(A) x2  3 (1 – a)x + a = 0 (B) 3 x2 – (1 – a)x + a 3 = 0
2 2
(C) x + 3 (1 + a)x - a = 0 (D) 3 x + (1 + a)x – a 3 = 0

3. sin .(sin  + sin 3) is


(A)  0 for all  (B)  0 only when   0
(C)  0 for all  (D)  0 only when   0

4. If sin ( + ) = 1, sin(  ) = 1/ 2, where ,   [0, /2], then tan( + 2) tan(2 + ) is


equal to
(A) 1 (B) 0
(C) –1 (D) none of these

2
 1  sin   cos  
5.   is equal to
 1  sin   cos  
 1  sin 
(A) cot2 (B)
2 1  sin 
(C) tan2/2 (D) none of these

6. k is rational if it is equal to
(A) sin 150 sin 300 (B) sin 150 cos 150
(C) sin 150 cos 750 (D) none of these

7. If tan, tan are the roots of the equation x2 + px + q = 0 (p  0) , then


(A) sin(+) = p (B) tan(+) = p/ (q1)
(C) cos (+) = 1 – q (D) none of these

1  3 3  2   4 
8. sin   sin      sin3      is equal to
sin 3   3   3 
(A) 4/3 (B) 3/4
(C) 3/4 (D) none of these

1  sin 4A  1
9. If y = , then one of the value of y is
1  sin 4A  1
(A) tanA (B) cotA
(C) tan(/4  A) (D) cot(/4 + A)

P-2022-CBSE-P1-MATHEMATICS-TRI
54
10. The number of solutions of the equation sin (ex) = 5x + 5–x is
(A) 0 (B) 1
(C) 2 (D) none of these

11. The value of tan 750 –cot 750 is equal to


(A) 1 + 2 3 (B) 2 + 3
(C) 2 – 3 (D) 2 3

12. If A = sin2  + cos4  then for all real values of 


(A) 1  A  2 (B) 3/4  A  1
(C) 13/16  A  1 (D) 3/4  A  13/16

13. The value of the expression tan1 tan2tan3  tan89 is equal to


(A) 0 (B) not defined
(C) 1 (D) none of these

14. The least value of cos2 – 6 sin cos + 3 sin2 + 2 is


(A) 4 + 10 (B) 4 – 10
(C) 0 (D) none of these

 3 4
15. If 0 <  <   , cos( + ) = and cos( – ) = , then sin2 is equals
4 5 5
(A) 1 (B) 0
(C) 2 (D) none of these

x2  y2
16. The equation sin2  = is possible if
2 xy
(A) x = y (B) x = -y
(C) 2x = y (D) none of these

17. Minimum value of the expression 2 sin x + 4 cos x + 3 5 is


(A) 5 5 (B) 2 5 + 3
(C) 2 5  3 (D) none of these

1 1
18. If tan  = , tan  = , then  +  is equal to
2 3
(A) 0 (B) /2
(C) /4 (D) 

19. The value of cos 100 –sin 100 is


(A) positive (B) negative
(C) 0 (D) 1

tan690  tan660
20. =
1  tan690 tan660
(A) 1 (B) –1
(C) 0 (D) none of these

P-2022-CBSE-P1-MATHEMATICS-TRI
55
Level  II

Multi-choice (Single correct answers):

1. If sin + cosec = 2, then the value of sinn + cosecn, n  2, n  N equals


(A) 2 (B) 2n
(C) 1 (D) none of these

   
2. The maximum value of 1 + sin     + 2 cos     ,   R, equals
 4   4 
(A) 3 (B) 5
(C) 4 (D) none of these

 5 7
3. The numerical value of sin . sin . sin is equal to
18 18 18
1
(A) 1 (B)
8
1
(C) (D) none of these
4

   
4. If tan. tan     . tan     = -1, ( 0 <  < /2), then value of 3 sin - 4 cos3 =
 3   3 
(A) 1 (B) -1
(C) 1/ 2 (D) -1/ 2

5. If  and  are two distinct roots of the equation a tan x + b sec x = c, then tan ( + ) is
equal to
a2  c 2 a2  c 2
(A) 2 2
(B) 2
a c a  c2
2ac 2ac
(C) 2 2
(D) 2
a c a  c2

6. If sin  = 3 sin ( + 2) then value of tan( + ) + 2 tan  is


(A) 3 (B) 2
(C) 1 (D) 0

7. In a ABC, if cotA cotB cotC > 0, then the  is


(A) acute angled (B) right angled
(C) obtuse angled (D) does not exist
8. The expression
  3      
3  sin4      sin4  3      2  sin6      sin6  5    is equal to
  2    2  
(A) 0 (B) 1
(C) sin4 + sin6 (D) none of these.

y
9. If cos (x – y), cos x and cos (x + y) are in H.P., then cos x sec is equal to
2
(A)  1 (B)  3
(C)  2 (D) none of these

P-2022-CBSE-P1-MATHEMATICS-TRI
56
2cos-1  
10. If cos= ,  0    ,0      , then tan cot is equal to
2-cos 2 2
(A) 1 (B) 2
(C) 3 (D) none of these

cos sin  p q
11. If  ,then  is equal to
p q sec 2 cosec2
(A) p (B) q
(C) qp (D) none of these

12. For 0  1, 2, …, n  /2 and cot 1 cot 2 … cot n = 1, the maximum value of
(cos 1) (cos 2) (cos 3) … (cos n) is
(A) 1/2n (B) 1/2n/2
(C) 21/n (D) none of these

13. If (m + 2)sin + (2m – 1)cos = 2m+1, then


2m
(A) tan = 3/ 4 (B) tan =
m2  1
2m
(C) tan = (D) none of these
m2  1

14. Maximum and minimum values of sin4 + cos4 are


1
(A) 0, 2 (B) 1,
2
(C) –1, 1 (D) none of these

Multi-choice (One or More correct answers):

15. If sinB is the geometric mean between sinA and cosA, then cos2B is equal to
   
(A) 2sin2   A  (B) 2cos2   A 
 4   4 
2  2 
(C) 2cos   A  (D) 2sin   A 
4  4 

 2   4 
16. Given a, b, c are non-zero. If asin  = b sin      c sin     ,   then
 3   3 
(A) ab + bc + ca = 1 (B) ab + bc + ca = 0
1 1 1 1 1 1
(C)    ab  bc  ca (D)    1
a b c a b c

1
17. If tan   x  , then sec  tan is equal to
4x
1
(A) 2x (B) 
2x
1
(C) 2x (D)
2x

P-2022-CBSE-P1-MATHEMATICS-TRI
57
 
18. If tan  (cosec  sin ) , then tan2 can be equal to
2 2
(A) 2  5 (B) 2  5
 
(C) 9  4 5 2  5   
(D) 9  4 5 2  5 
10
2
19. If R [0, ] for 1  k  10, then the maximum value of  (1  sin R )(1  cos2 R ) is
R1
10 10
3  9
(A)   (B)  
 2  4
20 5
3 9
(C)   (D)  
 2  4


20. If x = Xcos  Ysin, y = Xsin + Ycos and x2 + 4xy + y2 = AX2 + BY2 0    , then
2
 
(A)   (B)  
6 4
(C) A = B (D) B = 1

21. The quadratic equation whose roots are sec2 and cosec2 can be
(A) x2  2x + 2 = 0 (B) x2  5x + 5 = 0
2
(C) x  7x + 7 = 0 (D) x2  9x + 9 = 0

22. If , ,  are acute angle such that cos  = tan , cos  = tan  and cos  = tan , then
(A) sin  = sin  (B) sin  = sin 
5 1
(C) sin  = (D) none of these
2

23. If A = sinx cos3x and B = cosx sin3x, then


   
(A) A  B > 0  x   0,  (B) A  B < 0  x   0, 
 4  4
   
(C) A + B > 0  x   0,  (D) A + B < 0  x   0, 
 2  4

Assertion / Reason:
Questions (12–13) contain STATEMENT-1 (Assertion) and STATEMENT-2 (Reason).
(A) Statement -1 is True, Statement -2 is True; Statement-2 is a correct explanation for
Statement-1
(B) Statement -1 is True, Statement -2 is True; Statement-2 is NOT a correct explanation
for Statement-1
(C) Statement -1 is True, Statement -2 is False
(D) Statement -1 is False, Statement -2 is True

24. STATEMENT -1: If A, B, C are the angles of a triangle such that angle A is obtuse then
tan B tan C > 1.
because
tanB  tan C
STATEMENT -2: In any triangle tan A = .
tanB tan C  1

P-2022-CBSE-P1-MATHEMATICS-TRI
58
25. STATEMENT -1: The minimum value of the expression sin  + sin  + sin , where , , 
are real numbers such that  +  +  =  is negative.
because
STATEMENT -2: , ,  are angles of a triangle.

Comprehension:

Read the following passage and answer the questions that follow:
n n
If Pn = sin  + cos , where n is whole number and   R, then

26. If Pn2  Pn = sin2cos2P, then the value of  is


(A) n  1 (B) n  2
(C) n  3 (D) n  4

P7  P5
27. The value of is
P5  P3
P7 P5
(A) (B)
P5 P3
P3 P3
(C) (D)
P1 P5

28. The value of 2P6  3P4 + 10 is


(A) 0 (B) 6
(C) 9 (D) 15

Read the following passage and answer the questions that follow

ABC is a triangle, the incircle touches the sides BC, CA and AB at D, E and F respectively. BD,
CE and AF are consecutive natural numbers. I is the incentre of the triangles. The radius of the
incircle is 4 units.

29. Sides of the triangle ABC are


(A) 11, 12, 13 (B) 12, 13, 14
(C) 13, 14, 15 (D) 14, 15, 16

30. Angles of the triangle DEF are


(A)   2A,   2B,   2C (B)   A,   B,   C
A B C  A B C
(C) , , (D) , ,
2 2 2 2 2 2

A B C
31. cot  cot  cot is equal to
2 2 2
(A) 7/4 (B) 3/4
(C) 21/4 (D) ½

P-2022-CBSE-P1-MATHEMATICS-TRI
59
Match Matrix:

32. Match the following:


ABC is a right angled triangle at C and the angles A, B, C are in A.P.
Column  I Column  II
(A) sin A + sin B + sin C (p) 1
(B) cos A + cos B + cos C (q) 3
(C) sin 2A + sin 2B + sin 2C
(r)
3  3 
2
(D) cot B cot C + cot C cot A + cot A cot B
(s)
 3 1 
2

33. Matching List Type(3 Column & 4 Rows)


Column–I Column–II Column–III
3log 4 + 2 log 5 – sin3  sin5  sin7
(I) Value of if
1 1 3 cos3   cos5  cos7
log64  log16 where base (i) (P)
3 2 2 

of the logarithm is 10 equal 20
2log 3 – log 5. 3log 5 – log 2. 5log 2 – log 3
(II) (ii) 2 (Q) cos2 10  cos2 130  cos2 110
equals.
4 times the value of the
expression
Absolute values of solution of  3   3 
equation cos2    sin2   
(III) (iii) 4 (R)  2   2 
2 3 log2 49 
72x 5x 6  2  2  2
cosec      1 sec      1

2  2 

If log2 27  a and
Cube root of the reciprocal of
3a (S)
(IV) log6 16  k   then k is a (iv) 1  2 4
3a cos .cos .cos
9 9 9
positive integer number.
(I). Which of the following options is the only CORRECT combination?
(A) (II) (iv) (P) (B) (II) (iv) (Q)
(C) (II) (iii) (P) (D) (II) (ii) (Q)
(II). Which of the following options is the only CORRECT combination?
(A) (III) (i) (Q) (B) (III) (iii) (P)
(C) (III) (i) (P) (D) (III) (i) (S)
(III). Which of the following options is the only INCORRECT combination?
(A) (IV) (iii) (R) (B) (IV) (iii) (P)
(C) (I) (ii) (S) (D) (II) (ii) (P)

P-2022-CBSE-P1-MATHEMATICS-TRI
60
Numerical Based


34. Let 0 < A, B < satisfying the equalities 3sin2A + 2sin2B = 1 and 3sin2A – 2sin2B = 0.
2

Then A + 2B = , where k is equal to _____________.
k

35. If A lies in the second quadrant and 3tanA + 4 = 0, then the value of
10
(2cotA – 5cosA + sinA) is equal to ____________
23

36. If 0  a  3, 0  b  3 and the equation x2 + 4 + 3 cos (ax + b) = 2x has atleast one


solution, and the value of a + b is k/4, then minimum positive value of k is _________.

Decimal Type

a
1  cosa  tan2  
37. If  2   k cosa where k, w and p have no common factor other than
a w  pcosa
sin2  
2
1, then the value k2 + w2 + p2 is equal to

38. If y = (sin x + cosec x)2 + (cos x + sec x)2 + (tan x + cot x)2
p
where x  R – {x ; x = n, (2n + 1)/2  n  I} and minimum value of y = p then   is
3
where [.] denotes the greatest function

1
39. If , , ,  be first 4 positive solutions of sin x = with  <  <  < , then value of
4
   
sin  2 sin  3 sin  4 sin is
2 2 2 2

P-2022-CBSE-P1-MATHEMATICS-TRI
61
ANSWERS TO CHAPTER PRACTICE PROBLEMS

Subjective:

Level  I

1. minimum value is  3 and maximum value is 4

3. 4

5. 3

Level  II

Objective:

MCQ Single Correct

1. C 2. A 3. A 4. A
5. B 6. C 7. C 8. B
9. A 10. D

MCQ Multi Correct

1. D 2. A, C 3. B, C 4. A, C

Numerical Based

1. 0 2. 2 3. 4 4. 1

5. 0 6. 0 7. 4 8. 0

P-2022-CBSE-P1-MATHEMATICS-TRI
62
ANSWERS TO ASSIGNMENT PROBLEMS

Subjective:

Level  0

3
1. 8. 1
2

33 16 1
5. , 9.
56 63 2

Level  I

1 7 1
2. 4.
8 3

1
1
2 56
8. 2 12.
33

Level  II

1. 24, 48, 96, 192 .

3. (i)
2ab
, (ii)
b 2
 a2 b
2
 a2  2 
a2  b2 a2  b2

10. n2/3m4/3 + n4/3m2/3 = 1

Objective:

Level  I

1. B 2. B 3. A 4. A
5. C 6. B 7. B 8. C
9. B 10. A 11. D 12. B
13. C 14. B 15. A 16. A
17. D 18. C 19. A 20. B

Level  II

1. A 2. C 3. B 4. C
5. D 6. D 7. A 8. B
9. C 10. C 11. A 12. B

P-2022-CBSE-P1-MATHEMATICS-TRI
63
13. C 14. B 15. A, C 16. B, C
17. A, D 18. B, C 19. B, C 20. B, C, D
21. B, C, D 22. A, B, C 23. A, C 24. D
25. C 26. D 27. C 28. C
29. C 30. D 31. C
32. (A)  (r) (B)  (s) (C)  (q) (D)  (p)
33. (I). A (II). A (III). D
34. 2 35. 1 36. 4 37. 6.00
38. 4.00 39. 2.24

P-2022-CBSE-P1-MATHEMATICS-TRI

You might also like